SlideShare a Scribd company logo
1 of 206
Download to read offline
A 43 year old man is stabbed outside a nightclub. He suffers a transection of his median nerve just as it leaves the brachial plexus.
Which of the following features is least likely to ensue?
A. Ulnar deviation of the wrist
B. Complete loss of wrist flexion
C. Loss of pronation
D. Loss of flexion at the thumb joint
E. Inability to oppose the thumb
Next question
Loss of the median nerve will result in loss of function of the flexor muscles. However, flexor carpi ulnaris will still function and produce
ulnar deviation and some residual wrist flexion. High median nerve lesions result in complete loss of flexion at the thumb joint.
Median nerve
The median nerve is formed by the union of a lateral and medial root respectively from the lateral (C5,6,7) and medial (C8 and T1)
cords of the brachial plexus; the medial root passes anterior to the third part of the axillary artery. The nerve descends lateral to the
brachial artery, crosses to its medial side (usually passing anterior to the artery). It passes deep to the bicipital aponeurosis and the
median cubital vein at the elbow.
It passes between the two heads of the pronator teres muscle, and runs on the deep surface of flexor digitorum superficialis (within its
fascial sheath).
Near the wrist it becomes superficial between the tendons of flexor digitorum superficialis and flexor carpi radialis, deep to palmaris
longus tendon. It passes deep to the flexor retinaculum to enter the palm, but lies anterior to the long flexor tendons within the carpal
tunnel.
Branches
Region Branch
Upper arm No branches, although the nerve commonly communicates with the musculocutaneous nerve
Forearm Pronator teres
Flexor carpi radialis
Palmaris longus
Flexor digitorum superficialis
Flexor pollicis longus
Flexor digitorum profundus (only the radial half)
Distal
forearm
Palmar cutaneous branch
Hand (Motor) Motor supply (LOAF)
 Lateral 2 lumbricals
 Opponens pollicis
 Abductor pollicis brevis
 Flexor pollicis brevis
Hand
(Sensory)
 Over thumb and lateral 2 ½ fingers
 On the palmar aspect this projects proximally, on the dorsal aspect only the distal regions are innervated with the radial nerve
providing the more proximal cutaneous innervation.
Patterns of damage
Damage at wrist
 e.g. carpal tunnel syndrome
 paralysis and wasting of thenar eminence muscles and opponens pollicis (ape hand deformity)
 sensory loss to palmar aspect of lateral (radial) 2 ½ fingers
Damage at elbow, as above plus:
 unable to pronate forearm
 weak wrist flexion
 ulnar deviation of wrist
Anterior interosseous nerve (branch of median nerve)
 leaves just below the elbow
 results in loss of pronation of forearm and weakness of long flexors of thumb and index finger
Topography of the median nerve
A 24 year old man falls and sustains a fracture through his scaphoid bone. From which of the following areas does the scaphoid
derive the majority of its blood supply?
A. From its proximal medial border
B. From its proximal lateral border
C. From its proximal posterior surface
D. From the proximal end
E. From the distal end
Next question
Theme from April 2012 Exam
The blood supply to the scaphoid enters from a small non articular surface near its distal end. Transverse fractures through the
scaphoid therefore carry a risk of non union.
Scaphoid bone
The scaphoid has a concave articular surface for the head of the capitate and at the edge of this is a crescentic surface for the
corresponding area on the lunate.
Proximally, it has a wide convex articular surface with the radius. It has a distally sited tubercle that can be palpated. The remaining
articular surface is to the lateral side of the tubercle. It faces laterally and is associated with the trapezium and trapezoid bones.
The narrow strip between the radial and trapezial surfaces and the tubercle gives rise to the radial collateral carpal ligament. The
tubercle receives part of the flexor retinaculum. This area is the only part of the scaphoid that is available for the entry of blood
vessels. It is commonly fractured and avascular necrosis may result.
A 56 year old man requires long term parenteral nutrition
and the decision is made to insert a PICC line for long
term venous access. This is inserted into the basilic vein
at the region of the elbow. As the catheter is advanced,
into which venous structure is the tip of the catheter most
likely to pass from the basilic vein?
A. Subclavian vein
B. Axillary vein
C. Posterior circumflex humeral vein
D. Cephalic vein
E. Superior vena cava
Next question
The basilic vein drains into the axillary vein and although
PICC lines may end up in a variety of fascinating locations
the axillary vein is usually the commonest site following
from the basilic. The posterior circumflex humeral vein is
encountered prior to the axillary vein. However, a PICC
line is unlikely to enter this structure because of its angle
of entry into the basilic vein.
Basilic vein
The basilic and cephalic veins both provide the main
pathways of venous drainage for the arm and hand. It is
continuous with the palmar venous arch distally and the
axillary vein proximally.
Path
 Originates on the medial side of the dorsal venous
network of the hand, and passes up the forearm and
arm.
 Most of its course is superficial.
 Near the region anterior to the cubital fossa the vein
joins the cephalic vein.
 Midway up the humerus the basilic vein passes deep
under the muscles.
 At the lower border of the teres major muscle, the
anterior and posterior circumflex humeral veins feed
into it.
 Joins the brachial veins to form the axillary vein.
Theme: Nerve injury
A. Ulnar nerve
B. Musculocutaneous nerve
C. Radial nerve
D. Median nerve
E. Axillary nerve
F. Intercostobrachial nerve
What is the most likely nerve injury for the scenario given?
Each option may be used once, more than once or not at
all.
16. A 23 year old man is involved in a fight outside a
nightclub and sustains a laceration to his right arm. On
examination he has lost extension of the fingers in his
right hand.
Radial nerve
The radial nerve supplies the extensor muscle group.
17. A 40 year old lady trips and falls through a glass door
and sustains a severe laceration to her left arm. Amongst
her injuries it is noticed that she has lost the ability to
adduct the fingers of her left hand.
Ulnar nerve
The interossei are supplied by the ulnar nerve.
18. A 28 year old rugby player injures his right humerus and
on examination is noted to have a minor sensory deficit
overlying the point of deltoid insertion into the humerus.
Axillary nerve
This patch of skin is supplied by the axillary nerve
Next question
Brachial plexus
Origin Anterior rami of C5 to T1
Sections of the
plexus
 Roots, trunks, divisions, cords, branches
 Mnemonic:Real Teenagers Drink Cold Beer
Roots  Located in the posterior triangle
 Pass between scalenus anterior and medius
Trunks  Located posterior to middle third of clavicle
 Upper and middle trunks related superiorly to the
subclavian artery
 Lower trunk passes over 1st rib posterior to the
subclavian artery
Divisions Apex of axilla
Cords Related to axillary artery
Diagram illustrating the branches of the brachial plexus
Image sourced from Wikipedia
Cutaneous sensation of the upper limb
Image sourced from Wikipedia
A 21 year old man is stabbed in the antecubital fossa. A
decision is made to surgically explore the wound. At
operation the surgeon dissects down onto the brachial
artery. A nerve is identified medially, which nerve is it likely
to be?
A. Radial
B. Recurrent branch of median
C. Anterior interosseous
D. Ulnar
E. Median
Next question
Theme from September 2012 Exam
Median nerve
The median nerve is formed by the union of a lateral and
medial root respectively from the lateral (C5,6,7) and
medial (C8 and T1) cords of the brachial plexus; the
medial root passes anterior to the third part of the axillary
artery. The nerve descends lateral to the brachial artery,
crosses to its medial side (usually passing anterior to the
artery). It passes deep to the bicipital aponeurosis and the
median cubital vein at the elbow.
It passes between the two heads of the pronator teres
muscle, and runs on the deep surface of flexor digitorum
superficialis (within its fascial sheath).
Near the wrist it becomes superficial between the tendons
of flexor digitorum superficialis and flexor carpi radialis,
deep to palmaris longus tendon. It passes deep to the
flexor retinaculum to enter the palm, but lies anterior to the
long flexor tendons within the carpal tunnel.
Branches
Region Branch
Upper arm No branches, although the nerve commonly communicates
with the musculocutaneous nerve
Forearm Pronator teres
Flexor carpi radialis
Palmaris longus
Flexor digitorum superficialis
Flexor pollicis longus
Flexor digitorum profundus (only the radial half)
Distal
forearm
Palmar cutaneous branch
Hand
(Motor)
Motor supply (LOAF)
 Lateral 2 lumbricals
 Opponens pollicis
 Abductor pollicis brevis
 Flexor pollicis brevis
Hand
(Sensory)
 Over thumb and lateral 2 ½ fingers
 On the palmar aspect this projects proximally, on the
dorsal aspect only the distal regions are innervated with
the radial nerve providing the more proximal cutaneous
innervation.
Patterns of damage
Damage at wrist
 e.g. carpal tunnel syndrome
 paralysis and wasting of thenar eminence muscles
and opponens pollicis (ape hand deformity)
 sensory loss to palmar aspect of lateral (radial) 2 ½
fingers
Damage at elbow, as above plus:
 unable to pronate forearm
 weak wrist flexion
 ulnar deviation of wrist
Anterior interosseous nerve (branch of median nerve)
 leaves just below the elbow
 results in loss of pronation of forearm and weakness
of long flexors of thumb and index finger
Topography of the median nerve
Image sourced fro
A man sustains a laceration between the base of the little
finger and wrist. Several weeks after the injury there is
loss of thumb adduction power. Which nerve is most likely
to have been injured?
A. Superficial ulnar nerve
B. Deep ulnar nerve
C. Median nerve
D. Radial nerve
E. Recurrent branch of median nerve
Next question
Theme from 2009 Exam
Ulnar nerve injury at wrist
Branches of the ulnar nerve in the wrist and hand
At the wrist the ulnar nerve divides into superficial and
deep branches. The superficial branch lies deep to the
palmaris brevis. It divides into two; to produce digital
nerves, which innervate the skin of the medial third of the
palm and the palmar surface of one and a half fingers.
The deep branch arises from the nerve on the flexor
retinaculum lateral to the pisiform bone. It passes
posteriorly between the abductor and short flexor of the
little finger supplying them, and supplying and piercing the
opponens digiti minimi near its origin from the flexor
retinaculum, turns laterally over the distal surface of the
Hook of the Hamate bone. It eventually passes between
the two heads of adductor pollicis with the deep palmar
arch and ends in the first dorsal interosseous muscle. In
the palm the deep branch also innervates the lumbricals
and interosseous muscles.
A 25 year old man is stabbed in the upper arm. The
brachial artery is lacerated at the level of the proximal
humerus, and is being repaired. A nerve lying immediately
lateral to the brachial artery is also lacerated. Which of the
following is the nerve most likely to be?
A. Ulnar nerve
B. Median nerve
C. Radial nerve
D. Intercostobrachial nerve
E. Axillary nerve
Next question
The brachial artery begins at the lower border of teres
major and terminates in the cubital fossa by branching into
the radial and ulnar arteries. In the upper arm the median
nerve lies closest to it in the lateral position. In the cubital
fossa it lies medial to it.
Image sourced from Wikipedia
Brachial artery
The brachial artery begins at the lower border of teres
major as a continuation of the axillary artery. It terminates
in the cubital fossa at the level of the neck of the radius by
dividing into the radial and ulnar arteries.
Relations
Posterior relations include the long head of triceps with the
radial nerve and profunda vessels intervening. Anteriorly it
is overlapped by the medial border of biceps.
It is crossed by the median nerve in the middle of the arm.
In the cubital fossa it is separated from the median cubital
vein by the bicipital aponeurosis.
The basilic vein is in contact at the most proximal aspect
of the cubital fossa and lies medially.
What is the course of the median nerve relative to the
brachial artery in the upper arm?
A. Medial to anterior to lateral
B. Lateral to posterior to medial
C. Medial to posterior to lateral
D. Medial to anterior to medial
E. Lateral to anterior to medial
Next question
Relations of median nerve to the brachial artery:
Lateral -> Anterior -> Medial
Theme from 2009 and 2012 Exams
The median nerve descends lateral to the brachial artery,
it usually passes anterior to the artery to lie on its medial
side. It passes deep to the bicipital aponeurosis and the
median cubital vein at the elbow. It enters the forearm
between the two heads of the pronator teres muscle.
Image sourced from Wikipedia
Brachial artery
The brachial artery begins at the lower border of teres
major as a continuation of the axillary artery. It terminates
in the cubital fossa at the level of the neck of the radius by
dividing into the radial and ulnar arteries.
Relations
Posterior relations include the long head of triceps with the
radial nerve and profunda vessels intervening. Anteriorly it
is overlapped by the medial border of biceps.
It is crossed by the median nerve in the middle of the arm.
In the cubital fossa it is separated from the median cubital
vein by the bicipital aponeurosis.
The basilic vein is in contact at the most proximal aspect
of the cubital fossa and lies medially.
A 22 year old falls over and lands on a shard of glass. It
penetrates the palmar aspect of his hand, immediately
lateral to the pisiform bone. Which of the following
structures is most likely to be injured?
A. Palmar cutaneous branch of the median nerve
B. Lateral tendons of flexor digitorum superficialis
C. Ulnar artery
D. Flexor carpi radialis tendons
E. Lateral tendons of flexor digitorum profundus
Next question
The ulnar nerve and artery are at most immediate risk in
this injury. This is illustrated in the image below:
Image sourced from Wikipedia
Hand
Anatomy of the hand
Bones  8 Carpal bones
 5 Metacarpals
 14 phalanges
Intrinsic
Muscles
7 Interossei - Supplied by ulnar nerve
 3 palmar-adduct fingers
 4 dorsal- abduct fingers
Intrinsic
muscles
Lumbricals
 Flex MCPJ and extend the IPJ.
 Origin deep flexor tendon and insertion dorsal
extensor hood mechanism.
 Innervation: 1st and 2nd- median nerve, 3rd and
4th- deep branch of the ulnar nerve.
Thenar
eminence
 Abductor pollicis brevis
 Opponens pollicis
 Flexor pollicis brevis
Hypothenar
eminence
 Opponens digiti minimi
 Flexor digiti minimi brevis
 Abductor digiti minimi
A motorcyclist is involved in a road traffic accident. He
suffers a complex humeral shaft fracture which is plated.
Post operatively he complains of an inability to extend his
fingers. Which of the following structures is most likely to
have been injured?
A. Ulnar nerve
B. Radial nerve
C. Median nerve
D. Axillary nerve
E. None of the above
Next question
Mnemonic for radial nerve muscles: BEST
B rachioradialis
E xtensors
S upinator
T riceps
The radial nerve is responsible for innervation of the
extensor compartment of the forearm.
Radial nerve
Continuation of posterior cord of the brachial plexus (root
values C5 to T1)
Path
 In the axilla: lies posterior to the axillary artery on
subscapularis, latissimus dorsi and teres major.
 Enters the arm between the brachial artery and the
long head of triceps (medial to humerus).
 Spirals around the posterior surface of the humerus
in the groove for the radial nerve.
 At the distal third of the lateral border of the humerus
it then pierces the intermuscular septum and
descends in front of the lateral epicondyle.
 At the lateral epicondyle it lies deeply between
brachialis and brachioradialis where it then divides
into a superficial and deep terminal branch.
 Deep branch crosses the supinator to become the
posterior interosseous nerve.
In the image below the relationships of the radial nerve
can be appreciated
Image sourced from Wikipedia
Regions innervated
Motor (main
nerve)
 Triceps
 Anconeus
 Brachioradialis
 Extensor carpi radialis
Motor
(posterior
 Extensor carpi ulnaris
 Extensor digitorum
interosseous
branch
 Extensor indicis
 Extensor digiti minimi
 Extensor pollicis longus and brevis
 Abductor pollicis longus
Sensory The area of skin supplying the proximal
phalanges on the dorsal aspect of the hand
is supplied by the radial nerve (this does
not apply to the little finger and part of the
ring finger)
Muscular innervation and effect of denervation
Anatomical
location
Muscle affected Effect of paralysis
Shoulder Long head of
triceps
Minor effects on
shoulder stability in
abduction
Arm Triceps Loss of elbow extension
Forearm Supinator
Brachioradialis
Extensor carpi
radialis longus and
brevis
Weakening of
supination of prone
hand and elbow flexion
in mid prone position
The cutaneous sensation of the upper limb- illustrating the
contribution of the radial nerve
A 48 year old lady is undergoing an axillary node
clearance for breast cancer. Which of the structures listed
below are most likely to be encountered during the axillary
dissection?
A. Cords of the brachial plexus
B. Thoracodorsal trunk
C. Internal mammary artery
D. Thoracoacromial artery
E. None of the above
Next question
Beware of damaging the thoracodorsal trunk if a latissimus dorsi
flap reconstruction is planned.
Theme from 2009 Exam
The thoracodorsal trunk runs through the nodes in the
axilla. If injured it may compromise the function and blood
supply to latissimus dorsi, which is significant if it is to be
used as a flap for a reconstructive procedure.
Axilla
Boundaries of the axilla
Medially Chest wall and Serratus anterior
Laterally Humeral head
Floor Subscapularis
Anterior aspect Lateral border of Pectoralis major
Fascia Clavipectoral fascia
Content:
Long thoracic nerve
(of Bell)
Derived from C5-C7 and passes behind the
brachial plexus to enter the axilla. It lies on the
medial chest wall and supplies serratus anterior.
Its location puts it at risk during axillary surgery
and damage will lead to winging of the scapula.
Thoracodorsal nerve
and thoracodorsal
Innervate and vascularise latissimus dorsi.
trunk
Axillary vein Lies at the apex of the axilla, it is the continuation
of the basilic vein. Becomes the subclavian vein
at the outer border of the first rib.
Intercostobrachial
nerves
Traverse the axillary lymph nodes and are often
divided during axillary surgery. They provide
cutaneous sensation to the axillary skin.
Lymph nodes The axilla is the main site of lymphatic drainage
for the breast.
53 year old lady is recovering following a difficult
mastectomy and axillary nodal clearance for carcinoma of
the breast. She complains of shoulder pain and on
examination has obvious winging of the scapula. Loss of
innervation to which of the following is the most likely
underlying cause?
A. Latissimus dorsi
B. Serratus anterior
C. Pectoralis minor
D. Pectoralis major
E. Rhomboids
Next question
Theme from April 2012 Exam
Winging of the scapula is most commonly the result of
long thoracic nerve injury or dysfunction. Iatrogenic
damage during the course of the difficult axillary dissection
is the most likely cause in this scenario. Damage to the
rhomboids may produce winging of the scapula but would
be rare in the scenario given.
Long thoracic nerve
 Derived from ventral rami of C5, C6, and C7 (close to
their emergence from intervertebral foramina)
 It runs downward and passes either anterior or
posterior to the middle scalene muscle
 It reaches upper tip of serratus anterior muscle and
descends on outer surface of this muscle, giving
branches into it
 Winging of Scapula occurs in long thoracic nerve
injury (most common) or from spinal accessory nerve
injury (which denervates the trapezius) or a dorsal
scapular nerve injury
A 23 year old man falls and slips at a nightclub. A shard of
glass penetrates the skin at the level of the medial
epicondyle, which of the following sequelae is least likely
to occur?
A. Atrophy of the first dorsal interosseous muscle
B. Difficulty in abduction of the the 2nd, 3rd, 4th and
5th fingers
C. Claw like appearance of the hand
D. Loss of sensation on the anterior aspect of the 5th
finger
E. Partial denervation of flexor digitorum profundus
Next question
Injury to the ulnar nerve in the mid to distal forearm will
typically produce a claw hand. This consists of flexion of the 4th
and 5th interphalangeal joints and extension of the
metacarpophalangeal joints. The effects are potentiated when
flexor digitorum profundus is not affected, and the clawing is
more pronounced.More proximally sited ulnar nerve lesions
produce a milder clinical picture owing to the simultaneous
paralysis of flexor digitorum profundus (ulnar half).
This is the 'ulnar paradox', due to the more proximal level
of transection the hand will typically not have a claw like
appearance that may be seen following a more distal
injury. The first dorsal interosseous muscle will be affected
as it is supplied by the ulnar nerve.
Ulnar nerve
Origin
 C8, T1
Supplies (no muscles in the upper arm)
 Flexor carpi ulnaris
 Flexor digitorum profundus
 Flexor digiti minimi
 Abductor digiti minimi
 Opponens digiti minimi
 Adductor pollicis
 Interossei muscle
 Third and fourth lumbricals
 Palmaris brevis
Path
 Posteromedial aspect of ulna to flexor compartment
of forearm, then along the ulnar. Passes beneath the
flexor carpi ulnaris muscle, then superficially through
the flexor retinaculum into the palm of the hand.
Image sourced from Wikipedia
Branches
Branch Supplies
Articular branch Flexor carpi ulnaris
Medial half of the flexor digitorum
profundus
Palmar cutaneous branch (Arises
near the middle of the forearm)
Skin on the medial part of the palm
Dorsal cutaneous branch Dorsal surface of the medial part of the
hand
Superficial branch Cutaneous fibres to the anterior surfaces
of the medial one and one-half digits
Deep branch Hypothenar muscles
All the interosseous muscles
Third and fourth lumbricals
Adductor pollicis
Medial head of the flexor pollicis brevis
Effects of injury
Damage at the
wrist
 Wasting and paralysis of intrinsic hand muscles
(claw hand)
 Wasting and paralysis of hypothenar muscles
 Loss of sensation medial 1 and half fingers
Damage at the
elbow
 Radial deviation of the wrist
 Clawing less in 3rd and 4th digits
A 43 year old lady is due to undergo an axillary node
clearance as part of treatment for carcinoma of the breast.
Which of the following fascial layers will be divided during
the surgical approach to the axilla?
A. Sibsons fascia
B. Pre tracheal fascia
C. Waldayers fascia
D. Clavipectoral fascia
E. None of the above
Next question
The clavipectoral fascia is situated under the clavicular
portion of pectoralis major. It protects both the axillary
vessels and nodes. During an axillary node clearance for
breast cancer the clavipectoral fascia is incised and this
allows access to the nodal stations. The nodal stations
are; level 1 nodes inferior to pectoralis minor, level 2 lie
behind it and level 3 above it. During a Patey Mastectomy
surgeons divide pectoralis minor to gain access to level 3
nodes. The use of sentinel node biopsy (and stronger
assistants!) have made this procedure far less common.
A 23 year old climber falls and fractures his humerus. The
surgeons decide upon a posterior approach to the middle
third of the bone. Which of the following nerves is at
greatest risk in this approach?
A. Ulnar
B. Antebrachial
C. Musculocutaneous
D. Radial
E. Intercostobrachial
Next question
Theme from April 2012 Exam
The radial nerve wraps around the humerus and may be
injured during a posterior approach. An IM nail may be
preferred as it avoids the complex dissection needed for
direct bone exposure.
Radial nerve
Continuation of posterior cord of the brachial plexus (root
values C5 to T1)
Path
 In the axilla: lies posterior to the axillary artery on
subscapularis, latissimus dorsi and teres major.
 Enters the arm between the brachial artery and the
long head of triceps (medial to humerus).
 Spirals around the posterior surface of the humerus
in the groove for the radial nerve.
 At the distal third of the lateral border of the humerus
it then pierces the intermuscular septum and
descends in front of the lateral epicondyle.
 At the lateral epicondyle it lies deeply between
brachialis and brachioradialis where it then divides
into a superficial and deep terminal branch.
 Deep branch crosses the supinator to become the
posterior interosseous nerve.
In the image below the relationships of the radial nerve
can be appreciated
Image sourced from Wikipedia
Regions innervated
Motor (main
nerve)
 Triceps
 Anconeus
 Brachioradialis
 Extensor carpi radialis
Motor
(posterior
interosseous
branch
 Extensor carpi ulnaris
 Extensor digitorum
 Extensor indicis
 Extensor digiti minimi
 Extensor pollicis longus and brevis
 Abductor pollicis longus
Sensory The area of skin supplying the proximal
phalanges on the dorsal aspect of the hand
is supplied by the radial nerve (this does
not apply to the little finger and part of the
ring finger)
Muscular innervation and effect of denervation
Anatomical
location
Muscle affected Effect of paralysis
Shoulder Long head of
triceps
Minor effects on
shoulder stability in
abduction
Arm Triceps Loss of elbow extension
Forearm Supinator
Brachioradialis
Extensor carpi
radialis longus and
brevis
Weakening of
supination of prone
hand and elbow flexion
in mid prone position
The cutaneous sensation of the upper limb- illustrating the
contribution of the radial nerve
Image sourced from Wikipedia
heme: Nerve injury
A. Median nerve
B. Ulnar nerve
C. Radial nerve
D. Posterior interosseous nerve
E. Anterior interosseous nerve
F. Musculocutaneous nerve
G. Axillary nerve
H. Brachial Trunks C5-6
I. Brachial trunks C6-7
J. Brachial Trunks C8-T1
Please select the most likely lesion site for each scenario.
Each option may be used once, more than once or not at
all.
83. A 42 year old teacher is admitted with a fall. An x-ray
confirms a fracture of the surgical neck of the humerus.
Which nerve is at risk?
Axillary nerve
The Axillary nerve winds around the bone at the neck of
the humerus. The axillary nerve is also at risk during
shoulder dislocation.
84. A 32 year old window cleaner is admitted after falling off
the roof. He reports that he had slipped off the top of the
roof and was able to cling onto the gutter for a few
seconds. The patient has Horner's syndrome.
Brachial Trunks C8-T1
The patient has a Klumpke's paralysis involving brachial
trunks C8-T1. Classically there is weakness of the hand
intrinsic muscles. Involvement of T1 may cause a
Horner's syndrome. It occurs as a result of traction
injuries or during delivery.
85. A 32 year old rugby player is hit hard on the shoulder
during a rough tackle. Clinically his arm is hanging loose
on the side. It is pronated and medially rotated.
Brachial Trunks C5-6
The patient has an Erb's palsy involving brachial trunks
C5-6.
Next question
Brachial plexus
Origin Anterior rami of C5 to T1
Sections of the  Roots, trunks, divisions, cords, branches
plexus  Mnemonic:Real Teenagers Drink Cold Beer
Roots  Located in the posterior triangle
 Pass between scalenus anterior and medius
Trunks  Located posterior to middle third of clavicle
 Upper and middle trunks related superiorly to the
subclavian artery
 Lower trunk passes over 1st rib posterior to the
subclavian artery
Divisions Apex of axilla
Cords Related to axillary artery
Diagram illustrating the branches of the brachial plexus
Image sourced from Wikipedia
Cutaneous sensation of the upper limb
Image sourced
hich of the following nerves is responsible for innervation
of the triceps muscle?
A. Radial
B. Ulnar
C. Axillary
D. Median
E. None of the above
Next question
To remember nerve roots and their relexes:
1-2 Ankle (S1-S2)
3-4 Knee (L3-L4)
5-6 Biceps (C5-C6)
7-8 Triceps (C7-C8)
The radial nerve innervates all three heads of triceps, with
a separate branch to each head.
Triceps
Origin  Long head- infraglenoid tubercle of the scapula.
 Lateral head- dorsal surface of the humerus, lateral
and proximal to the groove of the radial nerve
 Medial head- posterior surface of the humerus on the
inferomedial side of the radial groove and both of the
intermuscular septae
Insertion  Olecranon process of the ulna. Here the olecranon
bursa is between the triceps tendon and olecranon.
 Some fibres insert to the deep fascia of the forearm,
posterior capsule of the elbow (preventing the capsule
from being trapped between olecranon and olecranon
fossa during extension)
Innervation Radial nerve
Blood
supply
Profunda brachii artery
Action Elbow extension. The long head can adduct the humerus and
and extend it from a flexed position
Relations The radial nerve and profunda brachii vessels lie between the
lateral and medial heads
Which of the following muscles inserts onto the lesser
tuberostiy of the the humerus?
A. Subscapularis
B. Deltoid
C. Supraspinatus
D. Teres minor
E. Infraspinatus
Next question
With the exception of subscapularis which inserts into the
lesser tuberosity, the muscles of the rotator cuff insert into
the greater tuberosity.
Shoulder joint
 Shallow synovial ball and socket type of joint.
 It is an inherently unstable joint, but is capable to a
wide range of movement.
 Stability is provided by muscles of the rotator cuff that
pass from the scapula to insert in the greater
tuberosity (all except sub scapularis-lesser
tuberosity).
Glenoid labrum
 Fibrocartilaginous rim attached to the free edge of
the glenoid cavity
 Tendon of the long head of biceps arises from within
the joint from the supraglenoid tubercle, and is fused
at this point to the labrum.
 The long head of triceps attaches to the infraglenoid
tubercle
Fibrous capsule
 Attaches to the scapula external to the glenoid
labrum and to the labrum itself (postero-superiorly)
 Attaches to the humerus at the level of the
anatomical neck superiorly and the surgical neck
inferiorly
 Anteriorly the capsule is in contact with the tendon of
subscapularis, superiorly with the supraspinatus
tendon, and posteriorly with the tendons of
infraspinatus and teres minor. All these blend with
the capsule towards their insertion.
 Two defects in the fibrous capsule; superiorly for the
tendon of biceps. Anteriorly there is a defect beneath
the subscapularis tendon.
 The inferior extension of the capsule is closely
related to the axillary nerve at the surgical neck and
this nerve is at risk in anteroinferior dislocations. It
also means that proximally sited osteomyelitis may
progress to septic arthritis.
Movements and muscles
Flexion Anterior part of deltoid
Pectoralis major
Biceps
Coracobrachialis
Extension Posterior deltoid
Teres major
Latissimus dorsi
Adduction Pectoralis major
Latissimus dorsi
Teres major
Coracobrachialis
Abduction Mid deltoid
Supraspinatus
Medial rotation Subscapularis
Anterior deltoid
Teres major
Latissimus dorsi
Lateral rotation Posterior deltoid
Infraspinatus
Teres minor
Important anatomical relations
Anteriorly Brachial plexus
Axillary artery and vein
Posterior Suprascapular nerve
Suprascapular vessels
Inferior Axillary nerve
Circumflex humeral vessels
hich of the following nerves is not contained within the
posterior triangle of the neck?
A. Accessory nerve
B. Phrenic nerve
C. Greater auricular nerve
D. Ansa cervicalis
E. Lesser occiptal nerve
Next question
Theme from September 2012 Exam
Ansa cervicalis is a content of the anterior triangle of the
neck.
Posterior triangle of the neck
Boundaries
Apex Sternocleidomastoid and the Trapezius muscles at the Occipital
bone
Anterior Posterior border of the Sternocleidomastoid
Posterior Anterior border of the Trapezius
Base Middle third of the clavicle
Image sourced from Wikipedia
Contents
Nerves  Accessory nerve
 Phrenic nerve
 Three trunks of the brachial plexus
 Branches of the cervical plexus: Supraclavicular nerve,
transverse cervical nerve, great auricular nerve, lesser
occipital nerve
Vessels  External jugular vein
 Subclavian artery
Muscles  Inferior belly of omohyoid
 Scalene
Lymph
nodes
 Supraclavicular
 Occipital
A 73 year old lady suffers a fracture at the surgical neck of
the humerus. The decision is made to operate. There are
difficulties in reducing the fracture and a vessel lying
posterior to the surgical neck is injured. Which of the
following is this vessel most likely to be?
A. Axillary artery
B. Brachial artery
C. Thoracoacromial artery
D. Transverse scapular artery
E. Posterior circumflex humeral artery
Next question
The circumflex humeral arteries lie at the surgical neck
and is this scenario the posterior circumflex is likely to be
injured. The thoracoacromial and transverse scapular
arteries lie more superomedially. The posterior circumflex
humeral artery is a branch of the axillary artery.
Shoulder joint
 Shallow synovial ball and socket type of joint.
 It is an inherently unstable joint, but is capable to a
wide range of movement.
 Stability is provided by muscles of the rotator cuff that
pass from the scapula to insert in the greater
tuberosity (all except sub scapularis-lesser
tuberosity).
Glenoid labrum
 Fibrocartilaginous rim attached to the free edge of
the glenoid cavity
 Tendon of the long head of biceps arises from within
the joint from the supraglenoid tubercle, and is fused
at this point to the labrum.
 The long head of triceps attaches to the infraglenoid
tubercle
Fibrous capsule
 Attaches to the scapula external to the glenoid
labrum and to the labrum itself (postero-superiorly)
 Attaches to the humerus at the level of the
anatomical neck superiorly and the surgical neck
inferiorly
 Anteriorly the capsule is in contact with the tendon of
subscapularis, superiorly with the supraspinatus
tendon, and posteriorly with the tendons of
infraspinatus and teres minor. All these blend with
the capsule towards their insertion.
 Two defects in the fibrous capsule; superiorly for the
tendon of biceps. Anteriorly there is a defect beneath
the subscapularis tendon.
 The inferior extension of the capsule is closely
related to the axillary nerve at the surgical neck and
this nerve is at risk in anteroinferior dislocations. It
also means that proximally sited osteomyelitis may
progress to septic arthritis.
Movements and muscles
Flexion Anterior part of deltoid
Pectoralis major
Biceps
Coracobrachialis
Extension Posterior deltoid
Teres major
Latissimus dorsi
Adduction Pectoralis major
Latissimus dorsi
Teres major
Coracobrachialis
Abduction Mid deltoid
Supraspinatus
Medial rotation Subscapularis
Anterior deltoid
Teres major
Latissimus dorsi
Lateral rotation Posterior deltoid
Infraspinatus
Teres minor
Important anatomical relations
Anteriorly Brachial plexus
Axillary artery and vein
Posterior Suprascapular nerve
Suprascapular vessels
Inferior Axillary nerve
Circumflex humeral vessels
Which of the structures listed below lies posterior to the
carotid sheath at the level of the 6th cervical vertebra?
A. Hypoglossal nerve
B. Vagus nerve
C. Cervical sympathetic chain
D. Ansa cervicalis
E. Glossopharyngeal nerve
Next question
The carotid sheath is crossed anteriorly by the
hypoglossal nerves and the ansa cervicalis. The vagus
lies within it. The cervical sympathetic chain lies
posteriorly between the sheath and the prevertebral
fascia.
Common carotid artery
The right common carotid artery arises at the bifurcation of
the brachiocephalic trunk, the left common carotid arises
from the arch of the aorta. Both terminate at the level of
the upper border of the thyroid cartilage (the lower border
of the third cervical vertebra) by dividing into the internal
and external carotid arteries.
Left common carotid artery
This vessel arises immediately to the left and slightly
behind the origin of the brachiocephalic trunk. Its thoracic
portion is 2.5- 3.5 cm in length and runs superolaterally to
the sternoclavicular joint.
In the thorax
The vessel is in contact, from below upwards, with the
trachea, left recurrent laryngeal nerve, left margin of the
oesophagus. Anteriorly the left brachiocephalic vein runs
across the artery, and the cardiac branches from the left
vagus descend in front of it. These structures together
with the thymus and the anterior margins of the left lung
and pleura separate the artery from the manubrium.
In the neck
The artery runs superiorly deep to sternocleidomastoid
and then enters the anterior triangle. At this point it lies
within the carotid sheath with the vagus nerve and the
internal jugular vein. Posteriorly the sympathetic trunk lies
between the vessel and the prevertebral fascia. At the
level of C7 the vertebral artery and thoracic duct lie behind
it. The anterior tubercle of C6 transverse process is
prominent and the artery can be compressed against this
structure (it corresponds to the level of the cricoid).
Anteriorly at C6 the omohyoid muscle passes superficial
to the artery.
Within the carotid sheath the jugular vein lies lateral to the
artery.
Right common carotid artery
The right common carotid arises from the brachiocephalic
artery. The right common carotid artery corresponds with
the cervical portion of the left common carotid, except that
there is no thoracic duct on the right. The oesophagus is
less closely related to the right carotid than the left.
Summary points about the carotid anatomy
Path
Passes behind the sternoclavicular joint (12% patients
above this level) to the upper border of the thyroid
cartilage, to divide into the external (ECA) and internal
carotid arteries (ICA).
Relations
 Level of 6th cervical vertebra crossed by omohyoid
 Then passes deep to the thyrohyoid, sternohyoid,
sternomastoid muscles.
 Passes behind the carotid tubercle (transverse
process 6th cervical vertebra)-NB compression here
stops haemorrhage.
 The inferior thyroid artery passes posterior to the
common carotid artery.
 Then : Left common carotid artery crossed by
thoracic duct, Right common carotid artery crossed
by recurrent laryngeal nerve
Image sourced from Wikipedia
A 45 year old man presents with a lipoma located
posterior to the posterior border of the
sternocleidomastoid muscle, approximately 4cm superior
to the middle third of the clavicle. During surgical excision
of the lesion troublesome bleeding is encountered. Which
of the following is the most likely source?
A. Internal jugular vein
B. External jugular vein
C. Common carotid artery
D. Vertebral artery
E. Second part of the subclavian artery
Next question
The external jugular vein runs obliquely in the superficial
fascia of the posterior triangle. It drains into the subclavian
vein. During surgical exploration of this area the external
jugular vein may be injured and troublesome bleeding may
result. The internal jugular vein and carotid arteries are
located in the anterior triangle. The third, and not the
second, part of the subclavian artery is also a content of
the posterior triangle
Posterior triangle of the neck
Boundaries
Apex Sternocleidomastoid and the Trapezius muscles at the Occipital
bone
Anterior Posterior border of the Sternocleidomastoid
Posterior Anterior border of the Trapezius
Base Middle third of the clavicle
Image sourced from Wikipedia
Contents
Nerves  Accessory nerve
 Phrenic nerve
 Three trunks of the brachial plexus
 Branches of the cervical plexus: Supraclavicular nerve,
transverse cervical nerve, great auricular nerve, lesser
occipital nerve
Vessels  External jugular vein
 Subclavian artery
Muscles  Inferior belly of omohyoid
 Scalene
Lymph
nodes
 Supraclavicular
 Occipital
Which of the following upper limb muscles is not innervated by the
radial nerve?
A. Extensor carpi ulnaris
B. Abductor digit minimi
C. Anconeus
D. Supinator
E. Brachioradialis
Next question
Mnemonic for radial nerve muscles: BEST
B rachioradialis
E xtensors
S upinator
T riceps
Abductor digiti minimi is innervated by the ulnar nerve.
Radial nerve
Continuation of posterior cord of the brachial plexus (root values C5
to T1)
Path
 In the axilla: lies posterior to the axillary artery on
subscapularis, latissimus dorsi and teres major.
 Enters the arm between the brachial artery and the long head
of triceps (medial to humerus).
 Spirals around the posterior surface of the humerus in the
groove for the radial nerve.
 At the distal third of the lateral border of the humerus it then
pierces the intermuscular septum and descends in front of the
lateral epicondyle.
 At the lateral epicondyle it lies deeply between brachialis and
brachioradialis where it then divides into a superficial and deep
terminal branch.
 Deep branch crosses the supinator to become the posterior
interosseous nerve.
In the image below the relationships of the radial nerve can be
appreciated
Image sourced from Wikipedia
Regions innervated
Motor (main
nerve)
 Triceps
 Anconeus
 Brachioradialis
 Extensor carpi radialis
Motor (posterior
interosseous
branch
 Extensor carpi ulnaris
 Extensor digitorum
 Extensor indicis
 Extensor digiti minimi
 Extensor pollicis longus and brevis
 Abductor pollicis longus
Sensory The area of skin supplying the proximal phalanges on
the dorsal aspect of the hand is supplied by the radial
nerve (this does not apply to the little finger and part
of the ring finger)
Muscular innervation and effect of denervation
Anatomical
location
Muscle affected Effect of paralysis
Shoulder Long head of triceps Minor effects on shoulder
stability in abduction
Arm Triceps Loss of elbow extension
Forearm Supinator
Brachioradialis
Extensor carpi radialis
longus and brevis
Weakening of supination of
prone hand and elbow flexion in
mid prone position
The cutaneous sensation of the upper limb- illustrating the
contribution of the radial nerve
Which of the following forms the floor of the anatomical snuffbox?
A. Radial artery
B. Cephalic vein
C. Extensor pollicis brevis
D. Scaphoid bone
E. Cutaneous branch of the radial nerve
Next question
The scaphoid bone forms the floor of the anatomical snuffbox. The
cutaneous branch of the radial nerve is much more superficially and
proximally located.
Anatomical snuffbox
Posterior border Tendon of extensor pollicis longus
Anterior border Tendons of extensor pollicis brevis and abductor pollicis longus
Proximal border Styloid process of the radius
Distal border Apex of snuffbox triangle
Floor Trapezium and scaphoid
Content Radial artery
Image showing the anatomical snuffbox
A 32 year old lady complains of carpal tunnel syndrome. The carpal
tunnel is explored surgically. Which of the following structures will lie
in closest proximity to the hamate bone within the carpal tunnel?
A. The tendon of abductor pollicis longus
B. The tendons of flexor digitorum profundus
C. The tendons of flexor carpi radialis longus
D. Median nerve
E. Radial artery
Next question
The carpal tunnel contains nine flexor tendons:
 Flexor digitorum profundus
 Flexor digitorum superficialis
 Flexor pollicis longus
The tendon of flexor digitorum profundus lies deepest in the tunnel
and will thus lie nearest to the hamate bone.
Carpal bones
Diagrammatic image of carpal bones
Image sourced from Wikipedia
Key to image
A Scaphoid
B Lunate
C Triquetrum
D Pisiform
E Trapezium
F Trapezoid
G Capitate
H Hamate
1 Radius
2 Ulna
3 Metacarpals
 No tendons attach to: Scaphoid, lunate, triquetrum (stabilised
by ligaments)
A 45 year man presents with hand weakness. He is given a piece of
paper to hold between his thumb and index finger. When the paper is
pulled, the patient has difficulty maintaining a grip. Grip pressure is
maintained by flexing the thumb at the interphalangeal joint. What is
the most likely nerve lesion?
A. Posterior interosseous nerve
B. Deep branch of ulnar nerve
C. Anterior interosseous nerve
D. Superficial branch of the ulnar nerve
E. Radial nerve
Next question
Theme from January 2012 exam
This is a description of Froment's sign, which tests for ulnar nerve
palsy. It mainly tests for the function of adductor pollicis. This is
supplied by the deep branch of the ulnar nerve. Remember the
anterior interosseous branch, which innervates the flexor pollicis
longus (hence causing flexion of the thumb IP joint), branches off
more proximally to the wrist.
Ulnar nerve
Origin
 C8, T1
Supplies (no muscles in the upper arm)
 Flexor carpi ulnaris
 Flexor digitorum profundus
 Flexor digiti minimi
 Abductor digiti minimi
 Opponens digiti minimi
 Adductor pollicis
 Interossei muscle
 Third and fourth lumbricals
 Palmaris brevis
Path
 Posteromedial aspect of ulna to flexor compartment of forearm,
then along the ulnar. Passes beneath the flexor carpi ulnaris
muscle, then superficially through the flexor retinaculum into
the palm of the hand.
Image sourced from Wikipedia
Branches
Branch Supplies
Articular branch Flexor carpi ulnaris
Medial half of the flexor digitorum profundus
Palmar cutaneous branch (Arises near the
middle of the forearm)
Skin on the medial part of the palm
Dorsal cutaneous branch Dorsal surface of the medial part of the hand
Superficial branch Cutaneous fibres to the anterior surfaces of the
medial one and one-half digits
Deep branch Hypothenar muscles
All the interosseous muscles
Third and fourth lumbricals
Adductor pollicis
Medial head of the flexor pollicis brevis
Effects of injury
Damage at the wrist  Wasting and paralysis of intrinsic hand muscles (claw hand)
 Wasting and paralysis of hypothenar muscles
 Loss of sensation medial 1 and half fingers
Damage at the
elbow
 Radial deviation of the wrist
 Clawing less in 3rd and 4th digits
A 10 year old by falls out of a tree has suffers a supracondylar
fracture. He complains of a painful elbow and forearm. There is an
obvious loss of pincer movement involving the thumb and index
finger with minimal loss of sensation. The most likely nerve injury is
to the:
A. Ulnar nerve
B. Radial nerve
C. Anterior interosseous nerve
D. Axillary nerve damage
E. Median nerve damage above the elbow
Next question
The anterior interosseous nerve is a motor branch of the median
nerve just below the elbow. When damaged it classically causes:
 Pain in the forearm
 Loss of pincer movement of the thumb and index finger
(innervates the long flexor muscles of flexor pollicis longus &
flexor digitorum profundus of the index and middle finger)
 Minimal loss of sensation due to lack of a cutaneous branch
Median nerve
The median nerve is formed by the union of a lateral and medial root
respectively from the lateral (C5,6,7) and medial (C8 and T1) cords of
the brachial plexus; the medial root passes anterior to the third part of
the axillary artery. The nerve descends lateral to the brachial artery,
crosses to its medial side (usually passing anterior to the artery). It
passes deep to the bicipital aponeurosis and the median cubital vein
at the elbow.
It passes between the two heads of the pronator teres muscle, and
runs on the deep surface of flexor digitorum superficialis (within its
fascial sheath).
Near the wrist it becomes superficial between the tendons of flexor
digitorum superficialis and flexor carpi radialis, deep to palmaris
longus tendon. It passes deep to the flexor retinaculum to enter the
palm, but lies anterior to the long flexor tendons within the carpal
tunnel.
Branches
Region Branch
Upper arm No branches, although the nerve commonly communicates with the
musculocutaneous nerve
Forearm Pronator teres
Flexor carpi radialis
Palmaris longus
Flexor digitorum superficialis
Flexor pollicis longus
Flexor digitorum profundus (only the radial half)
Distal
forearm
Palmar cutaneous branch
Hand
(Motor)
Motor supply (LOAF)
 Lateral 2 lumbricals
 Opponens pollicis
 Abductor pollicis brevis
 Flexor pollicis brevis
Hand
(Sensory)
 Over thumb and lateral 2 ½ fingers
 On the palmar aspect this projects proximally, on the dorsal aspect only
the distal regions are innervated with the radial nerve providing the
more proximal cutaneous innervation.
Patterns of damage
Damage at wrist
 e.g. carpal tunnel syndrome
 paralysis and wasting of thenar eminence muscles and
opponens pollicis (ape hand deformity)
 sensory loss to palmar aspect of lateral (radial) 2 ½ fingers
Damage at elbow, as above plus:
 unable to pronate forearm
 weak wrist flexion
 ulnar deviation of wrist
Anterior interosseous nerve (branch of median nerve)
 leaves just below the elbow
 results in loss of pronation of forearm and weakness of long
flexors of thumb and index finger
Topography of the median nerve
Image sourced from Wikipedia
A 32 year old attends neurology clinic complaining of tingling in his
hand. He has radial deviation of his wrist and there is mild clawing of
his fingers, with the 3rd and 4th digits being relatively spared. What is
the most likely lesion?
A. Ulnar nerve damage at the wrist
B. Ulnar nerve damage at the elbow
C. Radial nerve damage at the elbow
D. Median nerve damage at the wrist
E. Median nerve damage at the elbow
Next question
At the elbow the ulnar nerve lesion affects the flexor carpi ulnaris and
flexor digitorum profundus.
Ulnar nerve
Origin
 C8, T1
Supplies (no muscles in the upper arm)
 Flexor carpi ulnaris
 Flexor digitorum profundus
 Flexor digiti minimi
 Abductor digiti minimi
 Opponens digiti minimi
 Adductor pollicis
 Interossei muscle
 Third and fourth lumbricals
 Palmaris brevis
Path
 Posteromedial aspect of ulna to flexor compartment of forearm,
then along the ulnar. Passes beneath the flexor carpi ulnaris
muscle, then superficially through the flexor retinaculum into
the palm of the hand.
Image sourced from Wikipedia
Branches
Branch Supplies
Articular branch Flexor carpi ulnaris
Medial half of the flexor digitorum profundus
Palmar cutaneous branch (Arises near the
middle of the forearm)
Skin on the medial part of the palm
Dorsal cutaneous branch Dorsal surface of the medial part of the hand
Superficial branch Cutaneous fibres to the anterior surfaces of the
medial one and one-half digits
Deep branch Hypothenar muscles
All the interosseous muscles
Third and fourth lumbricals
Adductor pollicis
Medial head of the flexor pollicis brevis
Effects of injury
Damage at the wrist  Wasting and paralysis of intrinsic hand muscles (claw hand)
 Wasting and paralysis of hypothenar muscles
 Loss of sensation medial 1 and half fingers
Damage at the
elbow
 Radial deviation of the wrist
 Clawing less in 3rd and 4th digits
A 23 year old man is involved in a fight and is stabbed in his upper
arm. The ulnar nerve is transected. Which of the following muscles
will not demonstrate compromised function as a result?
A. Flexor carpi ulnaris
B. Medial half of flexor digitorum profundus
C. Palmaris brevis
D. Hypothenar muscles
E. Pronator teres
Next question
M edial lumbricals
A dductor pollicis
F lexor digitorum profundus/Flexor digiti minimi
I nterossei
A bductor digiti minimi and opponens
Innervates all intrinsic muscles of the hand (EXCEPT 2: thenar muscles & first
two lumbricals - supplied by median nerve)
Pronator teres is innervated by the median nerve. Palmaris brevis is
innervated by the ulnar nerve
Ulnar nerve
Origin
 C8, T1
Supplies (no muscles in the upper arm)
 Flexor carpi ulnaris
 Flexor digitorum profundus
 Flexor digiti minimi
 Abductor digiti minimi
 Opponens digiti minimi
 Adductor pollicis
 Interossei muscle
 Third and fourth lumbricals
 Palmaris brevis
Path
 Posteromedial aspect of ulna to flexor compartment of forearm,
then along the ulnar. Passes beneath the flexor carpi ulnaris
muscle, then superficially through the flexor retinaculum into
the palm of the hand.
Image sourced from Wikipedia
Branches
Branch Supplies
Articular branch Flexor carpi ulnaris
Medial half of the flexor digitorum profundus
Palmar cutaneous branch (Arises near the
middle of the forearm)
Skin on the medial part of the palm
Dorsal cutaneous branch Dorsal surface of the medial part of the hand
Superficial branch Cutaneous fibres to the anterior surfaces of the
medial one and one-half digits
Deep branch Hypothenar muscles
All the interosseous muscles
Third and fourth lumbricals
Adductor pollicis
Medial head of the flexor pollicis brevis
Effects of injury
Damage at the wrist  Wasting and paralysis of intrinsic hand muscles (claw hand)
 Wasting and paralysis of hypothenar muscles
 Loss of sensation medial 1 and half fingers
Damage at the
elbow
 Radial deviation of the wrist
 Clawing less in 3rd and 4th digits
Which of the structures listed below overlies the cephalic vein?
A. Extensor retinaculum
B. Bicipital aponeurosis
C. Biceps muscle
D. Antebrachial fascia
E. None of the above
Next question
The cephalic vein is superficially located in the upper limb and
overlies most the fascial planes. It pierces the coracoid membrane
(continuation of the clavipectoral fascia) to terminate in the axillary
vein. It lies anterolaterally to biceps.
Cephalic vein
Path
 Dorsal venous arch drains laterally into the cephalic vein
 Crosses the anatomical snuffbox and travels laterally up the
arm
 At the antecubital fossa connected to the basilic vein by the
median cubital vein
 Pierces deep fascia of deltopectoral groove to join axillary vein
A 22 year old man is involved in a fight. He sustains a laceration to
the posterior aspect of his wrist. In the emergency department the
wound is explored and the laceration is found to be transversely
orientated and overlies the region of the extensor retinaculum, which
is intact. Which of the following structures is least likely to be injured
in this scenario?
A. Dorsal cutaneous branch of the ulnar nerve
B. Tendon of extensor indicis
C. Basilic vein
D. Superficial branch of the radial nerve
E. Cephalic vein
Next question
The extensor retinaculum attaches to the radius proximal to the
styloid, thereafter it runs obliquely and distally to wind around the
ulnar styloid (but does not attach to it). The extensor tendons lie deep
to the extensor retinaculum and would therefore be less susceptible
to injury than the superficial structures.
Extensor retinaculum
The extensor rentinaculum is a thickening of the deep fascia that
stretches across the back of the wrist and holds the long extensor
tendons in position.
Its attachments are:
 The pisiform and hook of hamate medially
 The end of the radius laterally
Structures related to the extensor retinaculum
Structures superficial to the
retinaculum
 Basilic vein
 Dorsal cutaneous branch of the ulnar nerve
 Cephalic vein
 Superficial branch of the radial nerve
Structures passing deep to the
extensor retinaculum
 Extensor carpi ulnaris tendon
 Extensor digiti minimi tendon
 Extensor digitorum and extensor indicis
tendon
 Extensor pollicis longus tendon
 Extensor carpi radialis longus tendon
 Abductor pollicis longus and extensor
pollicis brevis tendons
Beneath the extensor retinaculum fibrous septa form six
compartments that contain the extensor muscle tendons. Each
compartment has its own synovial sheath.
The radial artery
The radial artery passes between the lateral collateral ligament of the
wrist joint and the tendons of the abductor pollicis longus and
extensor pollicis brevis.
Image illustrating the topography of tendons passing under the
extensor retinaculum
A man has an incision sited than runs 8cm from the deltopectoral
groove to the midline. Which of the following is not at risk of injury?
A. Cephalic vein
B. Shoulder joint capsule
C. Axillary artery
D. Pectoralis major
E. Trunk of the brachial plexus
Next question
Theme from April 2012 Exam
This region will typically lie medial to the joint capsule. The diagram
below illustrates the plane that this would transect and as it can be
appreciated the other structures are all at risk of injury.
Image sourced from Wikipedia
Pectoralis major muscle
Origin From the medial two thirds of the clavicle, manubrium and sternocostal angle
Insertion Crest of the greater tubercle of the humerus
Nerve supply Lateral pectoral nerve
Actions Adductor and medial rotator of the humerus
heme: Nerve Injury
A. Median nerve
B. Ulnar nerve
C. Radial nerve
D. Musculocutaneous nerve
E. Axillary nerve
F. Anterior interosseous nerve
G. Posterior interosseous nerve
For each scenario please select the most likely underlying nerve
injury. Each option may be used once, more than once or not at all.
171. A 19 year old student is admitted to A&E after falling off a
wall. He is unable to flex his index finger. An x-ray confirms a
supracondylar fracture.
You answered Anterior interosseous nerve
The correct answer is Median nerve
This median nerve is at risk during a supracondylar fracture.
172. A well toned weight lifter attends clinic reporting weakness of
his left arm. There is weakness of flexion and supination of
the forearm.
You answered Posterior interosseous nerve
The correct answer is Musculocutaneous nerve
Mucocutaneous nerve compression due to entrapment of the
nerve between biceps and brachialis. Elbow flexion and
supination of the arm are affected. This is a rare isolated
injury.
173. An 18 year old girl sustains an Holstein-Lewis fracture. Which
nerve is at risk?
You answered Axillary nerve
The correct answer is Radial nerve
Proximal lesions affect the triceps. Also paralysis of wrist
extensors and forearm supinators occur. Reduced sensation
of dorsoradial aspect of hand and dorsal 31/2 fingers.
Holstein-Lewis fractures are fractures of the distal humerus
with radial nerve entrapment.
A 35 year old farm labourer is injures the posterior aspect of his hand with a
mechanical scythe. He severs some of his extensor tendons in this injury. How many
tunnels lie in the extensor retinaculum that transmit the tendons of the extensor
muscles?
A. One
B. Three
C. Four
D. Five
E. Six
Next question
There are six tunnels, each lined by its own synovial sheath.
Extensor retinaculum
The extensor rentinaculum is a thickening of the deep fascia that stretches across
the back of the wrist and holds the long extensor tendons in position.
Its attachments are:
 The pisiform and hook of hamate medially
 The end of the radius laterally
Structures related to the extensor retinaculum
Structures superficial to the retinaculum  Basilic vein
 Dorsal cutaneous branch of the ulnar nerve
 Cephalic vein
 Superficial branch of the radial nerve
Structures passing deep to the extensor
retinaculum
 Extensor carpi ulnaris tendon
 Extensor digiti minimi tendon
 Extensor digitorum and extensor indicis tendon
 Extensor pollicis longus tendon
 Extensor carpi radialis longus tendon
 Abductor pollicis longus and extensor pollicis brevis
tendons
Beneath the extensor retinaculum fibrous septa form six compartments that contain
the extensor muscle tendons. Each compartment has its own synovial sheath.
The radial artery
The radial artery passes between the lateral collateral ligament of the wrist joint and
the tendons of the abductor pollicis longus and extensor pollicis brevis.
Image illustrating the topography of tendons passing under the extensor retinaculum
A 23 year old man is injured during a game of rugby. He suffers a
fracture of the distal third of his clavicle, it is a compound fracture and
there is evidence of arterial haemorrhage. Which of the following
vessels is most likely to be encountered first during subsequent
surgical exploration?
A. Posterior circumflex humeral artery
B. Axillary artery
C. Thoracoacromial artery
D. Sub scapular artery
E. Lateral thoracic artery
Next question
Similar theme in September 2011 Exam
The thoracoacromial artery arises from the second part of the axillary
artery. It is a short, wide trunk, which pierces the clavipectoral fascia,
and ends, deep to pectoralis major by dividing into four branches.
Thoracoacromial artery
The thoracoacromial artery (acromiothoracic artery; thoracic axis) is a
short trunk, which arises from the forepart of the axillary artery, its
origin being generally overlapped by the upper edge of the Pectoralis
minor.
Projecting forward to the upper border of the Pectoralis minor, it
pierces the coracoclavicular fascia and divides into four branches:
pectoral, acromial, clavicular, and deltoid.
Branch Description
Pectoral
branch
Descends between the two Pectoral muscles, and is distributed to them and to
the breast, anastomosing with the intercostal branches of the internal thoracic
artery and with the lateral thoracic.
Acromial
branch
Runs laterally over the coracoid process and under the Deltoid, to which it
gives branches; it then pierces that muscle and ends on the acromion in an
arterial network formed by branches from the suprascapular, thoracoacromial,
and posterior humeral circumflex arteries.
Clavicular
branch
Runs upwards and medially to the sternoclavicular joint, supplying this
articulation, and the Subclavius
Deltoid
branch
Arising with the acromial, it crosses over the Pectoralis minor and passes in the
same groove as the cephalic vein, between the Pectoralis major and Deltoid,
and gives branches to both muscles.
68 year old man falls onto an outstretched hand. Following the
accident he is examined in the emergency department. On palpating
his anatomical snuffbox there is tenderness noted in the base. What
is the most likely injury in this scenario?
A. Rupture of the tendon of flexor pollicis
B. Scaphoid fracture
C. Distal radius fracture
D. Rupture of flexor carpi ulnaris tendon
E. None of the above
Next question
A fall onto an outstretched hand is a common mechanism of injury for
a scaphoid fracture. This should be suspected clinically if there is
tenderness in the base of the anatomical snuffbox. A tendon rupture
would not result in bony tenderness.
Scaphoid bone
The scaphoid has a concave articular surface for the head of the
capitate and at the edge of this is a crescentic surface for the
corresponding area on the lunate.
Proximally, it has a wide convex articular surface with the radius. It
has a distally sited tubercle that can be palpated. The remaining
articular surface is to the lateral side of the tubercle. It faces laterally
and is associated with the trapezium and trapezoid bones.
The narrow strip between the radial and trapezial surfaces and the
tubercle gives rise to the radial collateral carpal ligament. The
tubercle receives part of the flexor retinaculum. This area is the only
part of the scaphoid that is available for the entry of blood vessels. It
is commonly fractured and avascular necrosis may result.
Scaphoid bone
Which of the following structures passes through the quadrangular
space near the humeral head?
A. Axillary artery
B. Radial nerve
C. Axillary nerve
D. Median nerve
E. Transverse scapular artery
Next question
The quadrangular space is bordered by the humerus laterally,
subscapularis superiorly, teres major inferiorly and the long head of
triceps medially. It lies lateral to the triangular space. It transmits the
axillary nerve and posterior circumflex humeral artery.
Image sourced from Wikipedia
Shoulder joint
 Shallow synovial ball and socket type of joint.
 It is an inherently unstable joint, but is capable to a wide range
of movement.
 Stability is provided by muscles of the rotator cuff that pass
from the scapula to insert in the greater tuberosity (all except
sub scapularis-lesser tuberosity).
Glenoid labrum
 Fibrocartilaginous rim attached to the free edge of the glenoid
cavity
 Tendon of the long head of biceps arises from within the joint
from the supraglenoid tubercle, and is fused at this point to the
labrum.
 The long head of triceps attaches to the infraglenoid tubercle
Fibrous capsule
 Attaches to the scapula external to the glenoid labrum and to
the labrum itself (postero-superiorly)
 Attaches to the humerus at the level of the anatomical neck
superiorly and the surgical neck inferiorly
 Anteriorly the capsule is in contact with the tendon of
subscapularis, superiorly with the supraspinatus tendon, and
posteriorly with the tendons of infraspinatus and teres minor.
All these blend with the capsule towards their insertion.
 Two defects in the fibrous capsule; superiorly for the tendon of
biceps. Anteriorly there is a defect beneath the subscapularis
tendon.
 The inferior extension of the capsule is closely related to the
axillary nerve at the surgical neck and this nerve is at risk in
anteroinferior dislocations. It also means that proximally sited
osteomyelitis may progress to septic arthritis.
Movements and muscles
Flexion Anterior part of deltoid
Pectoralis major
Biceps
Coracobrachialis
Extension Posterior deltoid
Teres major
Latissimus dorsi
Adduction Pectoralis major
Latissimus dorsi
Teres major
Coracobrachialis
Abduction Mid deltoid
Supraspinatus
Medial rotation Subscapularis
Anterior deltoid
Teres major
Latissimus dorsi
Lateral rotation Posterior deltoid
Infraspinatus
Teres minor
Important anatomical relations
Anteriorly Brachial plexus
Axillary artery and vein
Posterior Suprascapular nerve
Suprascapular vessels
Inferior Axillary nerve
Circumflex humeral vessels
Which of the following structures separates the ulnar artery from the
median nerve?
A. Brachioradialis
B. Pronator teres
C. Tendon of biceps brachii
D. Flexor carpi ulnaris
E. Brachialis
Next question
It lies deep to pronator teres and this separates it from the median
nerve.
Ulnar artery
Path
 Starts: middle of antecubital fossa
 Passes obliquely downward, reaching the ulnar side of the
forearm at a point about midway between the elbow and the
wrist. It follows the ulnar border to the wrist, crossing over the
flexor retinaculum. It then divides into the superficial and deep
volar arches.
Relations
Deep to- Pronator teres, Flexor carpi radialis, Palmaris longus
Lies on- Brachialis and Flexor digitorum profundus
Superficial to the flexor retinaculum at the wrist
The median nerve is in relation with the medial side of the artery for
about 2.5 cm. And then crosses the vessel, being separated from it
by the ulnar head of the Pronator teres
The ulnar nerve lies medially to the lower two-thirds of the artery
Branch
 Anterior interosseous artery
A 32 year old motorcyclist is involved in a road traffic accident. His
humerus is fractured and severely displaced. At the time of surgical
repair the surgeon notes that the radial nerve has been injured.
Which of the following muscles is least likely to be affected by an
injury at this site?
A. Extensor carpi radialis brevis
B. Brachioradialis
C. Abductor pollicis longus
D. Extensor pollicis brevis
E. None of the above
Next question
Muscles supplied by the radial nerve
BEST
Brachioradialis
Extensors
Supinator
Triceps
The radial nerve supplies the extensor muscles, abductor pollicis
longus and extensor pollicis brevis (the latter two being innervated by
the posterior interosseous branch of the radial nerve).
Radial nerve
Continuation of posterior cord of the brachial plexus (root values C5
to T1)
Path
 In the axilla: lies posterior to the axillary artery on
subscapularis, latissimus dorsi and teres major.
 Enters the arm between the brachial artery and the long head
of triceps (medial to humerus).
 Spirals around the posterior surface of the humerus in the
groove for the radial nerve.
 At the distal third of the lateral border of the humerus it then
pierces the intermuscular septum and descends in front of the
lateral epicondyle.
 At the lateral epicondyle it lies deeply between brachialis and
brachioradialis where it then divides into a superficial and deep
terminal branch.
 Deep branch crosses the supinator to become the posterior
interosseous nerve.
In the image below the relationships of the radial nerve can be
appreciated
Image sourced from Wikipedia
Regions innervated
Motor (main
nerve)
 Triceps
 Anconeus
 Brachioradialis
 Extensor carpi radialis
Motor (posterior
interosseous
branch
 Extensor carpi ulnaris
 Extensor digitorum
 Extensor indicis
 Extensor digiti minimi
 Extensor pollicis longus and brevis
 Abductor pollicis longus
Sensory The area of skin supplying the proximal phalanges on
the dorsal aspect of the hand is supplied by the radial
nerve (this does not apply to the little finger and part
of the ring finger)
Muscular innervation and effect of denervation
Anatomical
location
Muscle affected Effect of paralysis
Shoulder Long head of triceps Minor effects on shoulder
stability in abduction
Arm Triceps Loss of elbow extension
Forearm Supinator
Brachioradialis
Extensor carpi radialis
longus and brevis
Weakening of supination of
prone hand and elbow flexion in
mid prone position
The cutaneous sensation of the upper limb- illustrating the
contribution of the radial nerve
Image sourced from Wikipedia
Which muscle is responsible for causing flexion of the
interphalangeal joint of the thumb?
A. Flexor pollicis longus
B. Flexor pollicis brevis
C. Flexor digitorum superficialis
D. Flexor digitorum profundus
E. Adductor pollicis
Next question
There are 8 muscles:
1. Two flexors (flexor pollicis brevis and flexor pollicis longus)
2. Two extensors (extensor pollicis brevis and longus)
3. Two abductors (abductor pollicis brevis and longus)
4. One adductor (adductor pollicis)
5. One muscle that opposes the thumb by rotating the CMC joint
(opponens pollicis).
Flexor and extensor longus insert on the distal phalanx moving both
the MCP and IP joints.
Hand
Anatomy of the hand
Bones  8 Carpal bones
 5 Metacarpals
 14 phalanges
Intrinsic Muscles 7 Interossei - Supplied by ulnar nerve
 3 palmar-adduct fingers
 4 dorsal- abduct fingers
Intrinsic muscles Lumbricals
 Flex MCPJ and extend the IPJ.
 Origin deep flexor tendon and insertion dorsal extensor hood
mechanism.
 Innervation: 1st and 2nd- median nerve, 3rd and 4th- deep
branch of the ulnar nerve.
Thenar eminence  Abductor pollicis brevis
 Opponens pollicis
 Flexor pollicis brevis
Hypothenar
eminence
 Opponens digiti minimi
 Flexor digiti minimi brevis
 Abductor digiti minimi
Image sourced from Wikipedia
n 18 year old man is stabbed in the axilla during a fight. His axillary
artery is lacerated and repaired. However, the surgeon neglects to
repair an associated injury to the upper trunk of the brachial plexus.
Which of the following muscles is least likely to demonstrate impaired
function as a result?
A. Palmar interossei
B. Infraspinatus
C. Brachialis
D. Supinator brevis
E. None of the above
Next question
The palmar interossei are supplied by the ulnar nerve. Which lies
inferiorly and is therefore less likely to be injured.
Brachial plexus
Origin Anterior rami of C5 to T1
Sections of the
plexus
 Roots, trunks, divisions, cords, branches
 Mnemonic:Real Teenagers Drink Cold Beer
Roots  Located in the posterior triangle
 Pass between scalenus anterior and medius
Trunks  Located posterior to middle third of clavicle
 Upper and middle trunks related superiorly to the subclavian
artery
 Lower trunk passes over 1st rib posterior to the subclavian
artery
Divisions Apex of axilla
Cords Related to axillary artery
Diagram illustrating the branches of the brachial plexus
Image sourced from Wikipedia
Cutaneous sensation of the upper limb
Image sourced from Wikipedia
23 year old man is involved in a fight, during the dispute he sustains
a laceration to the posterior aspect of his right arm, approximately
2cm proximal to the olecranon process. On assessment in the
emergency department he is unable to extend his elbow joint. Which
of the following tendons is most likely to have been cut?
A. Triceps
B. Pronator teres
C. Brachioradialis
D. Brachialis
E. Biceps
Next question
Theme from 2009 Exam
The triceps muscle extends the elbow joint. The other muscles listed
all produce flexion of the elbow joint.
Triceps
Origin  Long head- infraglenoid tubercle of the scapula.
 Lateral head- dorsal surface of the humerus, lateral and proximal to
the groove of the radial nerve
 Medial head- posterior surface of the humerus on the inferomedial
side of the radial groove and both of the intermuscular septae
Insertion  Olecranon process of the ulna. Here the olecranon bursa is between
the triceps tendon and olecranon.
 Some fibres insert to the deep fascia of the forearm, posterior capsule
of the elbow (preventing the capsule from being trapped between
olecranon and olecranon fossa during extension)
Innervation Radial nerve
Blood
supply
Profunda brachii artery
Action Elbow extension. The long head can adduct the humerus and and extend it
from a flexed position
Relations The radial nerve and profunda brachii vessels lie between the lateral and
medial heads
Which of the following muscles does not attach to the radius?
A. Pronator quadratus
B. Biceps
C. Brachioradialis
D. Supinator
E. Brachialis
Next question
The brachialis muscle inserts into the ulna. The other muscles are all
inserted onto the radius.
Radius
 Bone of the forearm extending from the lateral side of the
elbow to the thumb side of the wrist
Upper end
 Articular cartilage- covers medial > lateral side
 Articulates with radial notch of the ulna by the annular ligament
 Muscle attachment- biceps brachii at the tuberosity
Shaft
 Muscle attachment-
Upper third of the body Supinator, Flexor digitorum superficialis,
Flexor pollicis longus
Middle third of the body Pronator teres
Lower quarter of the body Pronator quadratus , tendon of supinator
longus
Lower end
 Quadrilateral
 Anterior surface- capsule of wrist joint
 Medial surface- head of ulna
 Lateral surface- ends in the styloid process
 Posterior surface: 3 grooves containing:
1. Tendons of extensor carpi radialis longus and brevis
2. Tendon of extensor pollicis longus
3. Tendon of extensor indicis
Which of the following is not an intrinsic muscle of the hand?
A. Opponens pollicis
B. Palmaris longus
C. Flexor pollicis brevis
D. Flexor digiti minimi brevis
E. Opponens digiti minimi
Next question
Mnemonic for intrinsic hand muscles
'A OF A OF A'
A bductor pollicis brevis
O pponens pollicis
F lexor pollicis brevis
A dductor pollicis (thenar muscles)
O pponens digiti minimi
F lexor digiti minimi brevis
A bductor digiti minimi (hypothenar muscles)
Palmaris longus originates in the forearm.
Hand
Anatomy of the hand
Bones  8 Carpal bones
 5 Metacarpals
 14 phalanges
Intrinsic Muscles 7 Interossei - Supplied by ulnar nerve
 3 palmar-adduct fingers
 4 dorsal- abduct fingers
Intrinsic muscles Lumbricals
 Flex MCPJ and extend the IPJ.
 Origin deep flexor tendon and insertion dorsal extensor hood
mechanism.
 Innervation: 1st and 2nd- median nerve, 3rd and 4th- deep
branch of the ulnar nerve.
Thenar eminence  Abductor pollicis brevis
 Opponens pollicis
 Flexor pollicis brevis
Hypothenar
eminence
 Opponens digiti minimi
 Flexor digiti minimi brevis
 Abductor digiti minimi
Image sourced from Wikipedia
A 28 year old man lacerates the posterolateral aspect of his wrist with
a knife in an attempted suicide. On arrival in the emergency
department the wound is inspected and found to be located over the
lateral aspect of the extensor retinaculum (which is intact). Which of
the following structures is at greatest risk of injury?
A. Superficial branch of the radial nerve
B. Radial artery
C. Dorsal branch of the ulnar nerve
D. Tendon of extensor carpi radialis brevis
E. Tendon of extensor digiti minimi
Next question
The superficial branch of the radial nerve passes superior to the
extensor retinaculum in the position of this laceration and is at
greatest risk of injury. The dorsal branch of the ulnar nerve and artery
also pass superior to the extensor retinaculum n but are located
medially.
Extensor retinaculum
The extensor rentinaculum is a thickening of the deep fascia that
stretches across the back of the wrist and holds the long extensor
tendons in position.
Its attachments are:
 The pisiform and hook of hamate medially
 The end of the radius laterally
Structures related to the extensor retinaculum
Structures superficial to the
retinaculum
 Basilic vein
 Dorsal cutaneous branch of the ulnar nerve
 Cephalic vein
 Superficial branch of the radial nerve
Structures passing deep to the
extensor retinaculum
 Extensor carpi ulnaris tendon
 Extensor digiti minimi tendon
 Extensor digitorum and extensor indicis
tendon
 Extensor pollicis longus tendon
 Extensor carpi radialis longus tendon
 Abductor pollicis longus and extensor
pollicis brevis tendons
Beneath the extensor retinaculum fibrous septa form six
compartments that contain the extensor muscle tendons. Each
compartment has its own synovial sheath.
The radial artery
The radial artery passes between the lateral collateral ligament of the
wrist joint and the tendons of the abductor pollicis longus and
extensor pollicis brevis.
Image illustrating the topography of tendons passing under the
extensor retinaculum
ransection of the radial nerve at the level of the axilla will result in all
of the following except:
A. Loss of elbow extension.
B. Loss of extension of the interphalangeal joints.
C. Loss of metacarpophalangeal extension.
D. Loss of triceps reflex.
E. Loss of sensation overlying the first dorsal interosseous.
Next question
These may still extend by virtue of retained lumbrical muscle
function.
Radial nerve
Continuation of posterior cord of the brachial plexus (root values C5
to T1)
Path
 In the axilla: lies posterior to the axillary artery on
subscapularis, latissimus dorsi and teres major.
 Enters the arm between the brachial artery and the long head
of triceps (medial to humerus).
 Spirals around the posterior surface of the humerus in the
groove for the radial nerve.
 At the distal third of the lateral border of the humerus it then
pierces the intermuscular septum and descends in front of the
lateral epicondyle.
 At the lateral epicondyle it lies deeply between brachialis and
brachioradialis where it then divides into a superficial and deep
terminal branch.
 Deep branch crosses the supinator to become the posterior
interosseous nerve.
In the image below the relationships of the radial nerve can be
appreciated
Image sourced from Wikipedia
Regions innervated
Motor (main
nerve)
 Triceps
 Anconeus
 Brachioradialis
 Extensor carpi radialis
Motor (posterior
interosseous
branch
 Extensor carpi ulnaris
 Extensor digitorum
 Extensor indicis
 Extensor digiti minimi
 Extensor pollicis longus and brevis
 Abductor pollicis longus
Sensory The area of skin supplying the proximal phalanges on
the dorsal aspect of the hand is supplied by the radial
nerve (this does not apply to the little finger and part
of the ring finger)
Muscular innervation and effect of denervation
Anatomical
location
Muscle affected Effect of paralysis
Shoulder Long head of triceps Minor effects on shoulder
stability in abduction
Arm Triceps Loss of elbow extension
Forearm Supinator
Brachioradialis
Extensor carpi radialis
longus and brevis
Weakening of supination of
prone hand and elbow flexion in
mid prone position
The cutaneous sensation of the upper limb- illustrating the
contribution of the radial nerve
Image sourced from Wikipedia
A. Teres major
B. Pectoralis major
C. Coracobrachialis
D. Supraspinatus
E. Latissimus dorsi
Next question
Supraspinatus is an abductor of the shoulder.
Shoulder joint
 Shallow synovial ball and socket type of joint.
 It is an inherently unstable joint, but is capable to a wide range
of movement.
 Stability is provided by muscles of the rotator cuff that pass
from the scapula to insert in the greater tuberosity (all except
sub scapularis-lesser tuberosity).
Glenoid labrum
 Fibrocartilaginous rim attached to the free edge of the glenoid
cavity
 Tendon of the long head of biceps arises from within the joint
from the supraglenoid tubercle, and is fused at this point to the
labrum.
 The long head of triceps attaches to the infraglenoid tubercle
Fibrous capsule
 Attaches to the scapula external to the glenoid labrum and to
the labrum itself (postero-superiorly)
 Attaches to the humerus at the level of the anatomical neck
superiorly and the surgical neck inferiorly
 Anteriorly the capsule is in contact with the tendon of
subscapularis, superiorly with the supraspinatus tendon, and
posteriorly with the tendons of infraspinatus and teres minor.
All these blend with the capsule towards their insertion.
 Two defects in the fibrous capsule; superiorly for the tendon of
biceps. Anteriorly there is a defect beneath the subscapularis
tendon.
 The inferior extension of the capsule is closely related to the
axillary nerve at the surgical neck and this nerve is at risk in
anteroinferior dislocations. It also means that proximally sited
osteomyelitis may progress to septic arthritis.
Movements and muscles
Flexion Anterior part of deltoid
Pectoralis major
Biceps
Coracobrachialis
Extension Posterior deltoid
Teres major
Latissimus dorsi
Adduction Pectoralis major
Latissimus dorsi
Teres major
Coracobrachialis
Abduction Mid deltoid
Supraspinatus
Medial rotation Subscapularis
Anterior deltoid
Teres major
Latissimus dorsi
Lateral rotation Posterior deltoid
Infraspinatus
Teres minor
Important anatomical relations
Anteriorly Brachial plexus
Axillary artery and vein
Posterior Suprascapular nerve
Suprascapular vessels
Inferior Axillary nerve
Circumflex humeral vessels
low?
A. External jugular
B. Axillary
C. Internal jugular
D. Azygos
E. Brachial
Next question
Cephalic vein
Path
 Dorsal venous arch drains laterally into the cephalic vein
 Crosses the anatomical snuffbox and travels laterally up the
arm
 At the antecubital fossa connected to the basilic vein by the
median cubital vein
 Pierces deep fascia of deltopectoral groove to join axillary vein
 Which of the following structures is not closely related to the
brachial artery?
A. Ulnar nerve
B. Median nerve
C. Cephalic vein
D. Long head of triceps
E. Median cubital vein
 Next question

The cephalic vein lies superficially and on the contralateral side
of the arm to the brachial artery. The relation of the ulnar
nerves and others are demonstrated in the image below:
Image sourced from Wikipedia

 Brachial artery

The brachial artery begins at the lower border of teres major as
a continuation of the axillary artery. It terminates in the cubital
fossa at the level of the neck of the radius by dividing into the
radial and ulnar arteries.
Relations
Posterior relations include the long head of triceps with the
radial nerve and profunda vessels intervening. Anteriorly it is
overlapped by the medial border of biceps.
It is crossed by the median nerve in the middle of the arm.
In the cubital fossa it is separated from the median cubital vein
by the bicipital aponeurosis.
The basilic vein is in contact at the most proximal aspect of the
cubital fossa and lies medially.
The following statements relating to the musculocutaneous nerve are
true except?
A. It arises from the lateral cord of the brachial plexus
B. It provides cutaneous innervation to the lateral side of the
forearm
C. If damaged then extension of the elbow joint will be impaired
D. It supplies the biceps muscle
E. It runs beneath biceps
Next question
It supplies biceps, brachialis and coracobrachialis so if damaged then
elbow flexion will be impaired.
Musculocutaneous nerve
 Branch of lateral cord of brachial plexus
Path
 It penetrates the Coracobrachialis muscle
 Passes obliquely between the Biceps brachii and the Brachialis
to the lateral side of the arm
 Above the elbow it pierces the deep fascia lateral to the tendon
of the Biceps brachii
 Continues into the forearm as the lateral cutaneous nerve of
the forearm
Innervates
 Coracobrachialis
 Biceps brachii
 Brachialis
Which ligament keeps the head of the radius connected to the radial
notch of the ulna?
A. Annular (orbicular) ligament
B. Quadrate ligament
C. Radial collateral ligament of the elbow
D. Ulnar collateral ligament
E. Radial collateral ligament
Next question
The annular ligament connects the radial head to the radial notch of
the ulna. This is illustrated below:
Image sourced from Wikipedia
Radius
 Bone of the forearm extending from the lateral side of the
elbow to the thumb side of the wrist
Upper end
 Articular cartilage- covers medial > lateral side
 Articulates with radial notch of the ulna by the annular ligament
 Muscle attachment- biceps brachii at the tuberosity
Shaft
 Muscle attachment-
Upper third of the body Supinator, Flexor digitorum superficialis,
Flexor pollicis longus
Middle third of the body Pronator teres
Lower quarter of the body Pronator quadratus , tendon of supinator
longus
Lower end
 Quadrilateral
 Anterior surface- capsule of wrist joint
 Medial surface- head of ulna
 Lateral surface- ends in the styloid process
 Posterior surface: 3 grooves containing:
1. Tendons of extensor carpi radialis longus and brevis
2. Tendon of extensor pollicis longus
3. Tendon of extensor indicis
Image sourced from Wikipedia
A 38 year old man presents to the clinic with shoulder weakness. On
examination he has an inability to initiate shoulder abduction. Which
of the nerves listed below is least likely to be functioning normally?
A. Suprascapular nerve
B. Medial pectoral nerve
C. Axillary nerve
D. Median nerve
E. Radial nerve
Next question
Theme from April 2012 Exam
Suprascapular nerve
The suprascapular nerve arises from the upper trunk of the brachial
plexus. It lies superior to the trunks of the brachial plexus and passes
inferolaterally parallel to them. It passes through the scapular notch,
deep to trapezius. It innervates both supraspinatus and infraspinatus
and initiates abduction of the shoulder. If damaged, patients may be
able to abduct the shoulder by leaning over the affected side and
deltoid can then continue to abduct the shoulder.
Which of the following vessels provides the greatest contribution to
the arterial supply of the breast?
A. External mammary artery
B. Thoracoacromial artery
C. Internal mammary artery
D. Lateral thoracic artery
E. Subclavian artery
Next question
60% of the arterial supply to the breast is derived from the internal
mammary artery. The external mammary and lateral thoracic arteries
also make a significant (but lesser) contribution. This is of importance
clinically in performing reduction mammoplasty procedures.
Breast
The breast itself lies on a layer of pectoral fascia and the following
muscles:
1. Pectoralis major
2. Serratus anterior
3. External oblique
Image showing the topography of the female breast
Image sourced from Wikipedia
Breast anatomy
Nerve supply Branches of intercostal nerves from T4-T6.
Arterial supply  Internal mammary (thoracic) artery
 External mammary artery (laterally)
 Anterior intercostal arteries
 Thoraco-acromial artery
Venous drainage Superficial venous plexus to sub clavian, axillary and intercostal veins.
Lymphatic
drainage
 70% Axillary nodes
 Internal mammary chain
 Other lymphatic sites such as deep cervical and supraclavicular
fossa (later in disease)
A baby is found to have a Klumpke's palsy post delivery. Which of the
following is most likely to be present?
A. Loss of flexors of the wrist
B. Weak elbow flexion
C. Pronation of the forearm
D. Adducted shoulder
E. Shoulder medially rotated
Next question
Features of Klumpkes Paralysis
 Claw hand (MCP joints extended and IP joints flexed)
 Loss of sensation over medial aspect of forearm and hand
 Horner's syndrome
 Loss of flexors of the wrist
A C8, T1 root lesion is called Klumpke's paralysis and is caused by
delivery with the arm extended.
Brachial plexus
Origin Anterior rami of C5 to T1
Sections of the
plexus
 Roots, trunks, divisions, cords, branches
 Mnemonic:Real Teenagers Drink Cold Beer
Roots  Located in the posterior triangle
 Pass between scalenus anterior and medius
Trunks  Located posterior to middle third of clavicle
 Upper and middle trunks related superiorly to the subclavian
artery
 Lower trunk passes over 1st rib posterior to the subclavian
artery
Divisions Apex of axilla
Cords Related to axillary artery
Diagram illustrating the branches of the brachial plexus
Image sourced from Wikipedia
Cutaneous sensation of the upper limb
ith respect to the basilic vein, which statement is false?
A. Its deep anatomical location makes it unsuitable for use as an
arteriovenous access site in fistula surgery
B. It originates from the dorsal venous network on the hand
C. It travels up the medial aspect of the forearm
D. Halfway between the shoulder and the elbow it lies deep to
muscle
E. It joins the brachial vein to form the axillary vein
Next question
It is used in arteriovenous fistula surgery during a procedure known
as a basilic vein transposition.
Basilic vein
The basilic and cephalic veins both provide the main pathways of
venous drainage for the arm and hand. It is continuous with the
palmar venous arch distally and the axillary vein proximally.
Path
 Originates on the medial side of the dorsal venous network of
the hand, and passes up the forearm and arm.
 Most of its course is superficial.
 Near the region anterior to the cubital fossa the vein joins the
cephalic vein.
 Midway up the humerus the basilic vein passes deep under the
muscles.
 At the lower border of the teres major muscle, the anterior and
posterior circumflex humeral veins feed into it.
 Joins the brachial veins to form the axillary vein.
A 78 year old man is lifting a heavy object when a feels a pain in his
forearm and is unable to continue. He has a swelling over his upper
forearm. An MRI scan shows a small cuff of tendon still attached to
the radial tuberosity consistent with a recent tear. Which of the
following muscles has been injured?
A. Pronator teres
B. Supinator
C. Aconeus
D. Brachioradialis
E. Biceps brachii
Next question
Biceps inserts into the radial tuberosity. Distal injuries of this muscle
are rare but are reported and are clinically more important than more
proximal ruptures.
Radius
 Bone of the forearm extending from the lateral side of the
elbow to the thumb side of the wrist
Upper end
 Articular cartilage- covers medial > lateral side
 Articulates with radial notch of the ulna by the annular ligament
 Muscle attachment- biceps brachii at the tuberosity
Shaft
 Muscle attachment-
Upper third of the body Supinator, Flexor digitorum superficialis,
Flexor pollicis longus
Middle third of the body Pronator teres
Lower quarter of the body Pronator quadratus , tendon of supinator
longus
Lower end
 Quadrilateral
 Anterior surface- capsule of wrist joint
 Medial surface- head of ulna
 Lateral surface- ends in the styloid process
 Posterior surface: 3 grooves containing:
1. Tendons of extensor carpi radialis longus and brevis
2. Tendon of extensor pollicis longus
3. Tendon of extensor indicis
Which of the following is a branch of the third part of the axillary
artery?
A. Superior thoracic
B. Lateral thoracic
C. Dorsal scapular
D. Thoracoacromial
E. Posterior circumflex humeral
Next question
The other branches include:
 Subscapular
 Anterior circumflex humeral
Axilla
Boundaries of the axilla
Medially Chest wall and Serratus anterior
Laterally Humeral head
Floor Subscapularis
Anterior aspect Lateral border of Pectoralis major
Fascia Clavipectoral fascia
Content:
Long thoracic nerve (of
Bell)
Derived from C5-C7 and passes behind the brachial plexus to
enter the axilla. It lies on the medial chest wall and supplies
serratus anterior. Its location puts it at risk during axillary
surgery and damage will lead to winging of the scapula.
Thoracodorsal nerve
and thoracodorsal trunk
Innervate and vascularise latissimus dorsi.
Axillary vein Lies at the apex of the axilla, it is the continuation of the basilic
vein. Becomes the subclavian vein at the outer border of the first
rib.
Intercostobrachial Traverse the axillary lymph nodes and are often divided during
nerves axillary surgery. They provide cutaneous sensation to the axillary
skin.
Lymph nodes The axilla is the main site of lymphatic drainage for the breast.
Theme: Nerve lesions
A. Intercostobrachial
B. Median
C. Axillary
D. Radial
E. Ulnar
F. Musculocutaneous
G. Brachial plexus upper cord
H. Brachial plexus lower cord
Please select the most likely nerve injury for the scenarios given.
Each option may be used once, more than once or not at all.
268. A 23 year old rugby player sustains a Smiths Fracture. On examination
opposition of the thumb is markedly weakened.
Median
This high velocity injury can often produce significant angulation and
displacement. Both of these may impair the function of the median
nerve with loss of function of the muscles of the thenar eminence
269. A 45 year old lady recovering from a mastectomy and axillary node
clearance notices that sensation in her armpit is impaired.
Intercostobrachial
The intercostobrachial nerves are frequently injured during axillary
dissection. These nerves traverse the axilla and supply cutaneous
sensation.
270. An 8 year old boy falls onto an outstretched hand and sustains a
supracondylar fracture. In addition to a weak radial pulse the child is
noted to have loss of pronation of the affected hand.
Median
This is a common injury in children. In this case the angulation and
displacement have resulted in median nerve injury.
Next question
Brachial plexus
Origin Anterior rami of C5 to T1
Sections of the
plexus
 Roots, trunks, divisions, cords, branches
 Mnemonic:Real Teenagers Drink Cold Beer
Roots  Located in the posterior triangle
 Pass between scalenus anterior and medius
Trunks  Located posterior to middle third of clavicle
 Upper and middle trunks related superiorly to the subclavian
artery
 Lower trunk passes over 1st rib posterior to the subclavian
artery
Divisions Apex of axilla
Cords Related to axillary artery
Diagram illustrating the branches of the brachial plexus
Image sourced from Wikipedia
Cutaneous sensation of the upper limb
Image sourced from Wikipedia
froment's test which muscle function is tested?
A. Flexor pollicis longus
B. Adductor pollicis longus
C. Abductor pollicis brevis
D. Adductor pollicis
E. Opponens pollicis
Next question
Nerve signs
Froment's sign
 Assess for ulnar nerve palsy
 Adductor pollicis muscle function tested
 Hold a piece of paper between their thumb and index finger.
The object is then pulled away. If ulnar nerve palsy, unable to
hold the paper and will flex the flexor pollicis longus to
compensate (flexion of thumb at interphalangeal joint).
Phalen's test
 Assess carpal tunnel syndrome
 More sensitive than Tinel's sign
 Hold wrist in maximum flexion and the test is positive if there is
numbness in the median nerve distribution.
Tinel's sign
 Assess for carpal tunnel syndrome
 Tap the median nerve at the wrist and the test is positive if
there is tingling/electric-like sensations over the distribution of
the median nerve.
heme: Cutaneous innervation
A. Ulnar nerve
B. Fifth cervical spinal segment
C. Radial nerve
D. Musculocutaneous nerve
E. Median nerve
F. None of these
Please select the source of innervation for the region described.
Each option may be used once, more than once or not at all.
288. The skin on the palmar aspect of the thumb
Median nerve
The median nerve supplies cutaneous sensation to this region.
See diagram below
289. The nail bed of the index finger
You answered Radial nerve
The correct answer is Median nerve
290. The skin overlying the medial aspect of the palm
Ulnar nerve
This area is innervated by the ulnar nerve.
Next question
Brachial plexus
Origin Anterior rami of C5 to T1
Sections of the
plexus
 Roots, trunks, divisions, cords, branches
 Mnemonic:Real Teenagers Drink Cold Beer
Roots  Located in the posterior triangle
 Pass between scalenus anterior and medius
Trunks  Located posterior to middle third of clavicle
 Upper and middle trunks related superiorly to the subclavian
artery
 Lower trunk passes over 1st rib posterior to the subclavian
artery
Divisions Apex of axilla
Cords Related to axillary artery
Diagram illustrating the branches of the brachial plexus
Image sourced from Wikipedia
Cutaneous sensation of the upper limb
From which of the following foramina does the opthalmic branch of
the trigeminal nerve exit the skull?
A. Foramen ovale
MRCS preparation emrcs questions upperlimb
MRCS preparation emrcs questions upperlimb
MRCS preparation emrcs questions upperlimb
MRCS preparation emrcs questions upperlimb
MRCS preparation emrcs questions upperlimb
MRCS preparation emrcs questions upperlimb
MRCS preparation emrcs questions upperlimb
MRCS preparation emrcs questions upperlimb
MRCS preparation emrcs questions upperlimb
MRCS preparation emrcs questions upperlimb
MRCS preparation emrcs questions upperlimb
MRCS preparation emrcs questions upperlimb
MRCS preparation emrcs questions upperlimb
MRCS preparation emrcs questions upperlimb
MRCS preparation emrcs questions upperlimb
MRCS preparation emrcs questions upperlimb
MRCS preparation emrcs questions upperlimb
MRCS preparation emrcs questions upperlimb
MRCS preparation emrcs questions upperlimb
MRCS preparation emrcs questions upperlimb
MRCS preparation emrcs questions upperlimb
MRCS preparation emrcs questions upperlimb
MRCS preparation emrcs questions upperlimb
MRCS preparation emrcs questions upperlimb
MRCS preparation emrcs questions upperlimb
MRCS preparation emrcs questions upperlimb
MRCS preparation emrcs questions upperlimb
MRCS preparation emrcs questions upperlimb
MRCS preparation emrcs questions upperlimb
MRCS preparation emrcs questions upperlimb
MRCS preparation emrcs questions upperlimb
MRCS preparation emrcs questions upperlimb
MRCS preparation emrcs questions upperlimb
MRCS preparation emrcs questions upperlimb
MRCS preparation emrcs questions upperlimb
MRCS preparation emrcs questions upperlimb
MRCS preparation emrcs questions upperlimb
MRCS preparation emrcs questions upperlimb
MRCS preparation emrcs questions upperlimb
MRCS preparation emrcs questions upperlimb
MRCS preparation emrcs questions upperlimb
MRCS preparation emrcs questions upperlimb
MRCS preparation emrcs questions upperlimb
MRCS preparation emrcs questions upperlimb
MRCS preparation emrcs questions upperlimb
MRCS preparation emrcs questions upperlimb
MRCS preparation emrcs questions upperlimb
MRCS preparation emrcs questions upperlimb
MRCS preparation emrcs questions upperlimb
MRCS preparation emrcs questions upperlimb
MRCS preparation emrcs questions upperlimb
MRCS preparation emrcs questions upperlimb
MRCS preparation emrcs questions upperlimb
MRCS preparation emrcs questions upperlimb
MRCS preparation emrcs questions upperlimb
MRCS preparation emrcs questions upperlimb
MRCS preparation emrcs questions upperlimb
MRCS preparation emrcs questions upperlimb
MRCS preparation emrcs questions upperlimb
MRCS preparation emrcs questions upperlimb
MRCS preparation emrcs questions upperlimb
MRCS preparation emrcs questions upperlimb
MRCS preparation emrcs questions upperlimb
MRCS preparation emrcs questions upperlimb
MRCS preparation emrcs questions upperlimb
MRCS preparation emrcs questions upperlimb
MRCS preparation emrcs questions upperlimb
MRCS preparation emrcs questions upperlimb
MRCS preparation emrcs questions upperlimb
MRCS preparation emrcs questions upperlimb
MRCS preparation emrcs questions upperlimb
MRCS preparation emrcs questions upperlimb
MRCS preparation emrcs questions upperlimb
MRCS preparation emrcs questions upperlimb

More Related Content

What's hot

Purvi shah femur bone ppt
Purvi shah femur bone  pptPurvi shah femur bone  ppt
Purvi shah femur bone pptPurvi Shah
 
Anatomy of Cerebellum - Dr.Sajith MD RD
Anatomy of Cerebellum - Dr.Sajith MD RD Anatomy of Cerebellum - Dr.Sajith MD RD
Anatomy of Cerebellum - Dr.Sajith MD RD Sajith Selvaganesan
 
Muscular anatomy of upper limb, MRI Anatomy
Muscular anatomy of upper limb, MRI AnatomyMuscular anatomy of upper limb, MRI Anatomy
Muscular anatomy of upper limb, MRI AnatomyBishnu Khatiwada
 
Shoulder dislocation Co Norbert
Shoulder dislocation  Co Norbert Shoulder dislocation  Co Norbert
Shoulder dislocation Co Norbert Icyizanye Norbert
 
Slideshow: Forearm Superficial Flexors
Slideshow: Forearm Superficial FlexorsSlideshow: Forearm Superficial Flexors
Slideshow: Forearm Superficial FlexorsThe Funky Professor
 
LIGAMENTOUS ANATOMY OF KNEE
LIGAMENTOUS ANATOMY OF KNEELIGAMENTOUS ANATOMY OF KNEE
LIGAMENTOUS ANATOMY OF KNEETamilSelvan470
 
4. spaces of the hand & its applied anatomy[1]
4. spaces of the hand & its applied anatomy[1]4. spaces of the hand & its applied anatomy[1]
4. spaces of the hand & its applied anatomy[1]MBBS IMS MSU
 
Radiological anatomy of shoulder joint
Radiological anatomy of shoulder jointRadiological anatomy of shoulder joint
Radiological anatomy of shoulder jointChandan Prasad
 
Anterior approach to shoulder
Anterior approach to shoulderAnterior approach to shoulder
Anterior approach to shoulderBipulBorthakur
 
Anatomy of flexor compartment of forearm
Anatomy of  flexor compartment of forearm Anatomy of  flexor compartment of forearm
Anatomy of flexor compartment of forearm Shaifaly madan rustagi
 
skeleton on head + bony features of cervical vertebrae 2018
skeleton on head + bony features of  cervical vertebrae 2018skeleton on head + bony features of  cervical vertebrae 2018
skeleton on head + bony features of cervical vertebrae 2018Ahmed ElNashar
 
Pterygopalatine fossa
Pterygopalatine fossaPterygopalatine fossa
Pterygopalatine fossaChitransha03
 

What's hot (20)

Third ventricle
Third ventricle Third ventricle
Third ventricle
 
Purvi shah femur bone ppt
Purvi shah femur bone  pptPurvi shah femur bone  ppt
Purvi shah femur bone ppt
 
Anatomy of Cerebellum - Dr.Sajith MD RD
Anatomy of Cerebellum - Dr.Sajith MD RD Anatomy of Cerebellum - Dr.Sajith MD RD
Anatomy of Cerebellum - Dr.Sajith MD RD
 
Muscular anatomy of upper limb, MRI Anatomy
Muscular anatomy of upper limb, MRI AnatomyMuscular anatomy of upper limb, MRI Anatomy
Muscular anatomy of upper limb, MRI Anatomy
 
Shoulder dislocation Co Norbert
Shoulder dislocation  Co Norbert Shoulder dislocation  Co Norbert
Shoulder dislocation Co Norbert
 
Slideshow: Forearm Superficial Flexors
Slideshow: Forearm Superficial FlexorsSlideshow: Forearm Superficial Flexors
Slideshow: Forearm Superficial Flexors
 
LIGAMENTOUS ANATOMY OF KNEE
LIGAMENTOUS ANATOMY OF KNEELIGAMENTOUS ANATOMY OF KNEE
LIGAMENTOUS ANATOMY OF KNEE
 
4. spaces of the hand & its applied anatomy[1]
4. spaces of the hand & its applied anatomy[1]4. spaces of the hand & its applied anatomy[1]
4. spaces of the hand & its applied anatomy[1]
 
Radiological anatomy of shoulder joint
Radiological anatomy of shoulder jointRadiological anatomy of shoulder joint
Radiological anatomy of shoulder joint
 
Ankle Joint.pptx
Ankle Joint.pptxAnkle Joint.pptx
Ankle Joint.pptx
 
Anterior approach to shoulder
Anterior approach to shoulderAnterior approach to shoulder
Anterior approach to shoulder
 
Exam Questions Ankle Joint
Exam Questions Ankle JointExam Questions Ankle Joint
Exam Questions Ankle Joint
 
Slideshow: Adductor Canal
Slideshow: Adductor CanalSlideshow: Adductor Canal
Slideshow: Adductor Canal
 
Anatomy of flexor compartment of forearm
Anatomy of  flexor compartment of forearm Anatomy of  flexor compartment of forearm
Anatomy of flexor compartment of forearm
 
skeleton on head + bony features of cervical vertebrae 2018
skeleton on head + bony features of  cervical vertebrae 2018skeleton on head + bony features of  cervical vertebrae 2018
skeleton on head + bony features of cervical vertebrae 2018
 
Anatomy upper limb mrcs part a
Anatomy upper limb mrcs part a Anatomy upper limb mrcs part a
Anatomy upper limb mrcs part a
 
Pilon fracture
Pilon fracture Pilon fracture
Pilon fracture
 
Pterygopalatine fossa
Pterygopalatine fossaPterygopalatine fossa
Pterygopalatine fossa
 
X ray of elbow joint
X ray of elbow jointX ray of elbow joint
X ray of elbow joint
 
elbow joint .ppt
elbow joint .pptelbow joint .ppt
elbow joint .ppt
 

Viewers also liked

Tips for orthopedics exam
Tips for orthopedics examTips for orthopedics exam
Tips for orthopedics examKareem Hamimy
 
MRCS preparation emrcs questions Pathology
MRCS preparation emrcs questions PathologyMRCS preparation emrcs questions Pathology
MRCS preparation emrcs questions PathologyFaisol Kabir
 
BPH and obstructive uropathy
BPH and obstructive uropathy BPH and obstructive uropathy
BPH and obstructive uropathy Ahmed Tawfeek
 
How to observe In Surgery
How to observe In SurgeryHow to observe In Surgery
How to observe In SurgeryKareem Hamimy
 
liver mass - how to investigate?
liver mass - how to investigate?liver mass - how to investigate?
liver mass - how to investigate?hr77
 
Local Flaps For Lower Limb Reconstruction Version1
Local Flaps  For  Lower Limb Reconstruction Version1Local Flaps  For  Lower Limb Reconstruction Version1
Local Flaps For Lower Limb Reconstruction Version1Dr Anshul Govila
 
Intercollegiate MRCS Examiners Newsletter Volume 2
Intercollegiate MRCS Examiners Newsletter Volume 2Intercollegiate MRCS Examiners Newsletter Volume 2
Intercollegiate MRCS Examiners Newsletter Volume 2meducationdotnet
 
MRCS preparation eMrcs questions surgery
MRCS preparation eMrcs questions   surgeryMRCS preparation eMrcs questions   surgery
MRCS preparation eMrcs questions surgeryFaisol Kabir
 
Mrcs Part A Experience
Mrcs Part A ExperienceMrcs Part A Experience
Mrcs Part A ExperienceKareem Hamimy
 
Investigations of lymphatics
Investigations of lymphaticsInvestigations of lymphatics
Investigations of lymphaticsKareem Hamimy
 

Viewers also liked (20)

Tips for orthopedics exam
Tips for orthopedics examTips for orthopedics exam
Tips for orthopedics exam
 
Trauma
TraumaTrauma
Trauma
 
Scrotal disorders
Scrotal disorders Scrotal disorders
Scrotal disorders
 
Anuria & acute retention
Anuria & acute retentionAnuria & acute retention
Anuria & acute retention
 
Tips on using the MRC pre-ortho Trainer in myofunctional orthodontics
Tips on using the MRC pre-ortho Trainer in myofunctional orthodonticsTips on using the MRC pre-ortho Trainer in myofunctional orthodontics
Tips on using the MRC pre-ortho Trainer in myofunctional orthodontics
 
MRCS preparation emrcs questions Pathology
MRCS preparation emrcs questions PathologyMRCS preparation emrcs questions Pathology
MRCS preparation emrcs questions Pathology
 
BPH and obstructive uropathy
BPH and obstructive uropathy BPH and obstructive uropathy
BPH and obstructive uropathy
 
How to observe In Surgery
How to observe In SurgeryHow to observe In Surgery
How to observe In Surgery
 
liver mass - how to investigate?
liver mass - how to investigate?liver mass - how to investigate?
liver mass - how to investigate?
 
MRC's Decision Support Framework
MRC's Decision Support FrameworkMRC's Decision Support Framework
MRC's Decision Support Framework
 
New MRCS Part A Exam format
New MRCS Part A Exam formatNew MRCS Part A Exam format
New MRCS Part A Exam format
 
Local Flaps For Lower Limb Reconstruction Version1
Local Flaps  For  Lower Limb Reconstruction Version1Local Flaps  For  Lower Limb Reconstruction Version1
Local Flaps For Lower Limb Reconstruction Version1
 
Intercollegiate MRCS Examiners Newsletter Volume 2
Intercollegiate MRCS Examiners Newsletter Volume 2Intercollegiate MRCS Examiners Newsletter Volume 2
Intercollegiate MRCS Examiners Newsletter Volume 2
 
Emergencies (non traumatic)
Emergencies (non traumatic)Emergencies (non traumatic)
Emergencies (non traumatic)
 
MRCS preparation eMrcs questions surgery
MRCS preparation eMrcs questions   surgeryMRCS preparation eMrcs questions   surgery
MRCS preparation eMrcs questions surgery
 
Mrcs Part A Experience
Mrcs Part A ExperienceMrcs Part A Experience
Mrcs Part A Experience
 
Access to urinary system v2
Access to urinary system v2Access to urinary system v2
Access to urinary system v2
 
MRC case
MRC case MRC case
MRC case
 
Investigations of lymphatics
Investigations of lymphaticsInvestigations of lymphatics
Investigations of lymphatics
 
MRCS Syllabus and recommended textbooks
MRCS Syllabus and recommended textbooksMRCS Syllabus and recommended textbooks
MRCS Syllabus and recommended textbooks
 

Similar to MRCS preparation emrcs questions upperlimb

peripheral nerves of the upper limb - applied
peripheral nerves of the upper limb - appliedperipheral nerves of the upper limb - applied
peripheral nerves of the upper limb - appliedSumer Yadav
 
12peripheralnervesoftheupperlimb-120312033920-phpapp01-141228111052-conversio...
12peripheralnervesoftheupperlimb-120312033920-phpapp01-141228111052-conversio...12peripheralnervesoftheupperlimb-120312033920-phpapp01-141228111052-conversio...
12peripheralnervesoftheupperlimb-120312033920-phpapp01-141228111052-conversio...WdEaAlBoNy
 
Arterial Supply of Upper Limb
Arterial Supply of Upper LimbArterial Supply of Upper Limb
Arterial Supply of Upper LimbAsish Rajak
 
Elbow Region.pptx
Elbow Region.pptxElbow Region.pptx
Elbow Region.pptxChiibeneme
 
RADIAL NERVE palsy.pptx
RADIAL NERVE palsy.pptxRADIAL NERVE palsy.pptx
RADIAL NERVE palsy.pptxhariramhalder
 
Surgical anatomy of upper limb nerves and plexus
Surgical anatomy of upper limb nerves and plexusSurgical anatomy of upper limb nerves and plexus
Surgical anatomy of upper limb nerves and plexusPirah Azadi
 
Peripheral Nerves of Upper Limb
Peripheral Nerves of Upper LimbPeripheral Nerves of Upper Limb
Peripheral Nerves of Upper LimbSado Anatomist
 
Anatomy of Upper extremity
Anatomy of Upper extremityAnatomy of Upper extremity
Anatomy of Upper extremitySunil Pahari
 
Blood supply to the upper limb
Blood supply to the upper limbBlood supply to the upper limb
Blood supply to the upper limbkwizera denis
 
Blood supply of upper limb by Dr-Ismail Khan
Blood supply of upper limb by Dr-Ismail KhanBlood supply of upper limb by Dr-Ismail Khan
Blood supply of upper limb by Dr-Ismail KhanDr-Ismail Khan
 
Anatomy of brachial plexus
Anatomy of brachial plexusAnatomy of brachial plexus
Anatomy of brachial plexusPrince Lathiya
 
Forearm anatomy complete
Forearm anatomy completeForearm anatomy complete
Forearm anatomy completeVedikaAgrawal13
 
ARTERIAL SUPPLY OF UPPER LIMBThe arterial supply to the .docx
ARTERIAL SUPPLY OF UPPER LIMBThe arterial supply to the .docxARTERIAL SUPPLY OF UPPER LIMBThe arterial supply to the .docx
ARTERIAL SUPPLY OF UPPER LIMBThe arterial supply to the .docxrossskuddershamus
 
MUSCLES of the limbs.ppt
MUSCLES of the limbs.pptMUSCLES of the limbs.ppt
MUSCLES of the limbs.pptDaniyar19
 
PowerPointHandout_ArmCubitalFossaElbow.pptx
PowerPointHandout_ArmCubitalFossaElbow.pptxPowerPointHandout_ArmCubitalFossaElbow.pptx
PowerPointHandout_ArmCubitalFossaElbow.pptxShivani Bhardwaj
 

Similar to MRCS preparation emrcs questions upperlimb (20)

peripheral nerve injuries
peripheral nerve injuriesperipheral nerve injuries
peripheral nerve injuries
 
peripheral nerves of the upper limb - applied
peripheral nerves of the upper limb - appliedperipheral nerves of the upper limb - applied
peripheral nerves of the upper limb - applied
 
12peripheralnervesoftheupperlimb-120312033920-phpapp01-141228111052-conversio...
12peripheralnervesoftheupperlimb-120312033920-phpapp01-141228111052-conversio...12peripheralnervesoftheupperlimb-120312033920-phpapp01-141228111052-conversio...
12peripheralnervesoftheupperlimb-120312033920-phpapp01-141228111052-conversio...
 
forearm.ppt
forearm.pptforearm.ppt
forearm.ppt
 
Arterial Supply of Upper Limb
Arterial Supply of Upper LimbArterial Supply of Upper Limb
Arterial Supply of Upper Limb
 
Elbow Region.pptx
Elbow Region.pptxElbow Region.pptx
Elbow Region.pptx
 
RADIAL NERVE palsy.pptx
RADIAL NERVE palsy.pptxRADIAL NERVE palsy.pptx
RADIAL NERVE palsy.pptx
 
Surgical anatomy of upper limb nerves and plexus
Surgical anatomy of upper limb nerves and plexusSurgical anatomy of upper limb nerves and plexus
Surgical anatomy of upper limb nerves and plexus
 
Peripheral Nerves of Upper Limb
Peripheral Nerves of Upper LimbPeripheral Nerves of Upper Limb
Peripheral Nerves of Upper Limb
 
Anatomy of Upper extremity
Anatomy of Upper extremityAnatomy of Upper extremity
Anatomy of Upper extremity
 
Blood supply to the upper limb
Blood supply to the upper limbBlood supply to the upper limb
Blood supply to the upper limb
 
Blood supply of upper limb by Dr-Ismail Khan
Blood supply of upper limb by Dr-Ismail KhanBlood supply of upper limb by Dr-Ismail Khan
Blood supply of upper limb by Dr-Ismail Khan
 
Forearm Anatomy
Forearm AnatomyForearm Anatomy
Forearm Anatomy
 
The arm1
The arm1The arm1
The arm1
 
Anatomy of brachial plexus
Anatomy of brachial plexusAnatomy of brachial plexus
Anatomy of brachial plexus
 
Forearm anatomy complete
Forearm anatomy completeForearm anatomy complete
Forearm anatomy complete
 
ARTERIAL SUPPLY OF UPPER LIMBThe arterial supply to the .docx
ARTERIAL SUPPLY OF UPPER LIMBThe arterial supply to the .docxARTERIAL SUPPLY OF UPPER LIMBThe arterial supply to the .docx
ARTERIAL SUPPLY OF UPPER LIMBThe arterial supply to the .docx
 
MUSCLES of the limbs.ppt
MUSCLES of the limbs.pptMUSCLES of the limbs.ppt
MUSCLES of the limbs.ppt
 
PowerPointHandout_ArmCubitalFossaElbow.pptx
PowerPointHandout_ArmCubitalFossaElbow.pptxPowerPointHandout_ArmCubitalFossaElbow.pptx
PowerPointHandout_ArmCubitalFossaElbow.pptx
 
Femoral triangle
Femoral triangleFemoral triangle
Femoral triangle
 

Recently uploaded

Call Girls Aurangabad Just Call 9907093804 Top Class Call Girl Service Available
Call Girls Aurangabad Just Call 9907093804 Top Class Call Girl Service AvailableCall Girls Aurangabad Just Call 9907093804 Top Class Call Girl Service Available
Call Girls Aurangabad Just Call 9907093804 Top Class Call Girl Service AvailableDipal Arora
 
Call Girls Horamavu WhatsApp Number 7001035870 Meeting With Bangalore Escorts
Call Girls Horamavu WhatsApp Number 7001035870 Meeting With Bangalore EscortsCall Girls Horamavu WhatsApp Number 7001035870 Meeting With Bangalore Escorts
Call Girls Horamavu WhatsApp Number 7001035870 Meeting With Bangalore Escortsvidya singh
 
Call Girls Dehradun Just Call 9907093804 Top Class Call Girl Service Available
Call Girls Dehradun Just Call 9907093804 Top Class Call Girl Service AvailableCall Girls Dehradun Just Call 9907093804 Top Class Call Girl Service Available
Call Girls Dehradun Just Call 9907093804 Top Class Call Girl Service AvailableDipal Arora
 
Call Girls Ludhiana Just Call 9907093804 Top Class Call Girl Service Available
Call Girls Ludhiana Just Call 9907093804 Top Class Call Girl Service AvailableCall Girls Ludhiana Just Call 9907093804 Top Class Call Girl Service Available
Call Girls Ludhiana Just Call 9907093804 Top Class Call Girl Service AvailableDipal Arora
 
VIP Call Girls Indore Kirti 💚😋 9256729539 🚀 Indore Escorts
VIP Call Girls Indore Kirti 💚😋  9256729539 🚀 Indore EscortsVIP Call Girls Indore Kirti 💚😋  9256729539 🚀 Indore Escorts
VIP Call Girls Indore Kirti 💚😋 9256729539 🚀 Indore Escortsaditipandeya
 
Call Girls Nagpur Just Call 9907093804 Top Class Call Girl Service Available
Call Girls Nagpur Just Call 9907093804 Top Class Call Girl Service AvailableCall Girls Nagpur Just Call 9907093804 Top Class Call Girl Service Available
Call Girls Nagpur Just Call 9907093804 Top Class Call Girl Service AvailableDipal Arora
 
Call Girls Faridabad Just Call 9907093804 Top Class Call Girl Service Available
Call Girls Faridabad Just Call 9907093804 Top Class Call Girl Service AvailableCall Girls Faridabad Just Call 9907093804 Top Class Call Girl Service Available
Call Girls Faridabad Just Call 9907093804 Top Class Call Girl Service AvailableDipal Arora
 
Top Rated Bangalore Call Girls Mg Road ⟟ 8250192130 ⟟ Call Me For Genuine Sex...
Top Rated Bangalore Call Girls Mg Road ⟟ 8250192130 ⟟ Call Me For Genuine Sex...Top Rated Bangalore Call Girls Mg Road ⟟ 8250192130 ⟟ Call Me For Genuine Sex...
Top Rated Bangalore Call Girls Mg Road ⟟ 8250192130 ⟟ Call Me For Genuine Sex...narwatsonia7
 
💎VVIP Kolkata Call Girls Parganas🩱7001035870🩱Independent Girl ( Ac Rooms Avai...
💎VVIP Kolkata Call Girls Parganas🩱7001035870🩱Independent Girl ( Ac Rooms Avai...💎VVIP Kolkata Call Girls Parganas🩱7001035870🩱Independent Girl ( Ac Rooms Avai...
💎VVIP Kolkata Call Girls Parganas🩱7001035870🩱Independent Girl ( Ac Rooms Avai...Taniya Sharma
 
College Call Girls in Haridwar 9667172968 Short 4000 Night 10000 Best call gi...
College Call Girls in Haridwar 9667172968 Short 4000 Night 10000 Best call gi...College Call Girls in Haridwar 9667172968 Short 4000 Night 10000 Best call gi...
College Call Girls in Haridwar 9667172968 Short 4000 Night 10000 Best call gi...perfect solution
 
Call Girls Bareilly Just Call 9907093804 Top Class Call Girl Service Available
Call Girls Bareilly Just Call 9907093804 Top Class Call Girl Service AvailableCall Girls Bareilly Just Call 9907093804 Top Class Call Girl Service Available
Call Girls Bareilly Just Call 9907093804 Top Class Call Girl Service AvailableDipal Arora
 
Bangalore Call Girls Nelamangala Number 7001035870 Meetin With Bangalore Esc...
Bangalore Call Girls Nelamangala Number 7001035870  Meetin With Bangalore Esc...Bangalore Call Girls Nelamangala Number 7001035870  Meetin With Bangalore Esc...
Bangalore Call Girls Nelamangala Number 7001035870 Meetin With Bangalore Esc...narwatsonia7
 
Top Rated Bangalore Call Girls Ramamurthy Nagar ⟟ 8250192130 ⟟ Call Me For Ge...
Top Rated Bangalore Call Girls Ramamurthy Nagar ⟟ 8250192130 ⟟ Call Me For Ge...Top Rated Bangalore Call Girls Ramamurthy Nagar ⟟ 8250192130 ⟟ Call Me For Ge...
Top Rated Bangalore Call Girls Ramamurthy Nagar ⟟ 8250192130 ⟟ Call Me For Ge...narwatsonia7
 
Lucknow Call girls - 8800925952 - 24x7 service with hotel room
Lucknow Call girls - 8800925952 - 24x7 service with hotel roomLucknow Call girls - 8800925952 - 24x7 service with hotel room
Lucknow Call girls - 8800925952 - 24x7 service with hotel roomdiscovermytutordmt
 
(👑VVIP ISHAAN ) Russian Call Girls Service Navi Mumbai🖕9920874524🖕Independent...
(👑VVIP ISHAAN ) Russian Call Girls Service Navi Mumbai🖕9920874524🖕Independent...(👑VVIP ISHAAN ) Russian Call Girls Service Navi Mumbai🖕9920874524🖕Independent...
(👑VVIP ISHAAN ) Russian Call Girls Service Navi Mumbai🖕9920874524🖕Independent...Taniya Sharma
 
VIP Service Call Girls Sindhi Colony 📳 7877925207 For 18+ VIP Call Girl At Th...
VIP Service Call Girls Sindhi Colony 📳 7877925207 For 18+ VIP Call Girl At Th...VIP Service Call Girls Sindhi Colony 📳 7877925207 For 18+ VIP Call Girl At Th...
VIP Service Call Girls Sindhi Colony 📳 7877925207 For 18+ VIP Call Girl At Th...jageshsingh5554
 
Call Girls Coimbatore Just Call 9907093804 Top Class Call Girl Service Available
Call Girls Coimbatore Just Call 9907093804 Top Class Call Girl Service AvailableCall Girls Coimbatore Just Call 9907093804 Top Class Call Girl Service Available
Call Girls Coimbatore Just Call 9907093804 Top Class Call Girl Service AvailableDipal Arora
 
Call Girls Haridwar Just Call 9907093804 Top Class Call Girl Service Available
Call Girls Haridwar Just Call 9907093804 Top Class Call Girl Service AvailableCall Girls Haridwar Just Call 9907093804 Top Class Call Girl Service Available
Call Girls Haridwar Just Call 9907093804 Top Class Call Girl Service AvailableDipal Arora
 
Call Girls Gwalior Just Call 8617370543 Top Class Call Girl Service Available
Call Girls Gwalior Just Call 8617370543 Top Class Call Girl Service AvailableCall Girls Gwalior Just Call 8617370543 Top Class Call Girl Service Available
Call Girls Gwalior Just Call 8617370543 Top Class Call Girl Service AvailableDipal Arora
 
Night 7k to 12k Chennai City Center Call Girls 👉👉 7427069034⭐⭐ 100% Genuine E...
Night 7k to 12k Chennai City Center Call Girls 👉👉 7427069034⭐⭐ 100% Genuine E...Night 7k to 12k Chennai City Center Call Girls 👉👉 7427069034⭐⭐ 100% Genuine E...
Night 7k to 12k Chennai City Center Call Girls 👉👉 7427069034⭐⭐ 100% Genuine E...hotbabesbook
 

Recently uploaded (20)

Call Girls Aurangabad Just Call 9907093804 Top Class Call Girl Service Available
Call Girls Aurangabad Just Call 9907093804 Top Class Call Girl Service AvailableCall Girls Aurangabad Just Call 9907093804 Top Class Call Girl Service Available
Call Girls Aurangabad Just Call 9907093804 Top Class Call Girl Service Available
 
Call Girls Horamavu WhatsApp Number 7001035870 Meeting With Bangalore Escorts
Call Girls Horamavu WhatsApp Number 7001035870 Meeting With Bangalore EscortsCall Girls Horamavu WhatsApp Number 7001035870 Meeting With Bangalore Escorts
Call Girls Horamavu WhatsApp Number 7001035870 Meeting With Bangalore Escorts
 
Call Girls Dehradun Just Call 9907093804 Top Class Call Girl Service Available
Call Girls Dehradun Just Call 9907093804 Top Class Call Girl Service AvailableCall Girls Dehradun Just Call 9907093804 Top Class Call Girl Service Available
Call Girls Dehradun Just Call 9907093804 Top Class Call Girl Service Available
 
Call Girls Ludhiana Just Call 9907093804 Top Class Call Girl Service Available
Call Girls Ludhiana Just Call 9907093804 Top Class Call Girl Service AvailableCall Girls Ludhiana Just Call 9907093804 Top Class Call Girl Service Available
Call Girls Ludhiana Just Call 9907093804 Top Class Call Girl Service Available
 
VIP Call Girls Indore Kirti 💚😋 9256729539 🚀 Indore Escorts
VIP Call Girls Indore Kirti 💚😋  9256729539 🚀 Indore EscortsVIP Call Girls Indore Kirti 💚😋  9256729539 🚀 Indore Escorts
VIP Call Girls Indore Kirti 💚😋 9256729539 🚀 Indore Escorts
 
Call Girls Nagpur Just Call 9907093804 Top Class Call Girl Service Available
Call Girls Nagpur Just Call 9907093804 Top Class Call Girl Service AvailableCall Girls Nagpur Just Call 9907093804 Top Class Call Girl Service Available
Call Girls Nagpur Just Call 9907093804 Top Class Call Girl Service Available
 
Call Girls Faridabad Just Call 9907093804 Top Class Call Girl Service Available
Call Girls Faridabad Just Call 9907093804 Top Class Call Girl Service AvailableCall Girls Faridabad Just Call 9907093804 Top Class Call Girl Service Available
Call Girls Faridabad Just Call 9907093804 Top Class Call Girl Service Available
 
Top Rated Bangalore Call Girls Mg Road ⟟ 8250192130 ⟟ Call Me For Genuine Sex...
Top Rated Bangalore Call Girls Mg Road ⟟ 8250192130 ⟟ Call Me For Genuine Sex...Top Rated Bangalore Call Girls Mg Road ⟟ 8250192130 ⟟ Call Me For Genuine Sex...
Top Rated Bangalore Call Girls Mg Road ⟟ 8250192130 ⟟ Call Me For Genuine Sex...
 
💎VVIP Kolkata Call Girls Parganas🩱7001035870🩱Independent Girl ( Ac Rooms Avai...
💎VVIP Kolkata Call Girls Parganas🩱7001035870🩱Independent Girl ( Ac Rooms Avai...💎VVIP Kolkata Call Girls Parganas🩱7001035870🩱Independent Girl ( Ac Rooms Avai...
💎VVIP Kolkata Call Girls Parganas🩱7001035870🩱Independent Girl ( Ac Rooms Avai...
 
College Call Girls in Haridwar 9667172968 Short 4000 Night 10000 Best call gi...
College Call Girls in Haridwar 9667172968 Short 4000 Night 10000 Best call gi...College Call Girls in Haridwar 9667172968 Short 4000 Night 10000 Best call gi...
College Call Girls in Haridwar 9667172968 Short 4000 Night 10000 Best call gi...
 
Call Girls Bareilly Just Call 9907093804 Top Class Call Girl Service Available
Call Girls Bareilly Just Call 9907093804 Top Class Call Girl Service AvailableCall Girls Bareilly Just Call 9907093804 Top Class Call Girl Service Available
Call Girls Bareilly Just Call 9907093804 Top Class Call Girl Service Available
 
Bangalore Call Girls Nelamangala Number 7001035870 Meetin With Bangalore Esc...
Bangalore Call Girls Nelamangala Number 7001035870  Meetin With Bangalore Esc...Bangalore Call Girls Nelamangala Number 7001035870  Meetin With Bangalore Esc...
Bangalore Call Girls Nelamangala Number 7001035870 Meetin With Bangalore Esc...
 
Top Rated Bangalore Call Girls Ramamurthy Nagar ⟟ 8250192130 ⟟ Call Me For Ge...
Top Rated Bangalore Call Girls Ramamurthy Nagar ⟟ 8250192130 ⟟ Call Me For Ge...Top Rated Bangalore Call Girls Ramamurthy Nagar ⟟ 8250192130 ⟟ Call Me For Ge...
Top Rated Bangalore Call Girls Ramamurthy Nagar ⟟ 8250192130 ⟟ Call Me For Ge...
 
Lucknow Call girls - 8800925952 - 24x7 service with hotel room
Lucknow Call girls - 8800925952 - 24x7 service with hotel roomLucknow Call girls - 8800925952 - 24x7 service with hotel room
Lucknow Call girls - 8800925952 - 24x7 service with hotel room
 
(👑VVIP ISHAAN ) Russian Call Girls Service Navi Mumbai🖕9920874524🖕Independent...
(👑VVIP ISHAAN ) Russian Call Girls Service Navi Mumbai🖕9920874524🖕Independent...(👑VVIP ISHAAN ) Russian Call Girls Service Navi Mumbai🖕9920874524🖕Independent...
(👑VVIP ISHAAN ) Russian Call Girls Service Navi Mumbai🖕9920874524🖕Independent...
 
VIP Service Call Girls Sindhi Colony 📳 7877925207 For 18+ VIP Call Girl At Th...
VIP Service Call Girls Sindhi Colony 📳 7877925207 For 18+ VIP Call Girl At Th...VIP Service Call Girls Sindhi Colony 📳 7877925207 For 18+ VIP Call Girl At Th...
VIP Service Call Girls Sindhi Colony 📳 7877925207 For 18+ VIP Call Girl At Th...
 
Call Girls Coimbatore Just Call 9907093804 Top Class Call Girl Service Available
Call Girls Coimbatore Just Call 9907093804 Top Class Call Girl Service AvailableCall Girls Coimbatore Just Call 9907093804 Top Class Call Girl Service Available
Call Girls Coimbatore Just Call 9907093804 Top Class Call Girl Service Available
 
Call Girls Haridwar Just Call 9907093804 Top Class Call Girl Service Available
Call Girls Haridwar Just Call 9907093804 Top Class Call Girl Service AvailableCall Girls Haridwar Just Call 9907093804 Top Class Call Girl Service Available
Call Girls Haridwar Just Call 9907093804 Top Class Call Girl Service Available
 
Call Girls Gwalior Just Call 8617370543 Top Class Call Girl Service Available
Call Girls Gwalior Just Call 8617370543 Top Class Call Girl Service AvailableCall Girls Gwalior Just Call 8617370543 Top Class Call Girl Service Available
Call Girls Gwalior Just Call 8617370543 Top Class Call Girl Service Available
 
Night 7k to 12k Chennai City Center Call Girls 👉👉 7427069034⭐⭐ 100% Genuine E...
Night 7k to 12k Chennai City Center Call Girls 👉👉 7427069034⭐⭐ 100% Genuine E...Night 7k to 12k Chennai City Center Call Girls 👉👉 7427069034⭐⭐ 100% Genuine E...
Night 7k to 12k Chennai City Center Call Girls 👉👉 7427069034⭐⭐ 100% Genuine E...
 

MRCS preparation emrcs questions upperlimb

  • 1. A 43 year old man is stabbed outside a nightclub. He suffers a transection of his median nerve just as it leaves the brachial plexus. Which of the following features is least likely to ensue? A. Ulnar deviation of the wrist B. Complete loss of wrist flexion C. Loss of pronation D. Loss of flexion at the thumb joint E. Inability to oppose the thumb Next question Loss of the median nerve will result in loss of function of the flexor muscles. However, flexor carpi ulnaris will still function and produce ulnar deviation and some residual wrist flexion. High median nerve lesions result in complete loss of flexion at the thumb joint. Median nerve The median nerve is formed by the union of a lateral and medial root respectively from the lateral (C5,6,7) and medial (C8 and T1) cords of the brachial plexus; the medial root passes anterior to the third part of the axillary artery. The nerve descends lateral to the brachial artery, crosses to its medial side (usually passing anterior to the artery). It passes deep to the bicipital aponeurosis and the median cubital vein at the elbow. It passes between the two heads of the pronator teres muscle, and runs on the deep surface of flexor digitorum superficialis (within its fascial sheath). Near the wrist it becomes superficial between the tendons of flexor digitorum superficialis and flexor carpi radialis, deep to palmaris longus tendon. It passes deep to the flexor retinaculum to enter the palm, but lies anterior to the long flexor tendons within the carpal tunnel. Branches Region Branch Upper arm No branches, although the nerve commonly communicates with the musculocutaneous nerve Forearm Pronator teres Flexor carpi radialis Palmaris longus Flexor digitorum superficialis Flexor pollicis longus Flexor digitorum profundus (only the radial half) Distal forearm Palmar cutaneous branch Hand (Motor) Motor supply (LOAF)  Lateral 2 lumbricals  Opponens pollicis  Abductor pollicis brevis  Flexor pollicis brevis Hand (Sensory)  Over thumb and lateral 2 ½ fingers  On the palmar aspect this projects proximally, on the dorsal aspect only the distal regions are innervated with the radial nerve providing the more proximal cutaneous innervation.
  • 2. Patterns of damage Damage at wrist  e.g. carpal tunnel syndrome  paralysis and wasting of thenar eminence muscles and opponens pollicis (ape hand deformity)  sensory loss to palmar aspect of lateral (radial) 2 ½ fingers Damage at elbow, as above plus:  unable to pronate forearm  weak wrist flexion  ulnar deviation of wrist Anterior interosseous nerve (branch of median nerve)  leaves just below the elbow  results in loss of pronation of forearm and weakness of long flexors of thumb and index finger Topography of the median nerve
  • 3. A 24 year old man falls and sustains a fracture through his scaphoid bone. From which of the following areas does the scaphoid derive the majority of its blood supply? A. From its proximal medial border B. From its proximal lateral border C. From its proximal posterior surface D. From the proximal end E. From the distal end Next question Theme from April 2012 Exam The blood supply to the scaphoid enters from a small non articular surface near its distal end. Transverse fractures through the scaphoid therefore carry a risk of non union. Scaphoid bone The scaphoid has a concave articular surface for the head of the capitate and at the edge of this is a crescentic surface for the corresponding area on the lunate. Proximally, it has a wide convex articular surface with the radius. It has a distally sited tubercle that can be palpated. The remaining articular surface is to the lateral side of the tubercle. It faces laterally and is associated with the trapezium and trapezoid bones. The narrow strip between the radial and trapezial surfaces and the tubercle gives rise to the radial collateral carpal ligament. The tubercle receives part of the flexor retinaculum. This area is the only part of the scaphoid that is available for the entry of blood vessels. It is commonly fractured and avascular necrosis may result. A 56 year old man requires long term parenteral nutrition and the decision is made to insert a PICC line for long term venous access. This is inserted into the basilic vein at the region of the elbow. As the catheter is advanced, into which venous structure is the tip of the catheter most likely to pass from the basilic vein? A. Subclavian vein B. Axillary vein C. Posterior circumflex humeral vein
  • 4. D. Cephalic vein E. Superior vena cava Next question The basilic vein drains into the axillary vein and although PICC lines may end up in a variety of fascinating locations the axillary vein is usually the commonest site following from the basilic. The posterior circumflex humeral vein is encountered prior to the axillary vein. However, a PICC line is unlikely to enter this structure because of its angle of entry into the basilic vein. Basilic vein The basilic and cephalic veins both provide the main pathways of venous drainage for the arm and hand. It is continuous with the palmar venous arch distally and the axillary vein proximally. Path  Originates on the medial side of the dorsal venous network of the hand, and passes up the forearm and arm.  Most of its course is superficial.  Near the region anterior to the cubital fossa the vein joins the cephalic vein.
  • 5.  Midway up the humerus the basilic vein passes deep under the muscles.  At the lower border of the teres major muscle, the anterior and posterior circumflex humeral veins feed into it.  Joins the brachial veins to form the axillary vein. Theme: Nerve injury A. Ulnar nerve B. Musculocutaneous nerve C. Radial nerve D. Median nerve E. Axillary nerve F. Intercostobrachial nerve What is the most likely nerve injury for the scenario given? Each option may be used once, more than once or not at all. 16. A 23 year old man is involved in a fight outside a nightclub and sustains a laceration to his right arm. On examination he has lost extension of the fingers in his right hand. Radial nerve The radial nerve supplies the extensor muscle group.
  • 6. 17. A 40 year old lady trips and falls through a glass door and sustains a severe laceration to her left arm. Amongst her injuries it is noticed that she has lost the ability to adduct the fingers of her left hand. Ulnar nerve The interossei are supplied by the ulnar nerve. 18. A 28 year old rugby player injures his right humerus and on examination is noted to have a minor sensory deficit overlying the point of deltoid insertion into the humerus. Axillary nerve This patch of skin is supplied by the axillary nerve Next question Brachial plexus Origin Anterior rami of C5 to T1 Sections of the plexus  Roots, trunks, divisions, cords, branches  Mnemonic:Real Teenagers Drink Cold Beer Roots  Located in the posterior triangle  Pass between scalenus anterior and medius
  • 7. Trunks  Located posterior to middle third of clavicle  Upper and middle trunks related superiorly to the subclavian artery  Lower trunk passes over 1st rib posterior to the subclavian artery Divisions Apex of axilla Cords Related to axillary artery Diagram illustrating the branches of the brachial plexus Image sourced from Wikipedia Cutaneous sensation of the upper limb
  • 8. Image sourced from Wikipedia A 21 year old man is stabbed in the antecubital fossa. A decision is made to surgically explore the wound. At operation the surgeon dissects down onto the brachial artery. A nerve is identified medially, which nerve is it likely to be? A. Radial B. Recurrent branch of median C. Anterior interosseous D. Ulnar E. Median Next question
  • 9. Theme from September 2012 Exam Median nerve The median nerve is formed by the union of a lateral and medial root respectively from the lateral (C5,6,7) and medial (C8 and T1) cords of the brachial plexus; the medial root passes anterior to the third part of the axillary artery. The nerve descends lateral to the brachial artery, crosses to its medial side (usually passing anterior to the artery). It passes deep to the bicipital aponeurosis and the median cubital vein at the elbow. It passes between the two heads of the pronator teres muscle, and runs on the deep surface of flexor digitorum superficialis (within its fascial sheath). Near the wrist it becomes superficial between the tendons of flexor digitorum superficialis and flexor carpi radialis, deep to palmaris longus tendon. It passes deep to the flexor retinaculum to enter the palm, but lies anterior to the long flexor tendons within the carpal tunnel. Branches Region Branch Upper arm No branches, although the nerve commonly communicates with the musculocutaneous nerve Forearm Pronator teres Flexor carpi radialis
  • 10. Palmaris longus Flexor digitorum superficialis Flexor pollicis longus Flexor digitorum profundus (only the radial half) Distal forearm Palmar cutaneous branch Hand (Motor) Motor supply (LOAF)  Lateral 2 lumbricals  Opponens pollicis  Abductor pollicis brevis  Flexor pollicis brevis Hand (Sensory)  Over thumb and lateral 2 ½ fingers  On the palmar aspect this projects proximally, on the dorsal aspect only the distal regions are innervated with the radial nerve providing the more proximal cutaneous innervation. Patterns of damage Damage at wrist  e.g. carpal tunnel syndrome  paralysis and wasting of thenar eminence muscles and opponens pollicis (ape hand deformity)  sensory loss to palmar aspect of lateral (radial) 2 ½ fingers Damage at elbow, as above plus:  unable to pronate forearm
  • 11.  weak wrist flexion  ulnar deviation of wrist Anterior interosseous nerve (branch of median nerve)  leaves just below the elbow  results in loss of pronation of forearm and weakness of long flexors of thumb and index finger Topography of the median nerve Image sourced fro A man sustains a laceration between the base of the little finger and wrist. Several weeks after the injury there is
  • 12. loss of thumb adduction power. Which nerve is most likely to have been injured? A. Superficial ulnar nerve B. Deep ulnar nerve C. Median nerve D. Radial nerve E. Recurrent branch of median nerve Next question Theme from 2009 Exam Ulnar nerve injury at wrist Branches of the ulnar nerve in the wrist and hand At the wrist the ulnar nerve divides into superficial and deep branches. The superficial branch lies deep to the palmaris brevis. It divides into two; to produce digital nerves, which innervate the skin of the medial third of the palm and the palmar surface of one and a half fingers. The deep branch arises from the nerve on the flexor retinaculum lateral to the pisiform bone. It passes posteriorly between the abductor and short flexor of the little finger supplying them, and supplying and piercing the opponens digiti minimi near its origin from the flexor retinaculum, turns laterally over the distal surface of the
  • 13. Hook of the Hamate bone. It eventually passes between the two heads of adductor pollicis with the deep palmar arch and ends in the first dorsal interosseous muscle. In the palm the deep branch also innervates the lumbricals and interosseous muscles. A 25 year old man is stabbed in the upper arm. The brachial artery is lacerated at the level of the proximal humerus, and is being repaired. A nerve lying immediately lateral to the brachial artery is also lacerated. Which of the following is the nerve most likely to be? A. Ulnar nerve B. Median nerve C. Radial nerve D. Intercostobrachial nerve E. Axillary nerve Next question The brachial artery begins at the lower border of teres major and terminates in the cubital fossa by branching into the radial and ulnar arteries. In the upper arm the median nerve lies closest to it in the lateral position. In the cubital fossa it lies medial to it.
  • 14. Image sourced from Wikipedia Brachial artery The brachial artery begins at the lower border of teres major as a continuation of the axillary artery. It terminates in the cubital fossa at the level of the neck of the radius by dividing into the radial and ulnar arteries. Relations Posterior relations include the long head of triceps with the radial nerve and profunda vessels intervening. Anteriorly it is overlapped by the medial border of biceps. It is crossed by the median nerve in the middle of the arm. In the cubital fossa it is separated from the median cubital vein by the bicipital aponeurosis.
  • 15. The basilic vein is in contact at the most proximal aspect of the cubital fossa and lies medially. What is the course of the median nerve relative to the brachial artery in the upper arm? A. Medial to anterior to lateral B. Lateral to posterior to medial C. Medial to posterior to lateral D. Medial to anterior to medial E. Lateral to anterior to medial Next question Relations of median nerve to the brachial artery: Lateral -> Anterior -> Medial Theme from 2009 and 2012 Exams The median nerve descends lateral to the brachial artery, it usually passes anterior to the artery to lie on its medial side. It passes deep to the bicipital aponeurosis and the median cubital vein at the elbow. It enters the forearm between the two heads of the pronator teres muscle.
  • 16. Image sourced from Wikipedia Brachial artery The brachial artery begins at the lower border of teres major as a continuation of the axillary artery. It terminates in the cubital fossa at the level of the neck of the radius by dividing into the radial and ulnar arteries. Relations Posterior relations include the long head of triceps with the radial nerve and profunda vessels intervening. Anteriorly it is overlapped by the medial border of biceps. It is crossed by the median nerve in the middle of the arm. In the cubital fossa it is separated from the median cubital vein by the bicipital aponeurosis. The basilic vein is in contact at the most proximal aspect of the cubital fossa and lies medially. A 22 year old falls over and lands on a shard of glass. It penetrates the palmar aspect of his hand, immediately lateral to the pisiform bone. Which of the following structures is most likely to be injured?
  • 17. A. Palmar cutaneous branch of the median nerve B. Lateral tendons of flexor digitorum superficialis C. Ulnar artery D. Flexor carpi radialis tendons E. Lateral tendons of flexor digitorum profundus Next question The ulnar nerve and artery are at most immediate risk in this injury. This is illustrated in the image below:
  • 18. Image sourced from Wikipedia Hand Anatomy of the hand Bones  8 Carpal bones  5 Metacarpals  14 phalanges Intrinsic Muscles 7 Interossei - Supplied by ulnar nerve  3 palmar-adduct fingers  4 dorsal- abduct fingers Intrinsic muscles Lumbricals  Flex MCPJ and extend the IPJ.  Origin deep flexor tendon and insertion dorsal extensor hood mechanism.  Innervation: 1st and 2nd- median nerve, 3rd and 4th- deep branch of the ulnar nerve. Thenar eminence  Abductor pollicis brevis  Opponens pollicis  Flexor pollicis brevis Hypothenar eminence  Opponens digiti minimi  Flexor digiti minimi brevis  Abductor digiti minimi
  • 19. A motorcyclist is involved in a road traffic accident. He suffers a complex humeral shaft fracture which is plated. Post operatively he complains of an inability to extend his fingers. Which of the following structures is most likely to have been injured? A. Ulnar nerve B. Radial nerve C. Median nerve D. Axillary nerve E. None of the above Next question
  • 20. Mnemonic for radial nerve muscles: BEST B rachioradialis E xtensors S upinator T riceps The radial nerve is responsible for innervation of the extensor compartment of the forearm. Radial nerve Continuation of posterior cord of the brachial plexus (root values C5 to T1) Path  In the axilla: lies posterior to the axillary artery on subscapularis, latissimus dorsi and teres major.  Enters the arm between the brachial artery and the long head of triceps (medial to humerus).  Spirals around the posterior surface of the humerus in the groove for the radial nerve.  At the distal third of the lateral border of the humerus it then pierces the intermuscular septum and descends in front of the lateral epicondyle.
  • 21.  At the lateral epicondyle it lies deeply between brachialis and brachioradialis where it then divides into a superficial and deep terminal branch.  Deep branch crosses the supinator to become the posterior interosseous nerve. In the image below the relationships of the radial nerve can be appreciated Image sourced from Wikipedia Regions innervated Motor (main nerve)  Triceps  Anconeus  Brachioradialis  Extensor carpi radialis Motor (posterior  Extensor carpi ulnaris  Extensor digitorum
  • 22. interosseous branch  Extensor indicis  Extensor digiti minimi  Extensor pollicis longus and brevis  Abductor pollicis longus Sensory The area of skin supplying the proximal phalanges on the dorsal aspect of the hand is supplied by the radial nerve (this does not apply to the little finger and part of the ring finger) Muscular innervation and effect of denervation Anatomical location Muscle affected Effect of paralysis Shoulder Long head of triceps Minor effects on shoulder stability in abduction Arm Triceps Loss of elbow extension Forearm Supinator Brachioradialis Extensor carpi radialis longus and brevis Weakening of supination of prone hand and elbow flexion in mid prone position The cutaneous sensation of the upper limb- illustrating the contribution of the radial nerve
  • 23. A 48 year old lady is undergoing an axillary node clearance for breast cancer. Which of the structures listed below are most likely to be encountered during the axillary dissection? A. Cords of the brachial plexus B. Thoracodorsal trunk C. Internal mammary artery D. Thoracoacromial artery E. None of the above Next question
  • 24. Beware of damaging the thoracodorsal trunk if a latissimus dorsi flap reconstruction is planned. Theme from 2009 Exam The thoracodorsal trunk runs through the nodes in the axilla. If injured it may compromise the function and blood supply to latissimus dorsi, which is significant if it is to be used as a flap for a reconstructive procedure. Axilla Boundaries of the axilla Medially Chest wall and Serratus anterior Laterally Humeral head Floor Subscapularis Anterior aspect Lateral border of Pectoralis major Fascia Clavipectoral fascia Content: Long thoracic nerve (of Bell) Derived from C5-C7 and passes behind the brachial plexus to enter the axilla. It lies on the medial chest wall and supplies serratus anterior. Its location puts it at risk during axillary surgery and damage will lead to winging of the scapula. Thoracodorsal nerve and thoracodorsal Innervate and vascularise latissimus dorsi.
  • 25. trunk Axillary vein Lies at the apex of the axilla, it is the continuation of the basilic vein. Becomes the subclavian vein at the outer border of the first rib. Intercostobrachial nerves Traverse the axillary lymph nodes and are often divided during axillary surgery. They provide cutaneous sensation to the axillary skin. Lymph nodes The axilla is the main site of lymphatic drainage for the breast. 53 year old lady is recovering following a difficult mastectomy and axillary nodal clearance for carcinoma of the breast. She complains of shoulder pain and on examination has obvious winging of the scapula. Loss of innervation to which of the following is the most likely underlying cause? A. Latissimus dorsi B. Serratus anterior C. Pectoralis minor D. Pectoralis major E. Rhomboids Next question Theme from April 2012 Exam Winging of the scapula is most commonly the result of
  • 26. long thoracic nerve injury or dysfunction. Iatrogenic damage during the course of the difficult axillary dissection is the most likely cause in this scenario. Damage to the rhomboids may produce winging of the scapula but would be rare in the scenario given. Long thoracic nerve  Derived from ventral rami of C5, C6, and C7 (close to their emergence from intervertebral foramina)  It runs downward and passes either anterior or posterior to the middle scalene muscle  It reaches upper tip of serratus anterior muscle and descends on outer surface of this muscle, giving branches into it  Winging of Scapula occurs in long thoracic nerve injury (most common) or from spinal accessory nerve injury (which denervates the trapezius) or a dorsal scapular nerve injury A 23 year old man falls and slips at a nightclub. A shard of glass penetrates the skin at the level of the medial epicondyle, which of the following sequelae is least likely to occur? A. Atrophy of the first dorsal interosseous muscle B. Difficulty in abduction of the the 2nd, 3rd, 4th and 5th fingers
  • 27. C. Claw like appearance of the hand D. Loss of sensation on the anterior aspect of the 5th finger E. Partial denervation of flexor digitorum profundus Next question Injury to the ulnar nerve in the mid to distal forearm will typically produce a claw hand. This consists of flexion of the 4th and 5th interphalangeal joints and extension of the metacarpophalangeal joints. The effects are potentiated when flexor digitorum profundus is not affected, and the clawing is more pronounced.More proximally sited ulnar nerve lesions produce a milder clinical picture owing to the simultaneous paralysis of flexor digitorum profundus (ulnar half). This is the 'ulnar paradox', due to the more proximal level of transection the hand will typically not have a claw like appearance that may be seen following a more distal injury. The first dorsal interosseous muscle will be affected as it is supplied by the ulnar nerve. Ulnar nerve Origin  C8, T1
  • 28. Supplies (no muscles in the upper arm)  Flexor carpi ulnaris  Flexor digitorum profundus  Flexor digiti minimi  Abductor digiti minimi  Opponens digiti minimi  Adductor pollicis  Interossei muscle  Third and fourth lumbricals  Palmaris brevis Path  Posteromedial aspect of ulna to flexor compartment of forearm, then along the ulnar. Passes beneath the flexor carpi ulnaris muscle, then superficially through the flexor retinaculum into the palm of the hand.
  • 29. Image sourced from Wikipedia Branches Branch Supplies Articular branch Flexor carpi ulnaris Medial half of the flexor digitorum profundus Palmar cutaneous branch (Arises near the middle of the forearm) Skin on the medial part of the palm Dorsal cutaneous branch Dorsal surface of the medial part of the hand Superficial branch Cutaneous fibres to the anterior surfaces of the medial one and one-half digits
  • 30. Deep branch Hypothenar muscles All the interosseous muscles Third and fourth lumbricals Adductor pollicis Medial head of the flexor pollicis brevis Effects of injury Damage at the wrist  Wasting and paralysis of intrinsic hand muscles (claw hand)  Wasting and paralysis of hypothenar muscles  Loss of sensation medial 1 and half fingers Damage at the elbow  Radial deviation of the wrist  Clawing less in 3rd and 4th digits A 43 year old lady is due to undergo an axillary node clearance as part of treatment for carcinoma of the breast. Which of the following fascial layers will be divided during the surgical approach to the axilla? A. Sibsons fascia B. Pre tracheal fascia C. Waldayers fascia D. Clavipectoral fascia E. None of the above Next question
  • 31. The clavipectoral fascia is situated under the clavicular portion of pectoralis major. It protects both the axillary vessels and nodes. During an axillary node clearance for breast cancer the clavipectoral fascia is incised and this allows access to the nodal stations. The nodal stations are; level 1 nodes inferior to pectoralis minor, level 2 lie behind it and level 3 above it. During a Patey Mastectomy surgeons divide pectoralis minor to gain access to level 3 nodes. The use of sentinel node biopsy (and stronger assistants!) have made this procedure far less common. A 23 year old climber falls and fractures his humerus. The surgeons decide upon a posterior approach to the middle third of the bone. Which of the following nerves is at greatest risk in this approach? A. Ulnar B. Antebrachial C. Musculocutaneous D. Radial E. Intercostobrachial Next question Theme from April 2012 Exam The radial nerve wraps around the humerus and may be
  • 32. injured during a posterior approach. An IM nail may be preferred as it avoids the complex dissection needed for direct bone exposure. Radial nerve Continuation of posterior cord of the brachial plexus (root values C5 to T1) Path  In the axilla: lies posterior to the axillary artery on subscapularis, latissimus dorsi and teres major.  Enters the arm between the brachial artery and the long head of triceps (medial to humerus).  Spirals around the posterior surface of the humerus in the groove for the radial nerve.  At the distal third of the lateral border of the humerus it then pierces the intermuscular septum and descends in front of the lateral epicondyle.  At the lateral epicondyle it lies deeply between brachialis and brachioradialis where it then divides into a superficial and deep terminal branch.  Deep branch crosses the supinator to become the posterior interosseous nerve. In the image below the relationships of the radial nerve can be appreciated
  • 33. Image sourced from Wikipedia Regions innervated Motor (main nerve)  Triceps  Anconeus  Brachioradialis  Extensor carpi radialis Motor (posterior interosseous branch  Extensor carpi ulnaris  Extensor digitorum  Extensor indicis  Extensor digiti minimi  Extensor pollicis longus and brevis  Abductor pollicis longus Sensory The area of skin supplying the proximal phalanges on the dorsal aspect of the hand is supplied by the radial nerve (this does not apply to the little finger and part of the ring finger)
  • 34. Muscular innervation and effect of denervation Anatomical location Muscle affected Effect of paralysis Shoulder Long head of triceps Minor effects on shoulder stability in abduction Arm Triceps Loss of elbow extension Forearm Supinator Brachioradialis Extensor carpi radialis longus and brevis Weakening of supination of prone hand and elbow flexion in mid prone position The cutaneous sensation of the upper limb- illustrating the contribution of the radial nerve Image sourced from Wikipedia
  • 35. heme: Nerve injury A. Median nerve B. Ulnar nerve C. Radial nerve D. Posterior interosseous nerve E. Anterior interosseous nerve F. Musculocutaneous nerve G. Axillary nerve H. Brachial Trunks C5-6 I. Brachial trunks C6-7 J. Brachial Trunks C8-T1 Please select the most likely lesion site for each scenario. Each option may be used once, more than once or not at all. 83. A 42 year old teacher is admitted with a fall. An x-ray confirms a fracture of the surgical neck of the humerus. Which nerve is at risk? Axillary nerve The Axillary nerve winds around the bone at the neck of the humerus. The axillary nerve is also at risk during shoulder dislocation.
  • 36. 84. A 32 year old window cleaner is admitted after falling off the roof. He reports that he had slipped off the top of the roof and was able to cling onto the gutter for a few seconds. The patient has Horner's syndrome. Brachial Trunks C8-T1 The patient has a Klumpke's paralysis involving brachial trunks C8-T1. Classically there is weakness of the hand intrinsic muscles. Involvement of T1 may cause a Horner's syndrome. It occurs as a result of traction injuries or during delivery. 85. A 32 year old rugby player is hit hard on the shoulder during a rough tackle. Clinically his arm is hanging loose on the side. It is pronated and medially rotated. Brachial Trunks C5-6 The patient has an Erb's palsy involving brachial trunks C5-6. Next question Brachial plexus Origin Anterior rami of C5 to T1 Sections of the  Roots, trunks, divisions, cords, branches
  • 37. plexus  Mnemonic:Real Teenagers Drink Cold Beer Roots  Located in the posterior triangle  Pass between scalenus anterior and medius Trunks  Located posterior to middle third of clavicle  Upper and middle trunks related superiorly to the subclavian artery  Lower trunk passes over 1st rib posterior to the subclavian artery Divisions Apex of axilla Cords Related to axillary artery Diagram illustrating the branches of the brachial plexus Image sourced from Wikipedia Cutaneous sensation of the upper limb
  • 38. Image sourced hich of the following nerves is responsible for innervation of the triceps muscle? A. Radial B. Ulnar C. Axillary D. Median E. None of the above Next question
  • 39. To remember nerve roots and their relexes: 1-2 Ankle (S1-S2) 3-4 Knee (L3-L4) 5-6 Biceps (C5-C6) 7-8 Triceps (C7-C8) The radial nerve innervates all three heads of triceps, with a separate branch to each head. Triceps Origin  Long head- infraglenoid tubercle of the scapula.  Lateral head- dorsal surface of the humerus, lateral and proximal to the groove of the radial nerve  Medial head- posterior surface of the humerus on the inferomedial side of the radial groove and both of the intermuscular septae Insertion  Olecranon process of the ulna. Here the olecranon bursa is between the triceps tendon and olecranon.  Some fibres insert to the deep fascia of the forearm, posterior capsule of the elbow (preventing the capsule from being trapped between olecranon and olecranon fossa during extension) Innervation Radial nerve Blood supply Profunda brachii artery
  • 40. Action Elbow extension. The long head can adduct the humerus and and extend it from a flexed position Relations The radial nerve and profunda brachii vessels lie between the lateral and medial heads Which of the following muscles inserts onto the lesser tuberostiy of the the humerus? A. Subscapularis B. Deltoid C. Supraspinatus D. Teres minor E. Infraspinatus Next question With the exception of subscapularis which inserts into the lesser tuberosity, the muscles of the rotator cuff insert into the greater tuberosity. Shoulder joint  Shallow synovial ball and socket type of joint.  It is an inherently unstable joint, but is capable to a wide range of movement.  Stability is provided by muscles of the rotator cuff that pass from the scapula to insert in the greater
  • 41. tuberosity (all except sub scapularis-lesser tuberosity). Glenoid labrum  Fibrocartilaginous rim attached to the free edge of the glenoid cavity  Tendon of the long head of biceps arises from within the joint from the supraglenoid tubercle, and is fused at this point to the labrum.  The long head of triceps attaches to the infraglenoid tubercle Fibrous capsule  Attaches to the scapula external to the glenoid labrum and to the labrum itself (postero-superiorly)  Attaches to the humerus at the level of the anatomical neck superiorly and the surgical neck inferiorly  Anteriorly the capsule is in contact with the tendon of subscapularis, superiorly with the supraspinatus tendon, and posteriorly with the tendons of infraspinatus and teres minor. All these blend with the capsule towards their insertion.  Two defects in the fibrous capsule; superiorly for the tendon of biceps. Anteriorly there is a defect beneath the subscapularis tendon.
  • 42.  The inferior extension of the capsule is closely related to the axillary nerve at the surgical neck and this nerve is at risk in anteroinferior dislocations. It also means that proximally sited osteomyelitis may progress to septic arthritis. Movements and muscles Flexion Anterior part of deltoid Pectoralis major Biceps Coracobrachialis Extension Posterior deltoid Teres major Latissimus dorsi Adduction Pectoralis major Latissimus dorsi Teres major Coracobrachialis Abduction Mid deltoid Supraspinatus Medial rotation Subscapularis Anterior deltoid Teres major Latissimus dorsi Lateral rotation Posterior deltoid Infraspinatus Teres minor Important anatomical relations
  • 43. Anteriorly Brachial plexus Axillary artery and vein Posterior Suprascapular nerve Suprascapular vessels Inferior Axillary nerve Circumflex humeral vessels hich of the following nerves is not contained within the posterior triangle of the neck? A. Accessory nerve B. Phrenic nerve C. Greater auricular nerve D. Ansa cervicalis E. Lesser occiptal nerve Next question Theme from September 2012 Exam Ansa cervicalis is a content of the anterior triangle of the neck. Posterior triangle of the neck Boundaries Apex Sternocleidomastoid and the Trapezius muscles at the Occipital bone
  • 44. Anterior Posterior border of the Sternocleidomastoid Posterior Anterior border of the Trapezius Base Middle third of the clavicle Image sourced from Wikipedia Contents Nerves  Accessory nerve  Phrenic nerve  Three trunks of the brachial plexus  Branches of the cervical plexus: Supraclavicular nerve, transverse cervical nerve, great auricular nerve, lesser occipital nerve Vessels  External jugular vein  Subclavian artery Muscles  Inferior belly of omohyoid  Scalene Lymph nodes  Supraclavicular
  • 45.  Occipital A 73 year old lady suffers a fracture at the surgical neck of the humerus. The decision is made to operate. There are difficulties in reducing the fracture and a vessel lying posterior to the surgical neck is injured. Which of the following is this vessel most likely to be? A. Axillary artery B. Brachial artery C. Thoracoacromial artery D. Transverse scapular artery E. Posterior circumflex humeral artery Next question The circumflex humeral arteries lie at the surgical neck and is this scenario the posterior circumflex is likely to be injured. The thoracoacromial and transverse scapular arteries lie more superomedially. The posterior circumflex humeral artery is a branch of the axillary artery. Shoulder joint  Shallow synovial ball and socket type of joint.
  • 46.  It is an inherently unstable joint, but is capable to a wide range of movement.  Stability is provided by muscles of the rotator cuff that pass from the scapula to insert in the greater tuberosity (all except sub scapularis-lesser tuberosity). Glenoid labrum  Fibrocartilaginous rim attached to the free edge of the glenoid cavity  Tendon of the long head of biceps arises from within the joint from the supraglenoid tubercle, and is fused at this point to the labrum.  The long head of triceps attaches to the infraglenoid tubercle Fibrous capsule  Attaches to the scapula external to the glenoid labrum and to the labrum itself (postero-superiorly)  Attaches to the humerus at the level of the anatomical neck superiorly and the surgical neck inferiorly  Anteriorly the capsule is in contact with the tendon of subscapularis, superiorly with the supraspinatus tendon, and posteriorly with the tendons of
  • 47. infraspinatus and teres minor. All these blend with the capsule towards their insertion.  Two defects in the fibrous capsule; superiorly for the tendon of biceps. Anteriorly there is a defect beneath the subscapularis tendon.  The inferior extension of the capsule is closely related to the axillary nerve at the surgical neck and this nerve is at risk in anteroinferior dislocations. It also means that proximally sited osteomyelitis may progress to septic arthritis. Movements and muscles Flexion Anterior part of deltoid Pectoralis major Biceps Coracobrachialis Extension Posterior deltoid Teres major Latissimus dorsi Adduction Pectoralis major Latissimus dorsi Teres major Coracobrachialis Abduction Mid deltoid Supraspinatus Medial rotation Subscapularis Anterior deltoid Teres major Latissimus dorsi
  • 48. Lateral rotation Posterior deltoid Infraspinatus Teres minor Important anatomical relations Anteriorly Brachial plexus Axillary artery and vein Posterior Suprascapular nerve Suprascapular vessels Inferior Axillary nerve Circumflex humeral vessels Which of the structures listed below lies posterior to the carotid sheath at the level of the 6th cervical vertebra? A. Hypoglossal nerve B. Vagus nerve C. Cervical sympathetic chain D. Ansa cervicalis E. Glossopharyngeal nerve Next question The carotid sheath is crossed anteriorly by the hypoglossal nerves and the ansa cervicalis. The vagus lies within it. The cervical sympathetic chain lies
  • 49. posteriorly between the sheath and the prevertebral fascia. Common carotid artery The right common carotid artery arises at the bifurcation of the brachiocephalic trunk, the left common carotid arises from the arch of the aorta. Both terminate at the level of the upper border of the thyroid cartilage (the lower border of the third cervical vertebra) by dividing into the internal and external carotid arteries. Left common carotid artery This vessel arises immediately to the left and slightly behind the origin of the brachiocephalic trunk. Its thoracic portion is 2.5- 3.5 cm in length and runs superolaterally to the sternoclavicular joint. In the thorax The vessel is in contact, from below upwards, with the trachea, left recurrent laryngeal nerve, left margin of the oesophagus. Anteriorly the left brachiocephalic vein runs across the artery, and the cardiac branches from the left vagus descend in front of it. These structures together with the thymus and the anterior margins of the left lung and pleura separate the artery from the manubrium. In the neck The artery runs superiorly deep to sternocleidomastoid
  • 50. and then enters the anterior triangle. At this point it lies within the carotid sheath with the vagus nerve and the internal jugular vein. Posteriorly the sympathetic trunk lies between the vessel and the prevertebral fascia. At the level of C7 the vertebral artery and thoracic duct lie behind it. The anterior tubercle of C6 transverse process is prominent and the artery can be compressed against this structure (it corresponds to the level of the cricoid). Anteriorly at C6 the omohyoid muscle passes superficial to the artery. Within the carotid sheath the jugular vein lies lateral to the artery. Right common carotid artery The right common carotid arises from the brachiocephalic artery. The right common carotid artery corresponds with the cervical portion of the left common carotid, except that there is no thoracic duct on the right. The oesophagus is less closely related to the right carotid than the left. Summary points about the carotid anatomy Path Passes behind the sternoclavicular joint (12% patients above this level) to the upper border of the thyroid cartilage, to divide into the external (ECA) and internal carotid arteries (ICA). Relations
  • 51.  Level of 6th cervical vertebra crossed by omohyoid  Then passes deep to the thyrohyoid, sternohyoid, sternomastoid muscles.  Passes behind the carotid tubercle (transverse process 6th cervical vertebra)-NB compression here stops haemorrhage.  The inferior thyroid artery passes posterior to the common carotid artery.  Then : Left common carotid artery crossed by thoracic duct, Right common carotid artery crossed by recurrent laryngeal nerve Image sourced from Wikipedia A 45 year old man presents with a lipoma located posterior to the posterior border of the
  • 52. sternocleidomastoid muscle, approximately 4cm superior to the middle third of the clavicle. During surgical excision of the lesion troublesome bleeding is encountered. Which of the following is the most likely source? A. Internal jugular vein B. External jugular vein C. Common carotid artery D. Vertebral artery E. Second part of the subclavian artery Next question The external jugular vein runs obliquely in the superficial fascia of the posterior triangle. It drains into the subclavian vein. During surgical exploration of this area the external jugular vein may be injured and troublesome bleeding may result. The internal jugular vein and carotid arteries are located in the anterior triangle. The third, and not the second, part of the subclavian artery is also a content of the posterior triangle Posterior triangle of the neck Boundaries Apex Sternocleidomastoid and the Trapezius muscles at the Occipital bone
  • 53. Anterior Posterior border of the Sternocleidomastoid Posterior Anterior border of the Trapezius Base Middle third of the clavicle Image sourced from Wikipedia Contents Nerves  Accessory nerve  Phrenic nerve  Three trunks of the brachial plexus  Branches of the cervical plexus: Supraclavicular nerve, transverse cervical nerve, great auricular nerve, lesser occipital nerve Vessels  External jugular vein  Subclavian artery Muscles  Inferior belly of omohyoid  Scalene Lymph nodes  Supraclavicular
  • 54.  Occipital Which of the following upper limb muscles is not innervated by the radial nerve? A. Extensor carpi ulnaris B. Abductor digit minimi C. Anconeus D. Supinator E. Brachioradialis Next question Mnemonic for radial nerve muscles: BEST B rachioradialis E xtensors S upinator T riceps Abductor digiti minimi is innervated by the ulnar nerve. Radial nerve Continuation of posterior cord of the brachial plexus (root values C5 to T1) Path  In the axilla: lies posterior to the axillary artery on subscapularis, latissimus dorsi and teres major.
  • 55.  Enters the arm between the brachial artery and the long head of triceps (medial to humerus).  Spirals around the posterior surface of the humerus in the groove for the radial nerve.  At the distal third of the lateral border of the humerus it then pierces the intermuscular septum and descends in front of the lateral epicondyle.  At the lateral epicondyle it lies deeply between brachialis and brachioradialis where it then divides into a superficial and deep terminal branch.  Deep branch crosses the supinator to become the posterior interosseous nerve. In the image below the relationships of the radial nerve can be appreciated Image sourced from Wikipedia Regions innervated Motor (main nerve)  Triceps  Anconeus  Brachioradialis
  • 56.  Extensor carpi radialis Motor (posterior interosseous branch  Extensor carpi ulnaris  Extensor digitorum  Extensor indicis  Extensor digiti minimi  Extensor pollicis longus and brevis  Abductor pollicis longus Sensory The area of skin supplying the proximal phalanges on the dorsal aspect of the hand is supplied by the radial nerve (this does not apply to the little finger and part of the ring finger) Muscular innervation and effect of denervation Anatomical location Muscle affected Effect of paralysis Shoulder Long head of triceps Minor effects on shoulder stability in abduction Arm Triceps Loss of elbow extension Forearm Supinator Brachioradialis Extensor carpi radialis longus and brevis Weakening of supination of prone hand and elbow flexion in mid prone position The cutaneous sensation of the upper limb- illustrating the contribution of the radial nerve
  • 57. Which of the following forms the floor of the anatomical snuffbox? A. Radial artery B. Cephalic vein C. Extensor pollicis brevis D. Scaphoid bone E. Cutaneous branch of the radial nerve Next question The scaphoid bone forms the floor of the anatomical snuffbox. The cutaneous branch of the radial nerve is much more superficially and proximally located. Anatomical snuffbox Posterior border Tendon of extensor pollicis longus
  • 58. Anterior border Tendons of extensor pollicis brevis and abductor pollicis longus Proximal border Styloid process of the radius Distal border Apex of snuffbox triangle Floor Trapezium and scaphoid Content Radial artery Image showing the anatomical snuffbox A 32 year old lady complains of carpal tunnel syndrome. The carpal tunnel is explored surgically. Which of the following structures will lie in closest proximity to the hamate bone within the carpal tunnel? A. The tendon of abductor pollicis longus B. The tendons of flexor digitorum profundus C. The tendons of flexor carpi radialis longus D. Median nerve E. Radial artery Next question The carpal tunnel contains nine flexor tendons:  Flexor digitorum profundus
  • 59.  Flexor digitorum superficialis  Flexor pollicis longus The tendon of flexor digitorum profundus lies deepest in the tunnel and will thus lie nearest to the hamate bone. Carpal bones Diagrammatic image of carpal bones Image sourced from Wikipedia Key to image A Scaphoid B Lunate C Triquetrum D Pisiform E Trapezium F Trapezoid G Capitate H Hamate
  • 60. 1 Radius 2 Ulna 3 Metacarpals  No tendons attach to: Scaphoid, lunate, triquetrum (stabilised by ligaments) A 45 year man presents with hand weakness. He is given a piece of paper to hold between his thumb and index finger. When the paper is pulled, the patient has difficulty maintaining a grip. Grip pressure is maintained by flexing the thumb at the interphalangeal joint. What is the most likely nerve lesion? A. Posterior interosseous nerve B. Deep branch of ulnar nerve C. Anterior interosseous nerve D. Superficial branch of the ulnar nerve E. Radial nerve Next question Theme from January 2012 exam This is a description of Froment's sign, which tests for ulnar nerve palsy. It mainly tests for the function of adductor pollicis. This is supplied by the deep branch of the ulnar nerve. Remember the anterior interosseous branch, which innervates the flexor pollicis longus (hence causing flexion of the thumb IP joint), branches off more proximally to the wrist. Ulnar nerve
  • 61. Origin  C8, T1 Supplies (no muscles in the upper arm)  Flexor carpi ulnaris  Flexor digitorum profundus  Flexor digiti minimi  Abductor digiti minimi  Opponens digiti minimi  Adductor pollicis  Interossei muscle  Third and fourth lumbricals  Palmaris brevis Path  Posteromedial aspect of ulna to flexor compartment of forearm, then along the ulnar. Passes beneath the flexor carpi ulnaris muscle, then superficially through the flexor retinaculum into the palm of the hand.
  • 62. Image sourced from Wikipedia Branches Branch Supplies Articular branch Flexor carpi ulnaris Medial half of the flexor digitorum profundus Palmar cutaneous branch (Arises near the middle of the forearm) Skin on the medial part of the palm Dorsal cutaneous branch Dorsal surface of the medial part of the hand Superficial branch Cutaneous fibres to the anterior surfaces of the medial one and one-half digits Deep branch Hypothenar muscles All the interosseous muscles Third and fourth lumbricals Adductor pollicis Medial head of the flexor pollicis brevis
  • 63. Effects of injury Damage at the wrist  Wasting and paralysis of intrinsic hand muscles (claw hand)  Wasting and paralysis of hypothenar muscles  Loss of sensation medial 1 and half fingers Damage at the elbow  Radial deviation of the wrist  Clawing less in 3rd and 4th digits A 10 year old by falls out of a tree has suffers a supracondylar fracture. He complains of a painful elbow and forearm. There is an obvious loss of pincer movement involving the thumb and index finger with minimal loss of sensation. The most likely nerve injury is to the: A. Ulnar nerve B. Radial nerve C. Anterior interosseous nerve D. Axillary nerve damage E. Median nerve damage above the elbow Next question The anterior interosseous nerve is a motor branch of the median nerve just below the elbow. When damaged it classically causes:  Pain in the forearm  Loss of pincer movement of the thumb and index finger (innervates the long flexor muscles of flexor pollicis longus & flexor digitorum profundus of the index and middle finger)  Minimal loss of sensation due to lack of a cutaneous branch
  • 64. Median nerve The median nerve is formed by the union of a lateral and medial root respectively from the lateral (C5,6,7) and medial (C8 and T1) cords of the brachial plexus; the medial root passes anterior to the third part of the axillary artery. The nerve descends lateral to the brachial artery, crosses to its medial side (usually passing anterior to the artery). It passes deep to the bicipital aponeurosis and the median cubital vein at the elbow. It passes between the two heads of the pronator teres muscle, and runs on the deep surface of flexor digitorum superficialis (within its fascial sheath). Near the wrist it becomes superficial between the tendons of flexor digitorum superficialis and flexor carpi radialis, deep to palmaris longus tendon. It passes deep to the flexor retinaculum to enter the palm, but lies anterior to the long flexor tendons within the carpal tunnel. Branches Region Branch Upper arm No branches, although the nerve commonly communicates with the musculocutaneous nerve Forearm Pronator teres Flexor carpi radialis Palmaris longus Flexor digitorum superficialis Flexor pollicis longus Flexor digitorum profundus (only the radial half) Distal forearm Palmar cutaneous branch Hand (Motor) Motor supply (LOAF)  Lateral 2 lumbricals  Opponens pollicis  Abductor pollicis brevis
  • 65.  Flexor pollicis brevis Hand (Sensory)  Over thumb and lateral 2 ½ fingers  On the palmar aspect this projects proximally, on the dorsal aspect only the distal regions are innervated with the radial nerve providing the more proximal cutaneous innervation. Patterns of damage Damage at wrist  e.g. carpal tunnel syndrome  paralysis and wasting of thenar eminence muscles and opponens pollicis (ape hand deformity)  sensory loss to palmar aspect of lateral (radial) 2 ½ fingers Damage at elbow, as above plus:  unable to pronate forearm  weak wrist flexion  ulnar deviation of wrist Anterior interosseous nerve (branch of median nerve)  leaves just below the elbow  results in loss of pronation of forearm and weakness of long flexors of thumb and index finger Topography of the median nerve
  • 66. Image sourced from Wikipedia A 32 year old attends neurology clinic complaining of tingling in his hand. He has radial deviation of his wrist and there is mild clawing of his fingers, with the 3rd and 4th digits being relatively spared. What is the most likely lesion? A. Ulnar nerve damage at the wrist B. Ulnar nerve damage at the elbow C. Radial nerve damage at the elbow D. Median nerve damage at the wrist E. Median nerve damage at the elbow Next question
  • 67. At the elbow the ulnar nerve lesion affects the flexor carpi ulnaris and flexor digitorum profundus. Ulnar nerve Origin  C8, T1 Supplies (no muscles in the upper arm)  Flexor carpi ulnaris  Flexor digitorum profundus  Flexor digiti minimi  Abductor digiti minimi  Opponens digiti minimi  Adductor pollicis  Interossei muscle  Third and fourth lumbricals  Palmaris brevis Path  Posteromedial aspect of ulna to flexor compartment of forearm, then along the ulnar. Passes beneath the flexor carpi ulnaris muscle, then superficially through the flexor retinaculum into the palm of the hand.
  • 68. Image sourced from Wikipedia Branches Branch Supplies Articular branch Flexor carpi ulnaris Medial half of the flexor digitorum profundus Palmar cutaneous branch (Arises near the middle of the forearm) Skin on the medial part of the palm Dorsal cutaneous branch Dorsal surface of the medial part of the hand Superficial branch Cutaneous fibres to the anterior surfaces of the medial one and one-half digits Deep branch Hypothenar muscles All the interosseous muscles Third and fourth lumbricals Adductor pollicis Medial head of the flexor pollicis brevis
  • 69. Effects of injury Damage at the wrist  Wasting and paralysis of intrinsic hand muscles (claw hand)  Wasting and paralysis of hypothenar muscles  Loss of sensation medial 1 and half fingers Damage at the elbow  Radial deviation of the wrist  Clawing less in 3rd and 4th digits A 23 year old man is involved in a fight and is stabbed in his upper arm. The ulnar nerve is transected. Which of the following muscles will not demonstrate compromised function as a result? A. Flexor carpi ulnaris B. Medial half of flexor digitorum profundus C. Palmaris brevis D. Hypothenar muscles E. Pronator teres Next question M edial lumbricals A dductor pollicis F lexor digitorum profundus/Flexor digiti minimi I nterossei A bductor digiti minimi and opponens Innervates all intrinsic muscles of the hand (EXCEPT 2: thenar muscles & first two lumbricals - supplied by median nerve)
  • 70. Pronator teres is innervated by the median nerve. Palmaris brevis is innervated by the ulnar nerve Ulnar nerve Origin  C8, T1 Supplies (no muscles in the upper arm)  Flexor carpi ulnaris  Flexor digitorum profundus  Flexor digiti minimi  Abductor digiti minimi  Opponens digiti minimi  Adductor pollicis  Interossei muscle  Third and fourth lumbricals  Palmaris brevis Path  Posteromedial aspect of ulna to flexor compartment of forearm, then along the ulnar. Passes beneath the flexor carpi ulnaris muscle, then superficially through the flexor retinaculum into the palm of the hand.
  • 71. Image sourced from Wikipedia Branches Branch Supplies Articular branch Flexor carpi ulnaris Medial half of the flexor digitorum profundus Palmar cutaneous branch (Arises near the middle of the forearm) Skin on the medial part of the palm Dorsal cutaneous branch Dorsal surface of the medial part of the hand Superficial branch Cutaneous fibres to the anterior surfaces of the medial one and one-half digits Deep branch Hypothenar muscles All the interosseous muscles Third and fourth lumbricals Adductor pollicis Medial head of the flexor pollicis brevis
  • 72. Effects of injury Damage at the wrist  Wasting and paralysis of intrinsic hand muscles (claw hand)  Wasting and paralysis of hypothenar muscles  Loss of sensation medial 1 and half fingers Damage at the elbow  Radial deviation of the wrist  Clawing less in 3rd and 4th digits Which of the structures listed below overlies the cephalic vein? A. Extensor retinaculum B. Bicipital aponeurosis C. Biceps muscle D. Antebrachial fascia E. None of the above Next question The cephalic vein is superficially located in the upper limb and overlies most the fascial planes. It pierces the coracoid membrane (continuation of the clavipectoral fascia) to terminate in the axillary vein. It lies anterolaterally to biceps. Cephalic vein Path  Dorsal venous arch drains laterally into the cephalic vein  Crosses the anatomical snuffbox and travels laterally up the arm  At the antecubital fossa connected to the basilic vein by the median cubital vein
  • 73.  Pierces deep fascia of deltopectoral groove to join axillary vein A 22 year old man is involved in a fight. He sustains a laceration to the posterior aspect of his wrist. In the emergency department the wound is explored and the laceration is found to be transversely orientated and overlies the region of the extensor retinaculum, which is intact. Which of the following structures is least likely to be injured in this scenario? A. Dorsal cutaneous branch of the ulnar nerve B. Tendon of extensor indicis C. Basilic vein D. Superficial branch of the radial nerve E. Cephalic vein Next question The extensor retinaculum attaches to the radius proximal to the styloid, thereafter it runs obliquely and distally to wind around the ulnar styloid (but does not attach to it). The extensor tendons lie deep to the extensor retinaculum and would therefore be less susceptible to injury than the superficial structures. Extensor retinaculum The extensor rentinaculum is a thickening of the deep fascia that stretches across the back of the wrist and holds the long extensor tendons in position. Its attachments are:  The pisiform and hook of hamate medially  The end of the radius laterally
  • 74. Structures related to the extensor retinaculum Structures superficial to the retinaculum  Basilic vein  Dorsal cutaneous branch of the ulnar nerve  Cephalic vein  Superficial branch of the radial nerve Structures passing deep to the extensor retinaculum  Extensor carpi ulnaris tendon  Extensor digiti minimi tendon  Extensor digitorum and extensor indicis tendon  Extensor pollicis longus tendon  Extensor carpi radialis longus tendon  Abductor pollicis longus and extensor pollicis brevis tendons Beneath the extensor retinaculum fibrous septa form six compartments that contain the extensor muscle tendons. Each compartment has its own synovial sheath. The radial artery The radial artery passes between the lateral collateral ligament of the wrist joint and the tendons of the abductor pollicis longus and extensor pollicis brevis. Image illustrating the topography of tendons passing under the extensor retinaculum
  • 75. A man has an incision sited than runs 8cm from the deltopectoral groove to the midline. Which of the following is not at risk of injury? A. Cephalic vein B. Shoulder joint capsule C. Axillary artery D. Pectoralis major E. Trunk of the brachial plexus Next question Theme from April 2012 Exam This region will typically lie medial to the joint capsule. The diagram below illustrates the plane that this would transect and as it can be appreciated the other structures are all at risk of injury.
  • 76. Image sourced from Wikipedia Pectoralis major muscle Origin From the medial two thirds of the clavicle, manubrium and sternocostal angle Insertion Crest of the greater tubercle of the humerus Nerve supply Lateral pectoral nerve Actions Adductor and medial rotator of the humerus heme: Nerve Injury
  • 77. A. Median nerve B. Ulnar nerve C. Radial nerve D. Musculocutaneous nerve E. Axillary nerve F. Anterior interosseous nerve G. Posterior interosseous nerve For each scenario please select the most likely underlying nerve injury. Each option may be used once, more than once or not at all. 171. A 19 year old student is admitted to A&E after falling off a wall. He is unable to flex his index finger. An x-ray confirms a supracondylar fracture. You answered Anterior interosseous nerve The correct answer is Median nerve This median nerve is at risk during a supracondylar fracture. 172. A well toned weight lifter attends clinic reporting weakness of his left arm. There is weakness of flexion and supination of the forearm. You answered Posterior interosseous nerve The correct answer is Musculocutaneous nerve Mucocutaneous nerve compression due to entrapment of the nerve between biceps and brachialis. Elbow flexion and supination of the arm are affected. This is a rare isolated injury.
  • 78. 173. An 18 year old girl sustains an Holstein-Lewis fracture. Which nerve is at risk? You answered Axillary nerve The correct answer is Radial nerve Proximal lesions affect the triceps. Also paralysis of wrist extensors and forearm supinators occur. Reduced sensation of dorsoradial aspect of hand and dorsal 31/2 fingers. Holstein-Lewis fractures are fractures of the distal humerus with radial nerve entrapment. A 35 year old farm labourer is injures the posterior aspect of his hand with a mechanical scythe. He severs some of his extensor tendons in this injury. How many tunnels lie in the extensor retinaculum that transmit the tendons of the extensor muscles? A. One B. Three C. Four D. Five E. Six Next question There are six tunnels, each lined by its own synovial sheath. Extensor retinaculum The extensor rentinaculum is a thickening of the deep fascia that stretches across the back of the wrist and holds the long extensor tendons in position. Its attachments are:  The pisiform and hook of hamate medially  The end of the radius laterally
  • 79. Structures related to the extensor retinaculum Structures superficial to the retinaculum  Basilic vein  Dorsal cutaneous branch of the ulnar nerve  Cephalic vein  Superficial branch of the radial nerve Structures passing deep to the extensor retinaculum  Extensor carpi ulnaris tendon  Extensor digiti minimi tendon  Extensor digitorum and extensor indicis tendon  Extensor pollicis longus tendon  Extensor carpi radialis longus tendon  Abductor pollicis longus and extensor pollicis brevis tendons Beneath the extensor retinaculum fibrous septa form six compartments that contain the extensor muscle tendons. Each compartment has its own synovial sheath. The radial artery The radial artery passes between the lateral collateral ligament of the wrist joint and the tendons of the abductor pollicis longus and extensor pollicis brevis. Image illustrating the topography of tendons passing under the extensor retinaculum
  • 80. A 23 year old man is injured during a game of rugby. He suffers a fracture of the distal third of his clavicle, it is a compound fracture and there is evidence of arterial haemorrhage. Which of the following vessels is most likely to be encountered first during subsequent surgical exploration? A. Posterior circumflex humeral artery B. Axillary artery C. Thoracoacromial artery D. Sub scapular artery E. Lateral thoracic artery Next question Similar theme in September 2011 Exam
  • 81. The thoracoacromial artery arises from the second part of the axillary artery. It is a short, wide trunk, which pierces the clavipectoral fascia, and ends, deep to pectoralis major by dividing into four branches. Thoracoacromial artery The thoracoacromial artery (acromiothoracic artery; thoracic axis) is a short trunk, which arises from the forepart of the axillary artery, its origin being generally overlapped by the upper edge of the Pectoralis minor. Projecting forward to the upper border of the Pectoralis minor, it pierces the coracoclavicular fascia and divides into four branches: pectoral, acromial, clavicular, and deltoid. Branch Description Pectoral branch Descends between the two Pectoral muscles, and is distributed to them and to the breast, anastomosing with the intercostal branches of the internal thoracic artery and with the lateral thoracic. Acromial branch Runs laterally over the coracoid process and under the Deltoid, to which it gives branches; it then pierces that muscle and ends on the acromion in an arterial network formed by branches from the suprascapular, thoracoacromial, and posterior humeral circumflex arteries. Clavicular branch Runs upwards and medially to the sternoclavicular joint, supplying this articulation, and the Subclavius Deltoid branch Arising with the acromial, it crosses over the Pectoralis minor and passes in the same groove as the cephalic vein, between the Pectoralis major and Deltoid, and gives branches to both muscles. 68 year old man falls onto an outstretched hand. Following the accident he is examined in the emergency department. On palpating his anatomical snuffbox there is tenderness noted in the base. What is the most likely injury in this scenario? A. Rupture of the tendon of flexor pollicis
  • 82. B. Scaphoid fracture C. Distal radius fracture D. Rupture of flexor carpi ulnaris tendon E. None of the above Next question A fall onto an outstretched hand is a common mechanism of injury for a scaphoid fracture. This should be suspected clinically if there is tenderness in the base of the anatomical snuffbox. A tendon rupture would not result in bony tenderness. Scaphoid bone The scaphoid has a concave articular surface for the head of the capitate and at the edge of this is a crescentic surface for the corresponding area on the lunate. Proximally, it has a wide convex articular surface with the radius. It has a distally sited tubercle that can be palpated. The remaining articular surface is to the lateral side of the tubercle. It faces laterally and is associated with the trapezium and trapezoid bones. The narrow strip between the radial and trapezial surfaces and the tubercle gives rise to the radial collateral carpal ligament. The tubercle receives part of the flexor retinaculum. This area is the only part of the scaphoid that is available for the entry of blood vessels. It is commonly fractured and avascular necrosis may result. Scaphoid bone
  • 83. Which of the following structures passes through the quadrangular space near the humeral head? A. Axillary artery B. Radial nerve C. Axillary nerve D. Median nerve E. Transverse scapular artery Next question The quadrangular space is bordered by the humerus laterally, subscapularis superiorly, teres major inferiorly and the long head of triceps medially. It lies lateral to the triangular space. It transmits the axillary nerve and posterior circumflex humeral artery.
  • 84. Image sourced from Wikipedia Shoulder joint  Shallow synovial ball and socket type of joint.  It is an inherently unstable joint, but is capable to a wide range of movement.  Stability is provided by muscles of the rotator cuff that pass from the scapula to insert in the greater tuberosity (all except sub scapularis-lesser tuberosity). Glenoid labrum  Fibrocartilaginous rim attached to the free edge of the glenoid cavity  Tendon of the long head of biceps arises from within the joint from the supraglenoid tubercle, and is fused at this point to the labrum.  The long head of triceps attaches to the infraglenoid tubercle Fibrous capsule
  • 85.  Attaches to the scapula external to the glenoid labrum and to the labrum itself (postero-superiorly)  Attaches to the humerus at the level of the anatomical neck superiorly and the surgical neck inferiorly  Anteriorly the capsule is in contact with the tendon of subscapularis, superiorly with the supraspinatus tendon, and posteriorly with the tendons of infraspinatus and teres minor. All these blend with the capsule towards their insertion.  Two defects in the fibrous capsule; superiorly for the tendon of biceps. Anteriorly there is a defect beneath the subscapularis tendon.  The inferior extension of the capsule is closely related to the axillary nerve at the surgical neck and this nerve is at risk in anteroinferior dislocations. It also means that proximally sited osteomyelitis may progress to septic arthritis. Movements and muscles Flexion Anterior part of deltoid Pectoralis major Biceps Coracobrachialis Extension Posterior deltoid Teres major Latissimus dorsi Adduction Pectoralis major Latissimus dorsi Teres major Coracobrachialis Abduction Mid deltoid Supraspinatus Medial rotation Subscapularis Anterior deltoid Teres major Latissimus dorsi Lateral rotation Posterior deltoid
  • 86. Infraspinatus Teres minor Important anatomical relations Anteriorly Brachial plexus Axillary artery and vein Posterior Suprascapular nerve Suprascapular vessels Inferior Axillary nerve Circumflex humeral vessels Which of the following structures separates the ulnar artery from the median nerve? A. Brachioradialis B. Pronator teres C. Tendon of biceps brachii D. Flexor carpi ulnaris E. Brachialis Next question It lies deep to pronator teres and this separates it from the median nerve. Ulnar artery Path  Starts: middle of antecubital fossa  Passes obliquely downward, reaching the ulnar side of the forearm at a point about midway between the elbow and the wrist. It follows the ulnar border to the wrist, crossing over the
  • 87. flexor retinaculum. It then divides into the superficial and deep volar arches. Relations Deep to- Pronator teres, Flexor carpi radialis, Palmaris longus Lies on- Brachialis and Flexor digitorum profundus Superficial to the flexor retinaculum at the wrist The median nerve is in relation with the medial side of the artery for about 2.5 cm. And then crosses the vessel, being separated from it by the ulnar head of the Pronator teres The ulnar nerve lies medially to the lower two-thirds of the artery Branch  Anterior interosseous artery
  • 88. A 32 year old motorcyclist is involved in a road traffic accident. His humerus is fractured and severely displaced. At the time of surgical repair the surgeon notes that the radial nerve has been injured. Which of the following muscles is least likely to be affected by an injury at this site? A. Extensor carpi radialis brevis B. Brachioradialis C. Abductor pollicis longus D. Extensor pollicis brevis E. None of the above Next question
  • 89. Muscles supplied by the radial nerve BEST Brachioradialis Extensors Supinator Triceps The radial nerve supplies the extensor muscles, abductor pollicis longus and extensor pollicis brevis (the latter two being innervated by the posterior interosseous branch of the radial nerve). Radial nerve Continuation of posterior cord of the brachial plexus (root values C5 to T1) Path  In the axilla: lies posterior to the axillary artery on subscapularis, latissimus dorsi and teres major.  Enters the arm between the brachial artery and the long head of triceps (medial to humerus).  Spirals around the posterior surface of the humerus in the groove for the radial nerve.  At the distal third of the lateral border of the humerus it then pierces the intermuscular septum and descends in front of the lateral epicondyle.  At the lateral epicondyle it lies deeply between brachialis and brachioradialis where it then divides into a superficial and deep terminal branch.  Deep branch crosses the supinator to become the posterior interosseous nerve.
  • 90. In the image below the relationships of the radial nerve can be appreciated Image sourced from Wikipedia Regions innervated Motor (main nerve)  Triceps  Anconeus  Brachioradialis  Extensor carpi radialis Motor (posterior interosseous branch  Extensor carpi ulnaris  Extensor digitorum  Extensor indicis  Extensor digiti minimi  Extensor pollicis longus and brevis  Abductor pollicis longus Sensory The area of skin supplying the proximal phalanges on the dorsal aspect of the hand is supplied by the radial nerve (this does not apply to the little finger and part of the ring finger)
  • 91. Muscular innervation and effect of denervation Anatomical location Muscle affected Effect of paralysis Shoulder Long head of triceps Minor effects on shoulder stability in abduction Arm Triceps Loss of elbow extension Forearm Supinator Brachioradialis Extensor carpi radialis longus and brevis Weakening of supination of prone hand and elbow flexion in mid prone position The cutaneous sensation of the upper limb- illustrating the contribution of the radial nerve Image sourced from Wikipedia Which muscle is responsible for causing flexion of the interphalangeal joint of the thumb? A. Flexor pollicis longus
  • 92. B. Flexor pollicis brevis C. Flexor digitorum superficialis D. Flexor digitorum profundus E. Adductor pollicis Next question There are 8 muscles: 1. Two flexors (flexor pollicis brevis and flexor pollicis longus) 2. Two extensors (extensor pollicis brevis and longus) 3. Two abductors (abductor pollicis brevis and longus) 4. One adductor (adductor pollicis) 5. One muscle that opposes the thumb by rotating the CMC joint (opponens pollicis). Flexor and extensor longus insert on the distal phalanx moving both the MCP and IP joints. Hand Anatomy of the hand Bones  8 Carpal bones  5 Metacarpals  14 phalanges Intrinsic Muscles 7 Interossei - Supplied by ulnar nerve  3 palmar-adduct fingers  4 dorsal- abduct fingers Intrinsic muscles Lumbricals  Flex MCPJ and extend the IPJ.  Origin deep flexor tendon and insertion dorsal extensor hood mechanism.
  • 93.  Innervation: 1st and 2nd- median nerve, 3rd and 4th- deep branch of the ulnar nerve. Thenar eminence  Abductor pollicis brevis  Opponens pollicis  Flexor pollicis brevis Hypothenar eminence  Opponens digiti minimi  Flexor digiti minimi brevis  Abductor digiti minimi Image sourced from Wikipedia n 18 year old man is stabbed in the axilla during a fight. His axillary artery is lacerated and repaired. However, the surgeon neglects to repair an associated injury to the upper trunk of the brachial plexus. Which of the following muscles is least likely to demonstrate impaired function as a result? A. Palmar interossei
  • 94. B. Infraspinatus C. Brachialis D. Supinator brevis E. None of the above Next question The palmar interossei are supplied by the ulnar nerve. Which lies inferiorly and is therefore less likely to be injured. Brachial plexus Origin Anterior rami of C5 to T1 Sections of the plexus  Roots, trunks, divisions, cords, branches  Mnemonic:Real Teenagers Drink Cold Beer Roots  Located in the posterior triangle  Pass between scalenus anterior and medius Trunks  Located posterior to middle third of clavicle  Upper and middle trunks related superiorly to the subclavian artery  Lower trunk passes over 1st rib posterior to the subclavian artery Divisions Apex of axilla Cords Related to axillary artery Diagram illustrating the branches of the brachial plexus
  • 95. Image sourced from Wikipedia Cutaneous sensation of the upper limb Image sourced from Wikipedia 23 year old man is involved in a fight, during the dispute he sustains a laceration to the posterior aspect of his right arm, approximately 2cm proximal to the olecranon process. On assessment in the
  • 96. emergency department he is unable to extend his elbow joint. Which of the following tendons is most likely to have been cut? A. Triceps B. Pronator teres C. Brachioradialis D. Brachialis E. Biceps Next question Theme from 2009 Exam The triceps muscle extends the elbow joint. The other muscles listed all produce flexion of the elbow joint. Triceps Origin  Long head- infraglenoid tubercle of the scapula.  Lateral head- dorsal surface of the humerus, lateral and proximal to the groove of the radial nerve  Medial head- posterior surface of the humerus on the inferomedial side of the radial groove and both of the intermuscular septae Insertion  Olecranon process of the ulna. Here the olecranon bursa is between the triceps tendon and olecranon.  Some fibres insert to the deep fascia of the forearm, posterior capsule of the elbow (preventing the capsule from being trapped between olecranon and olecranon fossa during extension) Innervation Radial nerve Blood supply Profunda brachii artery Action Elbow extension. The long head can adduct the humerus and and extend it
  • 97. from a flexed position Relations The radial nerve and profunda brachii vessels lie between the lateral and medial heads Which of the following muscles does not attach to the radius? A. Pronator quadratus B. Biceps C. Brachioradialis D. Supinator E. Brachialis Next question The brachialis muscle inserts into the ulna. The other muscles are all inserted onto the radius. Radius  Bone of the forearm extending from the lateral side of the elbow to the thumb side of the wrist Upper end  Articular cartilage- covers medial > lateral side  Articulates with radial notch of the ulna by the annular ligament  Muscle attachment- biceps brachii at the tuberosity Shaft  Muscle attachment-
  • 98. Upper third of the body Supinator, Flexor digitorum superficialis, Flexor pollicis longus Middle third of the body Pronator teres Lower quarter of the body Pronator quadratus , tendon of supinator longus Lower end  Quadrilateral  Anterior surface- capsule of wrist joint  Medial surface- head of ulna  Lateral surface- ends in the styloid process  Posterior surface: 3 grooves containing: 1. Tendons of extensor carpi radialis longus and brevis 2. Tendon of extensor pollicis longus 3. Tendon of extensor indicis
  • 99. Which of the following is not an intrinsic muscle of the hand? A. Opponens pollicis B. Palmaris longus C. Flexor pollicis brevis D. Flexor digiti minimi brevis E. Opponens digiti minimi
  • 100. Next question Mnemonic for intrinsic hand muscles 'A OF A OF A' A bductor pollicis brevis O pponens pollicis F lexor pollicis brevis A dductor pollicis (thenar muscles) O pponens digiti minimi F lexor digiti minimi brevis A bductor digiti minimi (hypothenar muscles) Palmaris longus originates in the forearm. Hand Anatomy of the hand Bones  8 Carpal bones  5 Metacarpals  14 phalanges Intrinsic Muscles 7 Interossei - Supplied by ulnar nerve  3 palmar-adduct fingers  4 dorsal- abduct fingers Intrinsic muscles Lumbricals  Flex MCPJ and extend the IPJ.  Origin deep flexor tendon and insertion dorsal extensor hood mechanism.  Innervation: 1st and 2nd- median nerve, 3rd and 4th- deep branch of the ulnar nerve. Thenar eminence  Abductor pollicis brevis
  • 101.  Opponens pollicis  Flexor pollicis brevis Hypothenar eminence  Opponens digiti minimi  Flexor digiti minimi brevis  Abductor digiti minimi Image sourced from Wikipedia A 28 year old man lacerates the posterolateral aspect of his wrist with a knife in an attempted suicide. On arrival in the emergency department the wound is inspected and found to be located over the lateral aspect of the extensor retinaculum (which is intact). Which of the following structures is at greatest risk of injury? A. Superficial branch of the radial nerve B. Radial artery C. Dorsal branch of the ulnar nerve
  • 102. D. Tendon of extensor carpi radialis brevis E. Tendon of extensor digiti minimi Next question The superficial branch of the radial nerve passes superior to the extensor retinaculum in the position of this laceration and is at greatest risk of injury. The dorsal branch of the ulnar nerve and artery also pass superior to the extensor retinaculum n but are located medially. Extensor retinaculum The extensor rentinaculum is a thickening of the deep fascia that stretches across the back of the wrist and holds the long extensor tendons in position. Its attachments are:  The pisiform and hook of hamate medially  The end of the radius laterally Structures related to the extensor retinaculum Structures superficial to the retinaculum  Basilic vein  Dorsal cutaneous branch of the ulnar nerve  Cephalic vein  Superficial branch of the radial nerve Structures passing deep to the extensor retinaculum  Extensor carpi ulnaris tendon  Extensor digiti minimi tendon  Extensor digitorum and extensor indicis tendon  Extensor pollicis longus tendon  Extensor carpi radialis longus tendon  Abductor pollicis longus and extensor pollicis brevis tendons
  • 103. Beneath the extensor retinaculum fibrous septa form six compartments that contain the extensor muscle tendons. Each compartment has its own synovial sheath. The radial artery The radial artery passes between the lateral collateral ligament of the wrist joint and the tendons of the abductor pollicis longus and extensor pollicis brevis. Image illustrating the topography of tendons passing under the extensor retinaculum ransection of the radial nerve at the level of the axilla will result in all of the following except: A. Loss of elbow extension.
  • 104. B. Loss of extension of the interphalangeal joints. C. Loss of metacarpophalangeal extension. D. Loss of triceps reflex. E. Loss of sensation overlying the first dorsal interosseous. Next question These may still extend by virtue of retained lumbrical muscle function. Radial nerve Continuation of posterior cord of the brachial plexus (root values C5 to T1) Path  In the axilla: lies posterior to the axillary artery on subscapularis, latissimus dorsi and teres major.  Enters the arm between the brachial artery and the long head of triceps (medial to humerus).  Spirals around the posterior surface of the humerus in the groove for the radial nerve.  At the distal third of the lateral border of the humerus it then pierces the intermuscular septum and descends in front of the lateral epicondyle.  At the lateral epicondyle it lies deeply between brachialis and brachioradialis where it then divides into a superficial and deep terminal branch.  Deep branch crosses the supinator to become the posterior interosseous nerve.
  • 105. In the image below the relationships of the radial nerve can be appreciated Image sourced from Wikipedia Regions innervated Motor (main nerve)  Triceps  Anconeus  Brachioradialis  Extensor carpi radialis Motor (posterior interosseous branch  Extensor carpi ulnaris  Extensor digitorum  Extensor indicis  Extensor digiti minimi  Extensor pollicis longus and brevis  Abductor pollicis longus Sensory The area of skin supplying the proximal phalanges on the dorsal aspect of the hand is supplied by the radial nerve (this does not apply to the little finger and part of the ring finger)
  • 106. Muscular innervation and effect of denervation Anatomical location Muscle affected Effect of paralysis Shoulder Long head of triceps Minor effects on shoulder stability in abduction Arm Triceps Loss of elbow extension Forearm Supinator Brachioradialis Extensor carpi radialis longus and brevis Weakening of supination of prone hand and elbow flexion in mid prone position The cutaneous sensation of the upper limb- illustrating the contribution of the radial nerve Image sourced from Wikipedia A. Teres major
  • 107. B. Pectoralis major C. Coracobrachialis D. Supraspinatus E. Latissimus dorsi Next question Supraspinatus is an abductor of the shoulder. Shoulder joint  Shallow synovial ball and socket type of joint.  It is an inherently unstable joint, but is capable to a wide range of movement.  Stability is provided by muscles of the rotator cuff that pass from the scapula to insert in the greater tuberosity (all except sub scapularis-lesser tuberosity). Glenoid labrum  Fibrocartilaginous rim attached to the free edge of the glenoid cavity  Tendon of the long head of biceps arises from within the joint from the supraglenoid tubercle, and is fused at this point to the labrum.  The long head of triceps attaches to the infraglenoid tubercle Fibrous capsule  Attaches to the scapula external to the glenoid labrum and to the labrum itself (postero-superiorly)
  • 108.  Attaches to the humerus at the level of the anatomical neck superiorly and the surgical neck inferiorly  Anteriorly the capsule is in contact with the tendon of subscapularis, superiorly with the supraspinatus tendon, and posteriorly with the tendons of infraspinatus and teres minor. All these blend with the capsule towards their insertion.  Two defects in the fibrous capsule; superiorly for the tendon of biceps. Anteriorly there is a defect beneath the subscapularis tendon.  The inferior extension of the capsule is closely related to the axillary nerve at the surgical neck and this nerve is at risk in anteroinferior dislocations. It also means that proximally sited osteomyelitis may progress to septic arthritis. Movements and muscles Flexion Anterior part of deltoid Pectoralis major Biceps Coracobrachialis Extension Posterior deltoid Teres major Latissimus dorsi Adduction Pectoralis major Latissimus dorsi Teres major Coracobrachialis Abduction Mid deltoid Supraspinatus Medial rotation Subscapularis Anterior deltoid Teres major Latissimus dorsi Lateral rotation Posterior deltoid Infraspinatus Teres minor
  • 109. Important anatomical relations Anteriorly Brachial plexus Axillary artery and vein Posterior Suprascapular nerve Suprascapular vessels Inferior Axillary nerve Circumflex humeral vessels low? A. External jugular B. Axillary C. Internal jugular D. Azygos E. Brachial Next question Cephalic vein Path  Dorsal venous arch drains laterally into the cephalic vein  Crosses the anatomical snuffbox and travels laterally up the arm  At the antecubital fossa connected to the basilic vein by the median cubital vein  Pierces deep fascia of deltopectoral groove to join axillary vein  Which of the following structures is not closely related to the brachial artery?
  • 110. A. Ulnar nerve B. Median nerve C. Cephalic vein D. Long head of triceps E. Median cubital vein  Next question  The cephalic vein lies superficially and on the contralateral side of the arm to the brachial artery. The relation of the ulnar nerves and others are demonstrated in the image below: Image sourced from Wikipedia   Brachial artery  The brachial artery begins at the lower border of teres major as a continuation of the axillary artery. It terminates in the cubital fossa at the level of the neck of the radius by dividing into the radial and ulnar arteries. Relations Posterior relations include the long head of triceps with the radial nerve and profunda vessels intervening. Anteriorly it is overlapped by the medial border of biceps.
  • 111. It is crossed by the median nerve in the middle of the arm. In the cubital fossa it is separated from the median cubital vein by the bicipital aponeurosis. The basilic vein is in contact at the most proximal aspect of the cubital fossa and lies medially. The following statements relating to the musculocutaneous nerve are true except? A. It arises from the lateral cord of the brachial plexus B. It provides cutaneous innervation to the lateral side of the forearm C. If damaged then extension of the elbow joint will be impaired D. It supplies the biceps muscle E. It runs beneath biceps Next question It supplies biceps, brachialis and coracobrachialis so if damaged then elbow flexion will be impaired. Musculocutaneous nerve  Branch of lateral cord of brachial plexus Path  It penetrates the Coracobrachialis muscle  Passes obliquely between the Biceps brachii and the Brachialis to the lateral side of the arm  Above the elbow it pierces the deep fascia lateral to the tendon of the Biceps brachii
  • 112.  Continues into the forearm as the lateral cutaneous nerve of the forearm Innervates  Coracobrachialis  Biceps brachii  Brachialis Which ligament keeps the head of the radius connected to the radial notch of the ulna? A. Annular (orbicular) ligament B. Quadrate ligament C. Radial collateral ligament of the elbow D. Ulnar collateral ligament E. Radial collateral ligament Next question The annular ligament connects the radial head to the radial notch of the ulna. This is illustrated below:
  • 113. Image sourced from Wikipedia Radius  Bone of the forearm extending from the lateral side of the elbow to the thumb side of the wrist Upper end  Articular cartilage- covers medial > lateral side  Articulates with radial notch of the ulna by the annular ligament  Muscle attachment- biceps brachii at the tuberosity Shaft  Muscle attachment-
  • 114. Upper third of the body Supinator, Flexor digitorum superficialis, Flexor pollicis longus Middle third of the body Pronator teres Lower quarter of the body Pronator quadratus , tendon of supinator longus Lower end  Quadrilateral  Anterior surface- capsule of wrist joint  Medial surface- head of ulna  Lateral surface- ends in the styloid process  Posterior surface: 3 grooves containing: 1. Tendons of extensor carpi radialis longus and brevis 2. Tendon of extensor pollicis longus 3. Tendon of extensor indicis
  • 115. Image sourced from Wikipedia A 38 year old man presents to the clinic with shoulder weakness. On examination he has an inability to initiate shoulder abduction. Which of the nerves listed below is least likely to be functioning normally? A. Suprascapular nerve B. Medial pectoral nerve C. Axillary nerve
  • 116. D. Median nerve E. Radial nerve Next question Theme from April 2012 Exam Suprascapular nerve The suprascapular nerve arises from the upper trunk of the brachial plexus. It lies superior to the trunks of the brachial plexus and passes inferolaterally parallel to them. It passes through the scapular notch, deep to trapezius. It innervates both supraspinatus and infraspinatus and initiates abduction of the shoulder. If damaged, patients may be able to abduct the shoulder by leaning over the affected side and deltoid can then continue to abduct the shoulder.
  • 117. Which of the following vessels provides the greatest contribution to the arterial supply of the breast? A. External mammary artery B. Thoracoacromial artery C. Internal mammary artery D. Lateral thoracic artery E. Subclavian artery
  • 118. Next question 60% of the arterial supply to the breast is derived from the internal mammary artery. The external mammary and lateral thoracic arteries also make a significant (but lesser) contribution. This is of importance clinically in performing reduction mammoplasty procedures. Breast The breast itself lies on a layer of pectoral fascia and the following muscles: 1. Pectoralis major 2. Serratus anterior 3. External oblique Image showing the topography of the female breast Image sourced from Wikipedia Breast anatomy Nerve supply Branches of intercostal nerves from T4-T6. Arterial supply  Internal mammary (thoracic) artery
  • 119.  External mammary artery (laterally)  Anterior intercostal arteries  Thoraco-acromial artery Venous drainage Superficial venous plexus to sub clavian, axillary and intercostal veins. Lymphatic drainage  70% Axillary nodes  Internal mammary chain  Other lymphatic sites such as deep cervical and supraclavicular fossa (later in disease) A baby is found to have a Klumpke's palsy post delivery. Which of the following is most likely to be present? A. Loss of flexors of the wrist B. Weak elbow flexion C. Pronation of the forearm D. Adducted shoulder E. Shoulder medially rotated Next question Features of Klumpkes Paralysis  Claw hand (MCP joints extended and IP joints flexed)  Loss of sensation over medial aspect of forearm and hand  Horner's syndrome  Loss of flexors of the wrist A C8, T1 root lesion is called Klumpke's paralysis and is caused by delivery with the arm extended.
  • 120. Brachial plexus Origin Anterior rami of C5 to T1 Sections of the plexus  Roots, trunks, divisions, cords, branches  Mnemonic:Real Teenagers Drink Cold Beer Roots  Located in the posterior triangle  Pass between scalenus anterior and medius Trunks  Located posterior to middle third of clavicle  Upper and middle trunks related superiorly to the subclavian artery  Lower trunk passes over 1st rib posterior to the subclavian artery Divisions Apex of axilla Cords Related to axillary artery Diagram illustrating the branches of the brachial plexus Image sourced from Wikipedia Cutaneous sensation of the upper limb
  • 121. ith respect to the basilic vein, which statement is false? A. Its deep anatomical location makes it unsuitable for use as an arteriovenous access site in fistula surgery B. It originates from the dorsal venous network on the hand C. It travels up the medial aspect of the forearm D. Halfway between the shoulder and the elbow it lies deep to muscle E. It joins the brachial vein to form the axillary vein Next question It is used in arteriovenous fistula surgery during a procedure known as a basilic vein transposition. Basilic vein
  • 122. The basilic and cephalic veins both provide the main pathways of venous drainage for the arm and hand. It is continuous with the palmar venous arch distally and the axillary vein proximally. Path  Originates on the medial side of the dorsal venous network of the hand, and passes up the forearm and arm.  Most of its course is superficial.  Near the region anterior to the cubital fossa the vein joins the cephalic vein.  Midway up the humerus the basilic vein passes deep under the muscles.  At the lower border of the teres major muscle, the anterior and posterior circumflex humeral veins feed into it.  Joins the brachial veins to form the axillary vein. A 78 year old man is lifting a heavy object when a feels a pain in his forearm and is unable to continue. He has a swelling over his upper forearm. An MRI scan shows a small cuff of tendon still attached to the radial tuberosity consistent with a recent tear. Which of the following muscles has been injured? A. Pronator teres B. Supinator C. Aconeus D. Brachioradialis E. Biceps brachii Next question Biceps inserts into the radial tuberosity. Distal injuries of this muscle
  • 123. are rare but are reported and are clinically more important than more proximal ruptures. Radius  Bone of the forearm extending from the lateral side of the elbow to the thumb side of the wrist Upper end  Articular cartilage- covers medial > lateral side  Articulates with radial notch of the ulna by the annular ligament  Muscle attachment- biceps brachii at the tuberosity Shaft  Muscle attachment- Upper third of the body Supinator, Flexor digitorum superficialis, Flexor pollicis longus Middle third of the body Pronator teres Lower quarter of the body Pronator quadratus , tendon of supinator longus Lower end  Quadrilateral  Anterior surface- capsule of wrist joint  Medial surface- head of ulna  Lateral surface- ends in the styloid process  Posterior surface: 3 grooves containing:
  • 124. 1. Tendons of extensor carpi radialis longus and brevis 2. Tendon of extensor pollicis longus 3. Tendon of extensor indicis Which of the following is a branch of the third part of the axillary artery? A. Superior thoracic B. Lateral thoracic
  • 125. C. Dorsal scapular D. Thoracoacromial E. Posterior circumflex humeral Next question The other branches include:  Subscapular  Anterior circumflex humeral Axilla Boundaries of the axilla Medially Chest wall and Serratus anterior Laterally Humeral head Floor Subscapularis Anterior aspect Lateral border of Pectoralis major Fascia Clavipectoral fascia Content: Long thoracic nerve (of Bell) Derived from C5-C7 and passes behind the brachial plexus to enter the axilla. It lies on the medial chest wall and supplies serratus anterior. Its location puts it at risk during axillary surgery and damage will lead to winging of the scapula. Thoracodorsal nerve and thoracodorsal trunk Innervate and vascularise latissimus dorsi. Axillary vein Lies at the apex of the axilla, it is the continuation of the basilic vein. Becomes the subclavian vein at the outer border of the first rib. Intercostobrachial Traverse the axillary lymph nodes and are often divided during
  • 126. nerves axillary surgery. They provide cutaneous sensation to the axillary skin. Lymph nodes The axilla is the main site of lymphatic drainage for the breast. Theme: Nerve lesions A. Intercostobrachial B. Median C. Axillary D. Radial E. Ulnar F. Musculocutaneous G. Brachial plexus upper cord H. Brachial plexus lower cord Please select the most likely nerve injury for the scenarios given. Each option may be used once, more than once or not at all. 268. A 23 year old rugby player sustains a Smiths Fracture. On examination opposition of the thumb is markedly weakened. Median This high velocity injury can often produce significant angulation and displacement. Both of these may impair the function of the median nerve with loss of function of the muscles of the thenar eminence 269. A 45 year old lady recovering from a mastectomy and axillary node clearance notices that sensation in her armpit is impaired. Intercostobrachial The intercostobrachial nerves are frequently injured during axillary
  • 127. dissection. These nerves traverse the axilla and supply cutaneous sensation. 270. An 8 year old boy falls onto an outstretched hand and sustains a supracondylar fracture. In addition to a weak radial pulse the child is noted to have loss of pronation of the affected hand. Median This is a common injury in children. In this case the angulation and displacement have resulted in median nerve injury. Next question Brachial plexus Origin Anterior rami of C5 to T1 Sections of the plexus  Roots, trunks, divisions, cords, branches  Mnemonic:Real Teenagers Drink Cold Beer Roots  Located in the posterior triangle  Pass between scalenus anterior and medius Trunks  Located posterior to middle third of clavicle  Upper and middle trunks related superiorly to the subclavian artery  Lower trunk passes over 1st rib posterior to the subclavian artery Divisions Apex of axilla Cords Related to axillary artery Diagram illustrating the branches of the brachial plexus
  • 128. Image sourced from Wikipedia Cutaneous sensation of the upper limb Image sourced from Wikipedia froment's test which muscle function is tested? A. Flexor pollicis longus
  • 129. B. Adductor pollicis longus C. Abductor pollicis brevis D. Adductor pollicis E. Opponens pollicis Next question Nerve signs Froment's sign  Assess for ulnar nerve palsy  Adductor pollicis muscle function tested  Hold a piece of paper between their thumb and index finger. The object is then pulled away. If ulnar nerve palsy, unable to hold the paper and will flex the flexor pollicis longus to compensate (flexion of thumb at interphalangeal joint). Phalen's test  Assess carpal tunnel syndrome  More sensitive than Tinel's sign  Hold wrist in maximum flexion and the test is positive if there is numbness in the median nerve distribution. Tinel's sign  Assess for carpal tunnel syndrome
  • 130.  Tap the median nerve at the wrist and the test is positive if there is tingling/electric-like sensations over the distribution of the median nerve. heme: Cutaneous innervation A. Ulnar nerve B. Fifth cervical spinal segment C. Radial nerve D. Musculocutaneous nerve E. Median nerve F. None of these Please select the source of innervation for the region described. Each option may be used once, more than once or not at all. 288. The skin on the palmar aspect of the thumb Median nerve The median nerve supplies cutaneous sensation to this region. See diagram below 289. The nail bed of the index finger You answered Radial nerve The correct answer is Median nerve 290. The skin overlying the medial aspect of the palm Ulnar nerve
  • 131. This area is innervated by the ulnar nerve. Next question Brachial plexus Origin Anterior rami of C5 to T1 Sections of the plexus  Roots, trunks, divisions, cords, branches  Mnemonic:Real Teenagers Drink Cold Beer Roots  Located in the posterior triangle  Pass between scalenus anterior and medius Trunks  Located posterior to middle third of clavicle  Upper and middle trunks related superiorly to the subclavian artery  Lower trunk passes over 1st rib posterior to the subclavian artery Divisions Apex of axilla Cords Related to axillary artery Diagram illustrating the branches of the brachial plexus
  • 132. Image sourced from Wikipedia Cutaneous sensation of the upper limb From which of the following foramina does the opthalmic branch of the trigeminal nerve exit the skull? A. Foramen ovale